Miten kreationismi selittää globaalin tulvan vesimäärän?

Anonyymi-ap

Kreationistit yrittävät selittää Raamatun tulvatarua ikäänkuin se voisi olla todellisuudessa tapahtunut. Koska kyseessä on mielikuvituksellinen tarina, myös selityksiä rajoittaa vain kertojan mielikuvitus. Kerätäänpähän tähän noita selityksiä, jotta nähdään hieman niiden kirjoa.

Lähtötietoina on ainakin se, että vesi peitti kaikki maat ja korkeimmat vuoret, vesi tuli sateena 40 päivän ajan ja vuoden päästä vedet laskivat "normaali" tasolle.

1. Vesi oli varastoitunut taivaankanteen, josta se satoi alas. Vettä tulikin aika reippaasti muutama kilometri 40 päivässä, kun kovimmat todellisuuden rankkasattet mitataan kymmenissä senteissä.

2. Vesi tuli maan syvyyksistä. Muutaman kilometrin paksuinen vesimassa pullahtaa maasta ylös ilman geologista katastrogia. Jotkut kretuthan olivat kuulleet maaperässä olevasta vedestä ja heti oli tulvan selitys valmiina.

3. Vesi pulppuaa kuuman lähteen tavoin ilmoille. Kilometrien kuuma vesi olisi käräyttänyt Nooan porukan kaiken muun ohessa.

4. Maan ennen tulvaan oli tasainen ja vuoret pikakohosi sen jälkeen. Tiedämme geologiasta, että moinen olisi aiheuttanut globaalisti mm. järkyttävät tulivuoren purkaukset ja Nooan porukka olisi kärvistellyt vuosikymmeniä ydintalvessa. Ainakaan Raamattu ei tätä tue, jiten nämä vääntäö jo Raamatun sanaakin.

5. Maan kuori aaltoili tulvan aikoihin asteroudin törmäyksen seurauksena. Tämä viittaa edelliseen antaen selityksen, miksi vuoret pikakohosi. Toki sellainen asteroidi olisi hävittänyt taas Nooan porukan.

6. Vesi haihtui tulvan jälkeen taivaalle. Toisaalta olikin laskettu, että veden haihtuminen avaruuteen olisi vastinut lömpötilan, jossa myös Nooa arkkeineen olisi höyrystynyt. Sehän ei olisi voinut vain hsihtu sateeksi.

7. Tulvan jälkeen luonto jatkoi ikäänkuin tulvaa ei olisi ollutkaan, vaikka suolavesi tekee maasta suurelle osalle kasvillisuutta elinkelvottoman. Myös vuosi suolavedessä kilometrien syvyydessä olisi tappanut kasvillisuuden ilman auringon valoa.

Tässä näitä muutamia kreationistisen "luonnontieteen" ihmeitä. Laitoin alustavasti myös ihan terveellä järjellä päätellen, mitä oikeassa todellisuudessa kyseinen fantasia tarkoittaisi.

Onkohan joku törmännyt vielä muihin fantasioihin tulvatarun tiimoilta?

194

1123

    Vastaukset

    Anonyymi (Kirjaudu / Rekisteröidy)
    5000
    • Anonyymi

      Etkö sinä ole luonnontieteellinen ihme? Osaat varmaan kertoa, miten ja mistä sinuun on vedet ja muut ainekset ilmestyneet ja rakentuneet ihmisen muotoon.

      Jos et osaa, on turha kuvitella, että tämä kaikki muu olisi selitettävissä ns. terveellä järjellä.

      • Anonyymi

        Eli nostat kädet pystyyn etkä osaa mitenkään selittää vedenpaisumuksen veden ilmestymistä ja katoamista?


      • Anonyymi

        Ne ainekset ovat kaikki syntyneet tähtien nukleosynteesissä, supernovien räjähdyksissä ja valkeiden kääpiöiden törmäyksissä. Alkuaineet, josta muuta ovat syntyneet, ovat vety ja helium, jotka syntyivät alkuräjähdyksessä.
        Ihmisen muotoon rakentuminen tunnetaan hyvin.

        Näissä ei ole mitään järjelle selittämätöntä tai yliluonnollista, kuten vedenpaisumus vaatisi selityksekseen. Vedenpaisumustarua on aivan turha yrittää rinnastaa nukleosynteesiin tai evoluutioon.


      • Anonyymi
        Anonyymi kirjoitti:

        Eli nostat kädet pystyyn etkä osaa mitenkään selittää vedenpaisumuksen veden ilmestymistä ja katoamista?

        Ehkä samaan tapaan kuin juon vettä ja pissaan imeytymättömän osan ulos. Ei minulla muuta selitystä ole.

        Siinä veden katoamisessa oli kyse samasta asiasta. Maata kostutettiin, jotta siitä tuli elävämpää maanviljelykselle. Jostain maa sai vettä juodakseen, jotta tämä käsittämätön ihmiseksi kutsuttu olento pystyi maata viljelemään.


      • Anonyymi
        Anonyymi kirjoitti:

        Ne ainekset ovat kaikki syntyneet tähtien nukleosynteesissä, supernovien räjähdyksissä ja valkeiden kääpiöiden törmäyksissä. Alkuaineet, josta muuta ovat syntyneet, ovat vety ja helium, jotka syntyivät alkuräjähdyksessä.
        Ihmisen muotoon rakentuminen tunnetaan hyvin.

        Näissä ei ole mitään järjelle selittämätöntä tai yliluonnollista, kuten vedenpaisumus vaatisi selityksekseen. Vedenpaisumustarua on aivan turha yrittää rinnastaa nukleosynteesiin tai evoluutioon.

        No jopas on kattava selitys.


      • Anonyymi
        Anonyymi kirjoitti:

        Ne ainekset ovat kaikki syntyneet tähtien nukleosynteesissä, supernovien räjähdyksissä ja valkeiden kääpiöiden törmäyksissä. Alkuaineet, josta muuta ovat syntyneet, ovat vety ja helium, jotka syntyivät alkuräjähdyksessä.
        Ihmisen muotoon rakentuminen tunnetaan hyvin.

        Näissä ei ole mitään järjelle selittämätöntä tai yliluonnollista, kuten vedenpaisumus vaatisi selityksekseen. Vedenpaisumustarua on aivan turha yrittää rinnastaa nukleosynteesiin tai evoluutioon.

        Eikö tuohon riitä vastata, että äidistäni pääosin?

        Ehkä kreationisti ei tiedä miten ihminen syntyy ja luulee taika Jimin taikovan valmiita ihmisiä?


      • Anonyymi

        Heti yx kretu yrittää kääntää puheenaiheen hankalasta aiheesta pois. Kas kun ei alkanut evoluutioahdistustaan tässä purkamaan.


      • Anonyymi
        Anonyymi kirjoitti:

        No jopas on kattava selitys.

        Paljon kattavampi kuin mihin kreationistit pystyvät vedenpaisumukseen tarvittavien vesien selittämisessä.


      • Anonyymi
        Anonyymi kirjoitti:

        Ehkä samaan tapaan kuin juon vettä ja pissaan imeytymättömän osan ulos. Ei minulla muuta selitystä ole.

        Siinä veden katoamisessa oli kyse samasta asiasta. Maata kostutettiin, jotta siitä tuli elävämpää maanviljelykselle. Jostain maa sai vettä juodakseen, jotta tämä käsittämätön ihmiseksi kutsuttu olento pystyi maata viljelemään.

        Tuskinpa itsekään uskot, että kusi-selityksessäsi voisi olla järjen häivää.

        Raamatun mukaan maa oli viljelyskelpoista jo ennen tulvaa.
        Maa saa kosteutta säännöllisistä sateista. Kertakastelu ei auta pitkälle.
        Suolainen vesi tekee maasta useimmille kasveille kasvukelvottoman.


    • Anonyymi

      Ei ne vedet ole minnekään kadonneet. /0% maapallon pinnasta on nytkin veden peitossa. Ne vedet riittää hyvin peittämmän maapallon 3 kilomtriä, kuten sinulle on sanottu.
      Jos maa on ollut sadeveden kyllästämä, se ei ole imenyt juurikaan suolaa itseensä.
      Sitäpaitsi kaikki ei ollut veden peitossa samanaikaisesti, vesihän nousi vähitellen puoli vuotta..

      Ja tietysti vesi suihkusi ylös maankuoren halkeamien kohdalta, kun se pääsi magman kanssa kosketukseen. Se näkyi kuumina vesisuihkuina siellä täällä, eikä niin kuin jotkut esittävät, että kerran oli kaikki mantereet yhdellä puolella maapalloa, jolloin valtameri kiehui tietenkin siellä missä' ei ollut mannerlaattaa magman päällä !!

      Arkii oli turvallinen paikka veden noustessa ja laskiessa. Ei kai se muuten olisi edes juuttunut Araratin vuoristoon, niinkuin muinaiset historioitsijatkin kertovat.

      • Anonyymi

        Ahaa. Veljet mikä välkky. Sinun fysiikan oppien mukaan mantereiden päällä olisi vaikkapa 8 kilometrin vesikerros, mutta meren pinta olisi edelleen nykyisellä tasolla? Siis tyhjenikö meret ja vedet siirtyi mantereiden päälle? Heh, heh, vieläkin naurattaa. :D

        Lisää ihmeessä tuohon listaan tämä. Tämä on ihan uusi näkökulma.

        Ai niin mutta eihän tämäkään käy raamatun mukaan, koska silloin meren pohja näkyisi.


      • Anonyymi
        Anonyymi kirjoitti:

        Ahaa. Veljet mikä välkky. Sinun fysiikan oppien mukaan mantereiden päällä olisi vaikkapa 8 kilometrin vesikerros, mutta meren pinta olisi edelleen nykyisellä tasolla? Siis tyhjenikö meret ja vedet siirtyi mantereiden päälle? Heh, heh, vieläkin naurattaa. :D

        Lisää ihmeessä tuohon listaan tämä. Tämä on ihan uusi näkökulma.

        Ai niin mutta eihän tämäkään käy raamatun mukaan, koska silloin meren pohja näkyisi.

        Mistä sinä sait 8 kilometriä? Omasta päästäsi!

        Kuten on aikaisemmin sinulle kerrottu, maan kuoren on tarvinnut painua tai nousta alle promillen verran maan säteestä, jotta koko maapallo on tullut veden peittämäksi, ilman että vettä on tarvinnut olla yhtään enemmän.

        Uskotko itse että mannerlaatat olivat kerran yhdellä puolella maapalloa, jättäen toiselle puolelle vedet kiehumaan magman päälle?


      • Anonyymi
        Anonyymi kirjoitti:

        Mistä sinä sait 8 kilometriä? Omasta päästäsi!

        Kuten on aikaisemmin sinulle kerrottu, maan kuoren on tarvinnut painua tai nousta alle promillen verran maan säteestä, jotta koko maapallo on tullut veden peittämäksi, ilman että vettä on tarvinnut olla yhtään enemmän.

        Uskotko itse että mannerlaatat olivat kerran yhdellä puolella maapalloa, jättäen toiselle puolelle vedet kiehumaan magman päälle?

        Promille on 6 kilometriä siirrosta vertikaalusesti, mitä yrität ilmeisesti noilla sun promille-jutuilla häivyttää. On sulla kummat fantasiat. Et kykene lainkaan suhteuttamaan noita sun juttu todellisuuteen.


      • Anonyymi
        Anonyymi kirjoitti:

        Mistä sinä sait 8 kilometriä? Omasta päästäsi!

        Kuten on aikaisemmin sinulle kerrottu, maan kuoren on tarvinnut painua tai nousta alle promillen verran maan säteestä, jotta koko maapallo on tullut veden peittämäksi, ilman että vettä on tarvinnut olla yhtään enemmän.

        Uskotko itse että mannerlaatat olivat kerran yhdellä puolella maapalloa, jättäen toiselle puolelle vedet kiehumaan magman päälle?

        "Uskotko itse että mannerlaatat olivat kerran yhdellä puolella maapalloa, jättäen toiselle puolelle vedet kiehumaan ..."

        En, koska tuo on sinun tai jonkun toisen idioottikretun keksimä hölmö olkiukko, jota sinä nyt nylkytät. Tuo on oikeastaan säälittävintä, mitä kretut tekee: keksii jonkun järjettömyyden, sitten härskisti valehtelee tieteen väittävän omaa valhettaan ja sitten haukkuu tieteen perustuen oman valheeseen.


      • Anonyymi
        Anonyymi kirjoitti:

        Mistä sinä sait 8 kilometriä? Omasta päästäsi!

        Kuten on aikaisemmin sinulle kerrottu, maan kuoren on tarvinnut painua tai nousta alle promillen verran maan säteestä, jotta koko maapallo on tullut veden peittämäksi, ilman että vettä on tarvinnut olla yhtään enemmän.

        Uskotko itse että mannerlaatat olivat kerran yhdellä puolella maapalloa, jättäen toiselle puolelle vedet kiehumaan magman päälle?

        8 kilometriä ja 14 kasitonnista - eikös se Raamatun kertomus sano, että kaikki maat ofittyi veteen. Voidaan toki sopia, että 5 kilsaa riittää.

        Raamatun ei muuten kerro mitään järkyttävistä maanjäristyksistä, joita tuollainen kilometrien siirros maankuoressa aiheuttaa. Ettet nyt vaan keksi loruja omasta päästäsi ja häpäiset Raamatun sanaa?


      • Anonyymi

        Maapallon pinta ei ole eikä ole koskaan ollut täysin tasainen. Ja kun myös uskotte, että eläimet on älykkäästi suunniteltu, niin miten syvänmereneläimet on suunniteltu, jos maapallo ei ollut jo merieläimiä luotaessa epätasainen syvine merialueineen? Ja kun Raamattu sanoo, että vedenpaisumus peitti "korkeimmat vuoretkin", niin Raamatun tarinassakin siis oli siis sekä matalempia että korkampia vuoria.

        Jos makealla vedellä kyllästetyn maan päälle laitetaan suolaista, niin tietenkin suolapitoisuus pyrkii tasaantumaan suola liukenee makeaan veteen ja maaperästä tulee suolaista. Vielä kun suolaton vesi on kevyempää kuin suolainen, niin se pyrkii nousemaan yläs ja korvautuu suolaisella vedellä.

        Arkin kokoinen puulaiva ei ole turvallinen, koska se ei pysy tiiviinä, koska sen runko ei ole kyllin jäykkä. Kun pidätte videoista, niin katso tuo ja opi miten kävi maailman suurimmalle puulaivalle, kuunari Wyomingille, huolimatta että siinä oli 90 rautatankoa jäykistämässä laivan runkoa:
        https://www.youtube.com/watch?v=bKKkeMeGxmo

        Jotta vesi voisi suihkuta, sitä pitäisi ensin olla jossakin? Jos vedenpaisumuksen vesi oli enimmäkseen kiehuvan kuumaa, niin teidän tarinanne käy aina vain mahdottomammaksi. Ilmakehään vettä on sitoutunut määrä joka vastaa n. 2cm sademäärää. Joitakin kilometrejä kiehuvaa vettä tekisi olot arkissakin aika hikiseksi tai paremminkin kypsyttäväksi.


      • Anonyymi
        Anonyymi kirjoitti:

        8 kilometriä ja 14 kasitonnista - eikös se Raamatun kertomus sano, että kaikki maat ofittyi veteen. Voidaan toki sopia, että 5 kilsaa riittää.

        Raamatun ei muuten kerro mitään järkyttävistä maanjäristyksistä, joita tuollainen kilometrien siirros maankuoressa aiheuttaa. Ettet nyt vaan keksi loruja omasta päästäsi ja häpäiset Raamatun sanaa?

        ofittyi == peittyi

        ... keleen virtuaalinäppis ja automaattinen tekstin syöttö


      • Anonyymi
        Anonyymi kirjoitti:

        Maapallon pinta ei ole eikä ole koskaan ollut täysin tasainen. Ja kun myös uskotte, että eläimet on älykkäästi suunniteltu, niin miten syvänmereneläimet on suunniteltu, jos maapallo ei ollut jo merieläimiä luotaessa epätasainen syvine merialueineen? Ja kun Raamattu sanoo, että vedenpaisumus peitti "korkeimmat vuoretkin", niin Raamatun tarinassakin siis oli siis sekä matalempia että korkampia vuoria.

        Jos makealla vedellä kyllästetyn maan päälle laitetaan suolaista, niin tietenkin suolapitoisuus pyrkii tasaantumaan suola liukenee makeaan veteen ja maaperästä tulee suolaista. Vielä kun suolaton vesi on kevyempää kuin suolainen, niin se pyrkii nousemaan yläs ja korvautuu suolaisella vedellä.

        Arkin kokoinen puulaiva ei ole turvallinen, koska se ei pysy tiiviinä, koska sen runko ei ole kyllin jäykkä. Kun pidätte videoista, niin katso tuo ja opi miten kävi maailman suurimmalle puulaivalle, kuunari Wyomingille, huolimatta että siinä oli 90 rautatankoa jäykistämässä laivan runkoa:
        https://www.youtube.com/watch?v=bKKkeMeGxmo

        Jotta vesi voisi suihkuta, sitä pitäisi ensin olla jossakin? Jos vedenpaisumuksen vesi oli enimmäkseen kiehuvan kuumaa, niin teidän tarinanne käy aina vain mahdottomammaksi. Ilmakehään vettä on sitoutunut määrä joka vastaa n. 2cm sademäärää. Joitakin kilometrejä kiehuvaa vettä tekisi olot arkissakin aika hikiseksi tai paremminkin kypsyttäväksi.

        "Jos makealla vedellä kyllästetyn maan päälle laitetaan suolaista, niin tietenkin suolapitoisuus pyrkii tasaantumaan ..."

        Hyvä huomio.


      • Anonyymi
        Anonyymi kirjoitti:

        Maapallon pinta ei ole eikä ole koskaan ollut täysin tasainen. Ja kun myös uskotte, että eläimet on älykkäästi suunniteltu, niin miten syvänmereneläimet on suunniteltu, jos maapallo ei ollut jo merieläimiä luotaessa epätasainen syvine merialueineen? Ja kun Raamattu sanoo, että vedenpaisumus peitti "korkeimmat vuoretkin", niin Raamatun tarinassakin siis oli siis sekä matalempia että korkampia vuoria.

        Jos makealla vedellä kyllästetyn maan päälle laitetaan suolaista, niin tietenkin suolapitoisuus pyrkii tasaantumaan suola liukenee makeaan veteen ja maaperästä tulee suolaista. Vielä kun suolaton vesi on kevyempää kuin suolainen, niin se pyrkii nousemaan yläs ja korvautuu suolaisella vedellä.

        Arkin kokoinen puulaiva ei ole turvallinen, koska se ei pysy tiiviinä, koska sen runko ei ole kyllin jäykkä. Kun pidätte videoista, niin katso tuo ja opi miten kävi maailman suurimmalle puulaivalle, kuunari Wyomingille, huolimatta että siinä oli 90 rautatankoa jäykistämässä laivan runkoa:
        https://www.youtube.com/watch?v=bKKkeMeGxmo

        Jotta vesi voisi suihkuta, sitä pitäisi ensin olla jossakin? Jos vedenpaisumuksen vesi oli enimmäkseen kiehuvan kuumaa, niin teidän tarinanne käy aina vain mahdottomammaksi. Ilmakehään vettä on sitoutunut määrä joka vastaa n. 2cm sademäärää. Joitakin kilometrejä kiehuvaa vettä tekisi olot arkissakin aika hikiseksi tai paremminkin kypsyttäväksi.

        Ensiksikään Raamattu ei puhu laivasta vaan arkista. Arkki on neliskanttinen laatikko, arkku, kuten kymmenen käskyä oli arkussa, siis liitonarkissa.

        Joten arkun saattoi rakentaa suorista puista, vaikka tekemällä pohjan parin metrin paksuiseksi, ja pikeämällä sen, jolloin arkku olisi varmaan kellunut vaikka olisi tullut vettä sisään.

        Sitten tuo vouhotus kuumasta vedestä: maapallolla on kapeita murtumakohtia mannerlaattojen välissä, ja sieltä täätä tulee vieläkin joko tulivuoren purkauksia, kuumia vesisuihkuja, tai mustaa savua syvältä meren pohjasta (black smokers). Eihän koko valtameret tietenkäöään ole kiehuneet, mutta jos vettä on valunut johonkin halkeamaan, se tietysti on suihkunnut ylös niinkuin kuumat lähteet tekevät yhä.

        Että ihmiset uskovat Pangeaan, on paljon typerämpää: Mannerlaatat yhdellä puolella maapalloa, ja toisella puolen ei mitään mannerlaattoja, vaan meret suoraan kuuman magman päällä! Täysin järjetöntä: Eivätkö he tajua, että jos mannerlaatta on vaikka sata kilometriä paksu, ja meri vaikka viisi kilometriä , niin se meri on laatan yläpuolella ja siirtyisi laatan mukana!
        Ai niin, unohdin, että se vesi tietysti valui laatalta pois. Mutta mihin? Maapallon koko kuori on laattojen peitossa, ei edes millin rakoa! Todella hauskaa tämä tiedemiesten järkeily!


      • Anonyymi
        Anonyymi kirjoitti:

        Ensiksikään Raamattu ei puhu laivasta vaan arkista. Arkki on neliskanttinen laatikko, arkku, kuten kymmenen käskyä oli arkussa, siis liitonarkissa.

        Joten arkun saattoi rakentaa suorista puista, vaikka tekemällä pohjan parin metrin paksuiseksi, ja pikeämällä sen, jolloin arkku olisi varmaan kellunut vaikka olisi tullut vettä sisään.

        Sitten tuo vouhotus kuumasta vedestä: maapallolla on kapeita murtumakohtia mannerlaattojen välissä, ja sieltä täätä tulee vieläkin joko tulivuoren purkauksia, kuumia vesisuihkuja, tai mustaa savua syvältä meren pohjasta (black smokers). Eihän koko valtameret tietenkäöään ole kiehuneet, mutta jos vettä on valunut johonkin halkeamaan, se tietysti on suihkunnut ylös niinkuin kuumat lähteet tekevät yhä.

        Että ihmiset uskovat Pangeaan, on paljon typerämpää: Mannerlaatat yhdellä puolella maapalloa, ja toisella puolen ei mitään mannerlaattoja, vaan meret suoraan kuuman magman päällä! Täysin järjetöntä: Eivätkö he tajua, että jos mannerlaatta on vaikka sata kilometriä paksu, ja meri vaikka viisi kilometriä , niin se meri on laatan yläpuolella ja siirtyisi laatan mukana!
        Ai niin, unohdin, että se vesi tietysti valui laatalta pois. Mutta mihin? Maapallon koko kuori on laattojen peitossa, ei edes millin rakoa! Todella hauskaa tämä tiedemiesten järkeily!

        "Todella hauskaa tämä tiedemiesten järkeily!"

        He todellakin tekevät sen itse!!!


      • Anonyymi
        Anonyymi kirjoitti:

        Ensiksikään Raamattu ei puhu laivasta vaan arkista. Arkki on neliskanttinen laatikko, arkku, kuten kymmenen käskyä oli arkussa, siis liitonarkissa.

        Joten arkun saattoi rakentaa suorista puista, vaikka tekemällä pohjan parin metrin paksuiseksi, ja pikeämällä sen, jolloin arkku olisi varmaan kellunut vaikka olisi tullut vettä sisään.

        Sitten tuo vouhotus kuumasta vedestä: maapallolla on kapeita murtumakohtia mannerlaattojen välissä, ja sieltä täätä tulee vieläkin joko tulivuoren purkauksia, kuumia vesisuihkuja, tai mustaa savua syvältä meren pohjasta (black smokers). Eihän koko valtameret tietenkäöään ole kiehuneet, mutta jos vettä on valunut johonkin halkeamaan, se tietysti on suihkunnut ylös niinkuin kuumat lähteet tekevät yhä.

        Että ihmiset uskovat Pangeaan, on paljon typerämpää: Mannerlaatat yhdellä puolella maapalloa, ja toisella puolen ei mitään mannerlaattoja, vaan meret suoraan kuuman magman päällä! Täysin järjetöntä: Eivätkö he tajua, että jos mannerlaatta on vaikka sata kilometriä paksu, ja meri vaikka viisi kilometriä , niin se meri on laatan yläpuolella ja siirtyisi laatan mukana!
        Ai niin, unohdin, että se vesi tietysti valui laatalta pois. Mutta mihin? Maapallon koko kuori on laattojen peitossa, ei edes millin rakoa! Todella hauskaa tämä tiedemiesten järkeily!

        Se oliko arkissa virtaviivaista keulaa vai ei, ei vaikuta siihen, että puun jäykkyys ei riitä. Raamatussa mittasuhteet ovat pituus: 300, leveys: 50 ja korkeus 30. Yksikkö jää hämäräksi, mutta yleisemmin kyynärä on n. 50 cm eli arkki olisi ollut 150 x 25 x 15 metriä. Maailman suurimman puulaivan mitat olivat: 140m x 15m x 10m. Kun rungosta tehdään leveämpi eikä käytetä metallia tukemassa, niin puulaivan jäykkyys ei millään riitä.

        Toinen kysymys on, että miten 4000 vuotta sitten olisi pystytty rakentamaan suurempi puulaiva kuin tämän päivän tekniikalla.

        Vedenpaisumuksessa vettä "syvyyden lähteistä" tarvittiin kilometritolkulla, koska sateellahan ei saa aikaan kuin paikallisia tulvia. Jos kaikki ilmakehän ja pilvien sisältämä vesihöyry sataisi alas, niin meret nousisivat muutaman sentin. Tuolla yksi oman elämänsä vedenpaisumusasiantuntija (vai sinä) selitti, että "vesi suihkusi ylös maankuoren halkeamien kohdalta, kun se pääsi magman kanssa kosketukseen". Silloin se olisi ollut kiehuvan kuumaa ja kun siät olisi tarvittu niin paljon, että merien pinta olisi noussut kilometritolkulla, niin meret olisivat olleet niin kuumia, että kalat olisivat kuolleet, ennen kuin ne olisivat ehtineet kiivetä Himalajalle turvaan.

        On täysin typerää ja järjetöntä kouhottaa asiasta josta ei ymmärrä pätkääkään ja josta ei uskalla edes ottaa selvää, kun pelkää joutuvansa helvettiin jos ymmärtää mistä puhuu. "Mannerlaatat" ovat epämääräinen termi ja oikeampi olisi puhua litosfäärilaatoista, jotka siis peittävät koko maapallon eivätkä vain sitä aluetta, jossa on kuivaa maata. Esimerkiksi Tyynenmeren laatta on liki kokonaan meren peitossa.
        https://fi.wikipedia.org/wiki/Litosfäärilaatta#/media/Tiedosto:Plates_tect2_fi.svg


      • Anonyymi
        Anonyymi kirjoitti:

        Mistä sinä sait 8 kilometriä? Omasta päästäsi!

        Kuten on aikaisemmin sinulle kerrottu, maan kuoren on tarvinnut painua tai nousta alle promillen verran maan säteestä, jotta koko maapallo on tullut veden peittämäksi, ilman että vettä on tarvinnut olla yhtään enemmän.

        Uskotko itse että mannerlaatat olivat kerran yhdellä puolella maapalloa, jättäen toiselle puolelle vedet kiehumaan magman päälle?

        Meri magman päällä on näitä yecn harhoja, onhan valtameren allakin parin km paksuinen maankuori...


      • Anonyymi
        Anonyymi kirjoitti:

        Ensiksikään Raamattu ei puhu laivasta vaan arkista. Arkki on neliskanttinen laatikko, arkku, kuten kymmenen käskyä oli arkussa, siis liitonarkissa.

        Joten arkun saattoi rakentaa suorista puista, vaikka tekemällä pohjan parin metrin paksuiseksi, ja pikeämällä sen, jolloin arkku olisi varmaan kellunut vaikka olisi tullut vettä sisään.

        Sitten tuo vouhotus kuumasta vedestä: maapallolla on kapeita murtumakohtia mannerlaattojen välissä, ja sieltä täätä tulee vieläkin joko tulivuoren purkauksia, kuumia vesisuihkuja, tai mustaa savua syvältä meren pohjasta (black smokers). Eihän koko valtameret tietenkäöään ole kiehuneet, mutta jos vettä on valunut johonkin halkeamaan, se tietysti on suihkunnut ylös niinkuin kuumat lähteet tekevät yhä.

        Että ihmiset uskovat Pangeaan, on paljon typerämpää: Mannerlaatat yhdellä puolella maapalloa, ja toisella puolen ei mitään mannerlaattoja, vaan meret suoraan kuuman magman päällä! Täysin järjetöntä: Eivätkö he tajua, että jos mannerlaatta on vaikka sata kilometriä paksu, ja meri vaikka viisi kilometriä , niin se meri on laatan yläpuolella ja siirtyisi laatan mukana!
        Ai niin, unohdin, että se vesi tietysti valui laatalta pois. Mutta mihin? Maapallon koko kuori on laattojen peitossa, ei edes millin rakoa! Todella hauskaa tämä tiedemiesten järkeily!

        Mitä ihmettä sinä mahdat selittää tuossa viimeisessä kappaleessa, en tajunnut?!
        Edelleen, meri on aina jonkin maankuoren osan päällä. Mutta voithan sinä jatkaa jankkaamistasi magmasta...


    • Anonyymi

      Tänä päivänäkin on olemassa selittämätöntä massaa eli pimeää ainetta.
      Siksi Ajattelen että vesi saattaa olla tiedostamattomasta, mystisestä ja tutkimattomasta massasta lähtöisin.

      Myös paratiisi on ilmeisesti massaan kätketty ja siellä on syvyyden lähteitä. Eli vesivirtoja.

      Relativistinen massa?

      3 kvarkkia joista protoni muodostuu gluonien kera on noin 10 MeV mutta protoni on itse 939 MeV.

      Onhan noita tutkimattomia fysiikan salaisuuksia vielä jäljellä.

      Vanha vedestä rakennettu maailma tuhottiin vedellä. Pimeä aine.
      Nykyinen tulesta rakennettu maailma tuhotaan tulella. Pimeä energia.

      Käytössä on nuo kaksi tuhoamistapaa.
      Pimeä aine ja Pimeä energia.

      • Anonyymi

        Tämä myös listalle. Tulvatarun vesi oli pimeää ainetta.

        Kretulta hyvä veto sikäli, että liitetään Raamattu ohuesti johonkin tieteelliseen käsitteeseen, joka on mieluusti huonosti tunnettu vielä toistaiseksi.


      • Anonyymi
        Anonyymi kirjoitti:

        Tämä myös listalle. Tulvatarun vesi oli pimeää ainetta.

        Kretulta hyvä veto sikäli, että liitetään Raamattu ohuesti johonkin tieteelliseen käsitteeseen, joka on mieluusti huonosti tunnettu vielä toistaiseksi.

        Salli eri näkökulmien tulla esiin! Ei tietenkään jokainen kreationisti usko samalla tavalla.

        Anna ihmisillä olla mielipiteensä, äläkä yleistä!


      • Anonyymi
        Anonyymi kirjoitti:

        Salli eri näkökulmien tulla esiin! Ei tietenkään jokainen kreationisti usko samalla tavalla.

        Anna ihmisillä olla mielipiteensä, äläkä yleistä!

        Ei tietenkään ole tarkoitus rajoittaa mielipiteitä. Mutta kun tässä on kysymys siitä, että tulvatarun väitetään tapahtuneen todellisuudessa, niin silloin täytyisi olla rinnakkaistidellisuuksia. Tai sitten kertojan pitäisi ilmaista selvästi, että kyseessä on oma mielipide tai fantasia.

        Nythän täällä härskeimmät väittää oman mielipiteen kumoavan suunnilleen kaikki luonnontieteet; milloin menee fysiikka, milloin geologia uusiksi joidenkin horinoissa. Pienellä pintaraaputtelulla vaikkapa sivulliselle lukijalle näytetään näiden mielipiteiden todellinen luonne.


      • Anonyymi
        Anonyymi kirjoitti:

        Tämä myös listalle. Tulvatarun vesi oli pimeää ainetta.

        Kretulta hyvä veto sikäli, että liitetään Raamattu ohuesti johonkin tieteelliseen käsitteeseen, joka on mieluusti huonosti tunnettu vielä toistaiseksi.

        Pimeä aine on vettä! Siinäpä ratkesi suuri kosmologian ongelma. Ei kun Nobelia hakemaan tukholmasta. Menetkö Siljalla vai Viikkarilla?


      • Anonyymi
        Anonyymi kirjoitti:

        Ei tietenkään ole tarkoitus rajoittaa mielipiteitä. Mutta kun tässä on kysymys siitä, että tulvatarun väitetään tapahtuneen todellisuudessa, niin silloin täytyisi olla rinnakkaistidellisuuksia. Tai sitten kertojan pitäisi ilmaista selvästi, että kyseessä on oma mielipide tai fantasia.

        Nythän täällä härskeimmät väittää oman mielipiteen kumoavan suunnilleen kaikki luonnontieteet; milloin menee fysiikka, milloin geologia uusiksi joidenkin horinoissa. Pienellä pintaraaputtelulla vaikkapa sivulliselle lukijalle näytetään näiden mielipiteiden todellinen luonne.

        Juuri tämä.

        Sitten se pohja-aines ottaa tosissaan näitä mielipiteitä ja jatkaa sen levittämistä. Yleensä he vielä lisäävät sanoja kuten "faktaa", "tutkijat ovat tätä mieltä" tai "totuus".

        Sain nyt kuvan, että hän tietoisesti tekee tälläistä yhteistä tarinaprojektia. Hän ei tajua joidenkin luulevan sen olevan totta.

        Vähän kuten tähtien sodasta löytyy paljon fanien tekemiä tarinoita. Mutta siitä sentään kukaan ei luule sen olevan jotain oikeaa historiaa. Tosin tähtien sodan faniprojektissa sentään kunnioitetaan toisten tekeleitä sen verran että pyritään välttämään päällekkäisyyksiä. (Kyllä niitäkin toki tapahtuu)

        -C


      • Anonyymi
        Anonyymi kirjoitti:

        Ei tietenkään ole tarkoitus rajoittaa mielipiteitä. Mutta kun tässä on kysymys siitä, että tulvatarun väitetään tapahtuneen todellisuudessa, niin silloin täytyisi olla rinnakkaistidellisuuksia. Tai sitten kertojan pitäisi ilmaista selvästi, että kyseessä on oma mielipide tai fantasia.

        Nythän täällä härskeimmät väittää oman mielipiteen kumoavan suunnilleen kaikki luonnontieteet; milloin menee fysiikka, milloin geologia uusiksi joidenkin horinoissa. Pienellä pintaraaputtelulla vaikkapa sivulliselle lukijalle näytetään näiden mielipiteiden todellinen luonne.

        Vesi voi olla ns. pimeää, eli elotonta ainetta. Siihen tulee elämä eli aaltoilu vasta, kun se elävöityy. Vesi elävöittää kaiken kuolleen. Myös meidän tunne-elämä on kuin aaltoilua.

        Itse olen nähnyt, ilmeisesti tuollaisen rinnakkaistodellisuuden parikin kertaa. Siinä järven vesi siinsi täysin sinisenä, mutta se ei ollut todellinen vaan ikään kuin kuollut piirros. Eli elämä tulee meihin elämästä. Muuten olisimme litteitä, kuolleita piirroshahmoja.


      • Anonyymi

        Missä mielessä pimeä (eli tuntematon...) aine ja energia ovat tuhoamistapoja? Perustele.
        Pimeä energia iski jälleen ja aiheutti vedenpaisumuksen?
        Jos sinänsä ei edes "usko" suhteellisuusteorian eikä kosmologiaan muutenkaan, mutta sieltä voi kyllä tarvitessa lainata termin....


    • Anonyymi

      Otsanahka ryppyilee ajatellessa.

      • Anonyymi

        Pallea kramppaa naureskellessa.


    • Anonyymi

      Kuten aiemmin mainittu, Raamattu ei puhu laivasta vaan arkista. Arkki on neliskanttinen laatikko, arkku, kuten kymmenen käskyä oli arkussa, siis liitonarkissa.

      Samoin on selvitetty, että tarkun saattoi rakentaa suorista puista, vaikka tekemällä pohjan parin metrin paksuiseksi, ja pikeämällä sen, jolloin arkku olisi varmaan kellunut vaikka olisi tullut vettä sisään.

      Onhan vieläkin ulivuoren purkauksia, ja kuumia vesisuihkuja, .Jos nyt olisi sattunut vaikka asteroidi iskemään maan kuoreen niin, että on tullut halkeamia ta jos vettä on valunut halkeamaan, se tietysti on suihkunnut ylös niinkuin kuumat lähteet tekevät yhä.

      Hyvin sanottu aiemmin: "Että ihmiset uskovat Pangeaan, on paljon typerämpää: Mannerlaatat yhdellä puolella maapalloa, ja toisella puolen ei mitään mannerlaattoja, vaan meret suoraan kuuman magman päällä! Täysin järjetöntä: Eivätkö he tajua, että jos mannerlaatta on vaikka sata kilometriä paksu, ja meri vaikka viisi kilometriä , niin se meri on laatan yläpuolella ja siirtyisi laatan mukana!
      Ai niin, unohdin, että se vesi tietysti valui laatalta pois. Mutta mihin? Maapallon koko kuori on laattojen peitossa, ei edes millin rakoa! Todella hauskaa tämä tiedemiesten järkeily!"

      • Anonyymi

        Taidat olla tosi uskovainen (tm) kun taas olet vääntelemässä sanojen tarkoitusta muuksi. :D


      • Anonyymi

        Raamatun arkki oli kyynärissä mitattuna pituus 300, leveys 50 ja korkeus 30. Kyynärä on yleensä vähän yli tai alle 0,5m eli arkki olisi ollut noin 150 x 25 x 15 metriä. Tämä siis olisi muka rakennettu yli 4000 vuotta vanhalla teknologialla, kun suurin nykytekniikalla rakennettu puulaiva oli kuunari Wyoming: 140m x 15m x 10m. Wyomingin rungosta ei koskaan saatu niin jäykkää, että se ei olisi vuotanut vaikka sitä jäykistettiin 90:llä järeällä rautatangolla.

        Kun rungosta tehdään leveämpi kuin maailman suurimman puulaivan rungosta tehtiin, eikä käytetä metallia tukemassa, niin arkin jäykkyys ei millään riitä. Rungon massan lisääminen lisää myös runkoon kohdistuvaa rasitusta, kun aallot nostavat alusta. Puun taivutuslujuuden yksikkö on Newtonia / mm^2 eli taivutuslujuus on suhteessa puun POIKKIPINTA-ALAAN, kun taas laivan runkoon kohdistuva rasitus on verrannollinen laivan MASSAAN ja oma massa on puulaivaan käytetyn puun tilavuus. Kun rungon paksuus kasvaa kaksinkertaiseksi, niin lujuus 4-kertaistuu, mutta materiaalin massa ja rasitus jonka rungon pitäisi kestää 8-kertaistuu.

        Jos aluksessa ei ole hydrodynaamista keulaa, niin aaltojen rasitus kasvaa entisestään. Puu taipuu ja saumat eivät pidä ja laiva vuotaa. Wuoming pysyi pinnalla vain kun sitä koko ajan pumpattiin tyhjäksi sisään tihkuvasta vedestä. Jumala ja Nooa unohtivat arkista pumput ja siksi se on mahdottomuus.


      • Anonyymi
        Anonyymi kirjoitti:

        Raamatun arkki oli kyynärissä mitattuna pituus 300, leveys 50 ja korkeus 30. Kyynärä on yleensä vähän yli tai alle 0,5m eli arkki olisi ollut noin 150 x 25 x 15 metriä. Tämä siis olisi muka rakennettu yli 4000 vuotta vanhalla teknologialla, kun suurin nykytekniikalla rakennettu puulaiva oli kuunari Wyoming: 140m x 15m x 10m. Wyomingin rungosta ei koskaan saatu niin jäykkää, että se ei olisi vuotanut vaikka sitä jäykistettiin 90:llä järeällä rautatangolla.

        Kun rungosta tehdään leveämpi kuin maailman suurimman puulaivan rungosta tehtiin, eikä käytetä metallia tukemassa, niin arkin jäykkyys ei millään riitä. Rungon massan lisääminen lisää myös runkoon kohdistuvaa rasitusta, kun aallot nostavat alusta. Puun taivutuslujuuden yksikkö on Newtonia / mm^2 eli taivutuslujuus on suhteessa puun POIKKIPINTA-ALAAN, kun taas laivan runkoon kohdistuva rasitus on verrannollinen laivan MASSAAN ja oma massa on puulaivaan käytetyn puun tilavuus. Kun rungon paksuus kasvaa kaksinkertaiseksi, niin lujuus 4-kertaistuu, mutta materiaalin massa ja rasitus jonka rungon pitäisi kestää 8-kertaistuu.

        Jos aluksessa ei ole hydrodynaamista keulaa, niin aaltojen rasitus kasvaa entisestään. Puu taipuu ja saumat eivät pidä ja laiva vuotaa. Wuoming pysyi pinnalla vain kun sitä koko ajan pumpattiin tyhjäksi sisään tihkuvasta vedestä. Jumala ja Nooa unohtivat arkista pumput ja siksi se on mahdottomuus.

        Heyerthalin RA-lautta oli tehty kaisloista. Eiköhänsentään jykevät hongat ja muut puut ole vahvempia.

        Mitä ihmettä aaltojen olisi pitänyt hakata keulaa hajalle lautassa joka ajelehtii veden mukana? Ei kai arkissa ollut moottoria joka olisi puskenut.

        On turha yrittää todistella, että laivasta tulisi heikompi, kun siitä tekee paksumpipohjaisen, ja vaikka seinätkin paksummista hirsistä. Koska niiden ei tarvinnut taipua laivan muotoon, ne saattoivat olla vaikka metrin paksuisia.

        Titanic ei tainnut kestää juuri mitään. Jos se olisi ollut hirsistä tehty, useammat olisi selvinneet!


      • Anonyymi
        Anonyymi kirjoitti:

        Heyerthalin RA-lautta oli tehty kaisloista. Eiköhänsentään jykevät hongat ja muut puut ole vahvempia.

        Mitä ihmettä aaltojen olisi pitänyt hakata keulaa hajalle lautassa joka ajelehtii veden mukana? Ei kai arkissa ollut moottoria joka olisi puskenut.

        On turha yrittää todistella, että laivasta tulisi heikompi, kun siitä tekee paksumpipohjaisen, ja vaikka seinätkin paksummista hirsistä. Koska niiden ei tarvinnut taipua laivan muotoon, ne saattoivat olla vaikka metrin paksuisia.

        Titanic ei tainnut kestää juuri mitään. Jos se olisi ollut hirsistä tehty, useammat olisi selvinneet!

        Kova on yritys olla sivistymättä.

        Kaislalaivan ei tarvitse ole tiivis, koska sen runko on kevyt ja itsekantava. Ja se ei uppoa vuotojen takia vaan koska kaisla vettyy ja Heyerdahlin n. 60 päivän purjehdus oli ilmeisesti aika maksimi mihin se kykeni ennen kuin kasla vettyi liikaa.

        RA:n pituus oli 18m eli sen PITUUS oli vähemmän kuin Raamatussa kerrotun arkin LEVEYS. Kyllä honkapuusta tai tammesta pystyy vallan mainiosti tekemään merikelpoisen aluksen. Kolumbuksen matkalla olleen Pintan kansipituus oli 17m eli käytännössä sama kuin Heyerdahlin Ra:n.

        Aallot keikuttavat laivaa vaikka se vain ajelihtisi ja laivan runkoon kohdistuu rasitus joka on verrannollinen rungon painoon. Koska mainingit eivät ole 150m pitkiä niin aallon harja kohdistuu eri kohtaan alusta eri aikaan. Kun puulaivan rungon paksuus kaksinkertaistetaan, niin rungon lujuus 4-kertaistuu, mutta laivan massa 8-kertaistuu eli (tyhjän) laivan jäykkyys HEIKKENEE. Tämä on ihan perusfysiikkaa (ja matematiikkaa).


      • Anonyymi
        Anonyymi kirjoitti:

        Kova on yritys olla sivistymättä.

        Kaislalaivan ei tarvitse ole tiivis, koska sen runko on kevyt ja itsekantava. Ja se ei uppoa vuotojen takia vaan koska kaisla vettyy ja Heyerdahlin n. 60 päivän purjehdus oli ilmeisesti aika maksimi mihin se kykeni ennen kuin kasla vettyi liikaa.

        RA:n pituus oli 18m eli sen PITUUS oli vähemmän kuin Raamatussa kerrotun arkin LEVEYS. Kyllä honkapuusta tai tammesta pystyy vallan mainiosti tekemään merikelpoisen aluksen. Kolumbuksen matkalla olleen Pintan kansipituus oli 17m eli käytännössä sama kuin Heyerdahlin Ra:n.

        Aallot keikuttavat laivaa vaikka se vain ajelihtisi ja laivan runkoon kohdistuu rasitus joka on verrannollinen rungon painoon. Koska mainingit eivät ole 150m pitkiä niin aallon harja kohdistuu eri kohtaan alusta eri aikaan. Kun puulaivan rungon paksuus kaksinkertaistetaan, niin rungon lujuus 4-kertaistuu, mutta laivan massa 8-kertaistuu eli (tyhjän) laivan jäykkyys HEIKKENEE. Tämä on ihan perusfysiikkaa (ja matematiikkaa).

        Huonoa on matematiikkasi! Jos sinulla on 1 cm paksuinen lauta, ja siihen naulaat toisen yhtä paksun päälle, niin Väisäsen laskuopin mukaan silloin paino kaksinkertaistuu, mutta samalla tulee myös lauta puolta vahvemmaksi.

        Sinun teoriasi mukaan soutuvene pitäisi tehdä ohuemmista laudoista, ettei se hajoaisi aallokossa!

        Sitäpaitsi Nooa pikesi arkin jolloin vesi ei siihen imeytynyt.


      • Anonyymi
        Anonyymi kirjoitti:

        Huonoa on matematiikkasi! Jos sinulla on 1 cm paksuinen lauta, ja siihen naulaat toisen yhtä paksun päälle, niin Väisäsen laskuopin mukaan silloin paino kaksinkertaistuu, mutta samalla tulee myös lauta puolta vahvemmaksi.

        Sinun teoriasi mukaan soutuvene pitäisi tehdä ohuemmista laudoista, ettei se hajoaisi aallokossa!

        Sitäpaitsi Nooa pikesi arkin jolloin vesi ei siihen imeytynyt.

        Nooan arkki on oman ketjun arvoinen


      • Anonyymi
        Anonyymi kirjoitti:

        Huonoa on matematiikkasi! Jos sinulla on 1 cm paksuinen lauta, ja siihen naulaat toisen yhtä paksun päälle, niin Väisäsen laskuopin mukaan silloin paino kaksinkertaistuu, mutta samalla tulee myös lauta puolta vahvemmaksi.

        Sinun teoriasi mukaan soutuvene pitäisi tehdä ohuemmista laudoista, ettei se hajoaisi aallokossa!

        Sitäpaitsi Nooa pikesi arkin jolloin vesi ei siihen imeytynyt.

        Luuletko, että Wyoming ei olisi tervattu / maalattu "tiiviiksi". Silti se ei pysynyt tiiviinä.

        On sula mahdottomuus rakentaa merikelpoinen 150m pitkä puulaiva nykytekniikalla puhumattakaan 4000 vuotta vanhalla tietotaidolla.


      • Anonyymi
        Anonyymi kirjoitti:

        Luuletko, että Wyoming ei olisi tervattu / maalattu "tiiviiksi". Silti se ei pysynyt tiiviinä.

        On sula mahdottomuus rakentaa merikelpoinen 150m pitkä puulaiva nykytekniikalla puhumattakaan 4000 vuotta vanhalla tietotaidolla.

        Mitä tiedät ihmisten tekniikasta ennen vedenpaisumusta?

        Osasivathan he sahata graniittiakin kun rakensivat kolme suurinta pyramidia. Ja saumat olivat yhtä hyvät kuin ntimanttisahalla tehdyt nykyään.

        Sitäpaitsi sahasivat marmorilaattoja, joilla peittivät koko pyramidit. Näitä on enää jäljellä yhden pyramidin huipulla.

        Ei siihen kelpaa rautasaha. Kokeile, jos et usko!


      • Anonyymi
        Anonyymi kirjoitti:

        Mitä tiedät ihmisten tekniikasta ennen vedenpaisumusta?

        Osasivathan he sahata graniittiakin kun rakensivat kolme suurinta pyramidia. Ja saumat olivat yhtä hyvät kuin ntimanttisahalla tehdyt nykyään.

        Sitäpaitsi sahasivat marmorilaattoja, joilla peittivät koko pyramidit. Näitä on enää jäljellä yhden pyramidin huipulla.

        Ei siihen kelpaa rautasaha. Kokeile, jos et usko!

        Millä kreikkalaiset ja roomalaiset sahasivat?


      • Anonyymi
        Anonyymi kirjoitti:

        Mitä tiedät ihmisten tekniikasta ennen vedenpaisumusta?

        Osasivathan he sahata graniittiakin kun rakensivat kolme suurinta pyramidia. Ja saumat olivat yhtä hyvät kuin ntimanttisahalla tehdyt nykyään.

        Sitäpaitsi sahasivat marmorilaattoja, joilla peittivät koko pyramidit. Näitä on enää jäljellä yhden pyramidin huipulla.

        Ei siihen kelpaa rautasaha. Kokeile, jos et usko!

        Laivanrakennuksen haasteet ovat ihan muualla kuin lautojen sahaamisessa. Esimerkiksi viikingit tekivät laivalautansa kokonaan ilman sahoja kirveillä ja kiiloilla.

        Kuunari Wyomingin esimerkki osoittaa, että ei ole sattumaa ja pelkkää ihmisten tyhmyyttä, että KOSKAAN ei ole tehty 150 metrin pituisia puulaivoja. Suurimmat antiikin laivat olivat yli 2000 vuotta väitettyä Nooan arkkia myöhemmin tehdyt ns. Nemi-järven laivat (tai proomut) jotka nekin oli tehty järvelle eikä valtamerille. Nemi-laivojen pituus oli reilu 70 metriä ja leveämmän leveys 24m. Eli tilavuudeltaan alle puolet väitetystä arkista.

        Jos vertaa väitettyä arkkia niihin laivoihin, joita oikeasti tiedämme rakennetun varhaisimpien merenkävijäkulttuurien toimesta, niin tarinan mielikuvituksellisuus tulee selväksi. Ns. Puntin laivat (1500 eaa - 1000 eaa eli egympitläiset puulaivat olivat n. 25m pitkiä - eli yhtä pitkiä kuin arkin väitettiin olevan leveä. Samaa luokkaa oli ylivertaisina merenkävijöinä tunnettujen foinikialaisten suurimmat laivat. Kreikkalaisten suuret sotalaivat, trierinit eli kolmisoudut (n. 300 eaa) olivat jopa yli 40 metriä pitkiä, mutta leveydeltään vain 5,5 metrisiä.

        Kysyä sopii myös että mihin katosi laivanrakennusosaaminen, kun tarinan mukaan Nooan jälkeen KOKO IHMISKUNTA koostui heistä, jotka olivat olleet mukana rakentamassa Ruotsinlautan kokoista puulaivaa? Oli siis muka aika, jolloin KAIKKI miehet osasivat rakentaa 150m pitkän merikelpoisen puulaivan, kun sen jälkeen KUKAAN ei ole osannut?

        Tässä timanttisahalla millintarkasti sahattuja pyramidin kivenlohkareita:
        https://www.shutterstock.com/fi/image-photo/two-friends-pyramids-together-one-sitting-1094045420


      • Anonyymi

        "Onhan vieläkin ulivuoren purkauksia, ja kuumia vesisuihkuja, .Jos nyt olisi sattunut vaikka asteroidi iskemään maan kuoreen niin, että on tullut halkeamia ta jos vettä on valunut halkeamaan, se tietysti on suihkunnut ylös niinkuin kuumat lähteet tekevät yhä."

        Maankuoressa on aina halkeamia, vaikka kreationistit taitavat väittää ettei ole? En ole selvillä lahkouskomuksista


      • Anonyymi
        Anonyymi kirjoitti:

        Mitä tiedät ihmisten tekniikasta ennen vedenpaisumusta?

        Osasivathan he sahata graniittiakin kun rakensivat kolme suurinta pyramidia. Ja saumat olivat yhtä hyvät kuin ntimanttisahalla tehdyt nykyään.

        Sitäpaitsi sahasivat marmorilaattoja, joilla peittivät koko pyramidit. Näitä on enää jäljellä yhden pyramidin huipulla.

        Ei siihen kelpaa rautasaha. Kokeile, jos et usko!

        Pyramidiit olivat peitetty kalkkikivellä.

        Jos ihmiset olivat heti Aatamin ja Eevan aikana superälykkäitä, miten kesti n vähintään 800 vuotta ennenkuin kirjoitustaito (kuvakirjoitus) keksittiin?

        Ai niin joo "ei voi tietää, mutta ajattelemalla ihan itse voi olettaa että se keksittiinkin jo paratiisissa!! Me päästään taivaaseen kun me keksitään omasta päästämme näitä juttuja."


      • Anonyymi
        Anonyymi kirjoitti:

        Pyramidiit olivat peitetty kalkkikivellä.

        Jos ihmiset olivat heti Aatamin ja Eevan aikana superälykkäitä, miten kesti n vähintään 800 vuotta ennenkuin kirjoitustaito (kuvakirjoitus) keksittiin?

        Ai niin joo "ei voi tietää, mutta ajattelemalla ihan itse voi olettaa että se keksittiinkin jo paratiisissa!! Me päästään taivaaseen kun me keksitään omasta päästämme näitä juttuja."

        Miten voidaan sanoa, että kirjoitustaidon ajoitus on luotettava, jos kaikki muut samoilla menetelmillä tehdyt, mutta uskomuksiin sopimattomat ajoitukset ovat virheellisiä?


      • Anonyymi
        Anonyymi kirjoitti:

        Miten voidaan sanoa, että kirjoitustaidon ajoitus on luotettava, jos kaikki muut samoilla menetelmillä tehdyt, mutta uskomuksiin sopimattomat ajoitukset ovat virheellisiä?

        No nyt voisit kyllä vähän täsmentää; esim mitkä samat menetelmät?
        Ja mitkä uskomukset ja mitkä niihin sopimattomat ajoitukset
        ;)


    • Anonyymi

      Suosittelen lukemaan Hans-Joachim Zillmerin kirjan Die Erde im Umbruch - Katastrophen form(t)en diese Welt , Beweise aus Historischen Zeit (Herbig, 300 s., 2011).

      Tässä hieman happaman perinteisistä teorioista kiinnipitävän wikitieteilijän kritikkiä Zillmeristä, tämän lukemattomiin kiistattomiin todisteisiin, ja perustuvista arvioista:
      https://de.wikipedia.org/wiki/Hans-Joachim_Zillmer

      Asenteestaan huolimatta wikitieteilijä ei pysty kumoamaan Zillmerin (ei ainoana) esiintuomia kiistattomia tosiasioita (joista joitakin nykytieteen edustajat ovatkin vihdoin joutuneet tunnustamaan).

      Väärässä olemisen tunnustaminen on yleensä liian kova paikka mainettaan yli kaiken puolustaville eliittiprofessoreille (heidän elinaikanaan).

      • Anonyymi

        Zillmer taitaa olla saman lainen besserwisser kuin Heyerdahl jonka "hypoteesit" olivat pelkkää mielikuvitusta. Esimerkiksi Kon-Tiki lautta, jolla Heyerdahl yritti todistella, että Pääsiäissaaret eivät olleet polynesialaisten asuttamia (vaikka polynesialaiset olivat löytäneet KAIKKI tyynenmeren saaret Hawaijilta Uuteen Seelantiin), vaan Inkat.

        Tieteentekijöiden ja Heyerdahlin tapaisten besserwisserien ero on siinä, että Heyerdahlin tarkoitus EI OLLUT selvittää miten esimerkiksi Pääsiäissaaret asutettiin, vaan HÄN ITSE OLI OIKEASSA, vaikka ei ollut minkäänlaista ammattimaista tutkimusta ollut edes tehnyt.

        Vai onko joku vertaisarvioinut Zillmerin "tutkimukset"? Onko hän edes tehnyt mitään tutkimuksia vai tyypillisen kreationistin tavoin lainauslouhii niitä näitä, leikkaa sopimattomat kohdat pois ja yhdistelee sopivat osat mielivaltaisesti ja saa aikaan keitoksen, joka tukee sitä että se, minkä kirjoittaja jo ennestään "tiesi" olisi totta.


      • Anonyymi
        Anonyymi kirjoitti:

        Zillmer taitaa olla saman lainen besserwisser kuin Heyerdahl jonka "hypoteesit" olivat pelkkää mielikuvitusta. Esimerkiksi Kon-Tiki lautta, jolla Heyerdahl yritti todistella, että Pääsiäissaaret eivät olleet polynesialaisten asuttamia (vaikka polynesialaiset olivat löytäneet KAIKKI tyynenmeren saaret Hawaijilta Uuteen Seelantiin), vaan Inkat.

        Tieteentekijöiden ja Heyerdahlin tapaisten besserwisserien ero on siinä, että Heyerdahlin tarkoitus EI OLLUT selvittää miten esimerkiksi Pääsiäissaaret asutettiin, vaan HÄN ITSE OLI OIKEASSA, vaikka ei ollut minkäänlaista ammattimaista tutkimusta ollut edes tehnyt.

        Vai onko joku vertaisarvioinut Zillmerin "tutkimukset"? Onko hän edes tehnyt mitään tutkimuksia vai tyypillisen kreationistin tavoin lainauslouhii niitä näitä, leikkaa sopimattomat kohdat pois ja yhdistelee sopivat osat mielivaltaisesti ja saa aikaan keitoksen, joka tukee sitä että se, minkä kirjoittaja jo ennestään "tiesi" olisi totta.

        Etkö kestä lukea ja arvioida Zillmerin esittämiä kovia tosiasioita pallomme eri puolilta.
        Vai etkö jaksa edes lukea 300 sivuista asiajulkaisua?
        Hengenvoimasi taitavat riittää vain A4 -tiivistelmiin ja Vihreään lankaan.

        Et taida (pakkoruotsin seuraamuksena) ymmärtää saksankieltä? Osaathan perustason englantia, mutta täsmällisiä asiatekstejä et edes englanniksi.


      • Anonyymi
        Anonyymi kirjoitti:

        Etkö kestä lukea ja arvioida Zillmerin esittämiä kovia tosiasioita pallomme eri puolilta.
        Vai etkö jaksa edes lukea 300 sivuista asiajulkaisua?
        Hengenvoimasi taitavat riittää vain A4 -tiivistelmiin ja Vihreään lankaan.

        Et taida (pakkoruotsin seuraamuksena) ymmärtää saksankieltä? Osaathan perustason englantia, mutta täsmällisiä asiatekstejä et edes englanniksi.

        Zillerman on huuhaa-kirjoja kirjoittava rakennusinsinööri, joka ei ole tehnyt ensimmäistäkään geologista tutkimusta eikä kirjoittanut ensimmäistäkään vertaisarvioitua tieteellistä artikkelia. mutta luulee kumoavansa kaiken, mitä geologinen tutkimus on viimeisen 200 vuoden aikana saanut selville. Hän on myös niitä kreationisteja, jotka yrittävät todistaa Raamatun luomistarun keksimällä tarinan joka ristiriidassa sekä Raamatun kertomuksen kanssa että täysin ristiriidassa kaiken geologisen tiedon kanssa.

        Tuossa aiemmin verrattiin Zillermannia Heyerdahliin, mutta vaikka Heyerdahlikin on omiin ideoihinsa sokeasti ihastunut huuhaa-amatööri, jonka hypoteesit ovat roskaa, niin hän on hyvin maltillinen verrattuna Zillermanniin. Suomalainen vastine Zillermannille vastaa lähinnä Kauko Nieminen joka kehitti oman "eetteripyörreteoriansa".


      • Anonyymi
        Anonyymi kirjoitti:

        Zillerman on huuhaa-kirjoja kirjoittava rakennusinsinööri, joka ei ole tehnyt ensimmäistäkään geologista tutkimusta eikä kirjoittanut ensimmäistäkään vertaisarvioitua tieteellistä artikkelia. mutta luulee kumoavansa kaiken, mitä geologinen tutkimus on viimeisen 200 vuoden aikana saanut selville. Hän on myös niitä kreationisteja, jotka yrittävät todistaa Raamatun luomistarun keksimällä tarinan joka ristiriidassa sekä Raamatun kertomuksen kanssa että täysin ristiriidassa kaiken geologisen tiedon kanssa.

        Tuossa aiemmin verrattiin Zillermannia Heyerdahliin, mutta vaikka Heyerdahlikin on omiin ideoihinsa sokeasti ihastunut huuhaa-amatööri, jonka hypoteesit ovat roskaa, niin hän on hyvin maltillinen verrattuna Zillermanniin. Suomalainen vastine Zillermannille vastaa lähinnä Kauko Nieminen joka kehitti oman "eetteripyörreteoriansa".

        Olet tyypillinen fundamentalisti, kuten esimerkiksi islam-fundamentalistit.
        Et suostu lukemaan (et tosin pystykään lukemaan vieraskielistä asiatekstiä).

        Pelkistynyt ja lukkiutunut maailmankuvasi häiriintyisi lukemisesta - havaitsisit että ilmiöt ovat olleet monimutkaisempia ja niistä on lukemattomasti kovia, kiistattomia todisteita.


      • "Zillmeristä, tämän lukemattomiin kiistattomiin todisteisiin, ja perustuvista arvioista"

        Todisteita on siis peräti lukemattomia, mutta yhtäkään et vaivautunut antamaan esimerkiksi.

        "kumoamaan Zillmerin (ei ainoana) esiintuomia kiistattomia tosiasioita (joista joitakin nykytieteen edustajat ovatkin vihdoin joutuneet tunnustamaan)."

        Niin siis mitkä tosiasiat?

        Huonolla lukiosaksallanikin ymmärsin, että hänen väitteensä ja johtopäätöksensä eivät näytä pohjautuvan tosiasioihin.

        Mutta keskustellaan toki asiasta. Osaatko kertoa mikä mielestäsi on hänen paras väitteensä ja millä hän sitä perustelee?


      • Anonyymi
        Anonyymi kirjoitti:

        Olet tyypillinen fundamentalisti, kuten esimerkiksi islam-fundamentalistit.
        Et suostu lukemaan (et tosin pystykään lukemaan vieraskielistä asiatekstiä).

        Pelkistynyt ja lukkiutunut maailmankuvasi häiriintyisi lukemisesta - havaitsisit että ilmiöt ovat olleet monimutkaisempia ja niistä on lukemattomasti kovia, kiistattomia todisteita.

        Voi ku hianoa sanahelinää...


      • Anonyymi
        siare kirjoitti:

        "Zillmeristä, tämän lukemattomiin kiistattomiin todisteisiin, ja perustuvista arvioista"

        Todisteita on siis peräti lukemattomia, mutta yhtäkään et vaivautunut antamaan esimerkiksi.

        "kumoamaan Zillmerin (ei ainoana) esiintuomia kiistattomia tosiasioita (joista joitakin nykytieteen edustajat ovatkin vihdoin joutuneet tunnustamaan)."

        Niin siis mitkä tosiasiat?

        Huonolla lukiosaksallanikin ymmärsin, että hänen väitteensä ja johtopäätöksensä eivät näytä pohjautuvan tosiasioihin.

        Mutta keskustellaan toki asiasta. Osaatko kertoa mikä mielestäsi on hänen paras väitteensä ja millä hän sitä perustelee?

        Asiat ovat laajoja ja monimutkaisia, kuten niistä esitetyt kovat todisteet.
        Ei kenenkään aika riitä niitä tänne monistamaan ja lukemattomia valokuvia, karttoja ja muuta aineistoa tänne tuottamaan.

        Jos ei yksityiskohtaisia asiajulkaisuja halua tai pysty olemattoman kielitaitonsa ja yleissivistyksensä vuoksi lukemaan, silloin säilyy onnellisen tietämättömänä elämänsä loppuun asti. Ylimielisenä tiedosta kieltäytyjänä.


      • Anonyymi kirjoitti:

        Asiat ovat laajoja ja monimutkaisia, kuten niistä esitetyt kovat todisteet.
        Ei kenenkään aika riitä niitä tänne monistamaan ja lukemattomia valokuvia, karttoja ja muuta aineistoa tänne tuottamaan.

        Jos ei yksityiskohtaisia asiajulkaisuja halua tai pysty olemattoman kielitaitonsa ja yleissivistyksensä vuoksi lukemaan, silloin säilyy onnellisen tietämättömänä elämänsä loppuun asti. Ylimielisenä tiedosta kieltäytyjänä.

        "Asiat ovat laajoja ja monimutkaisia, kuten niistä esitetyt kovat todisteet."

        No niin, rohkenisitko nyt vihdoin paljastaa jonkin niistä todisteista?

        "Ei kenenkään aika riitä niitä tänne monistamaan ja lukemattomia valokuvia, karttoja ja muuta aineistoa tänne tuottamaan."

        Ei niin, enkä kukaan ole keneltäkään sellaista pyytänyt.

        "Jos ei yksityiskohtaisia asiajulkaisuja halua tai pysty olemattoman kielitaitonsa ja yleissivistyksensä vuoksi lukemaan, silloin säilyy onnellisen tietämättömänä elämänsä loppuun asti. Ylimielisenä tiedosta kieltäytyjänä."

        Juu juu. Mutta onneksi minä en ole tällainen, vaan toki suostun lukemaan ainakin saksankielisiä tekstejä. Mikä teksti minun pitäisi lukea?

        Kummallista että itse otat aiheeksi Zillmerin väitteet, mutta sitten et niin millään halua keskustella niistä.


      • Anonyymi
        Anonyymi kirjoitti:

        Olet tyypillinen fundamentalisti, kuten esimerkiksi islam-fundamentalistit.
        Et suostu lukemaan (et tosin pystykään lukemaan vieraskielistä asiatekstiä).

        Pelkistynyt ja lukkiutunut maailmankuvasi häiriintyisi lukemisesta - havaitsisit että ilmiöt ovat olleet monimutkaisempia ja niistä on lukemattomasti kovia, kiistattomia todisteita.

        Miksi geologiaa tutkitaan yliopistoissa ja siitä tehdään väitöskirjoja, jos joku Jeesuksella päänsä sekoittanut rakennusmestari pystyy ymmärtämään geologian paremmin tekemättä itse ENSIMMÄISTÄKÄÄN geologista tutkimusta?

        Monien muiden taitojen lisäksi sinulta puuttuu tähdellisesti kyky lähdekritiikkiin.


      • Anonyymi
        Anonyymi kirjoitti:

        Miksi geologiaa tutkitaan yliopistoissa ja siitä tehdään väitöskirjoja, jos joku Jeesuksella päänsä sekoittanut rakennusmestari pystyy ymmärtämään geologian paremmin tekemättä itse ENSIMMÄISTÄKÄÄN geologista tutkimusta?

        Monien muiden taitojen lisäksi sinulta puuttuu tähdellisesti kyky lähdekritiikkiin.

        Ei tuo halua, että kirjan väitteitä aletaan ruotimaan yksityiskohtaisesti. Jos palstalla on oikeaa geologiaa opiskellut tai ammattigeologi, niin tuolta hihhulilta loppuu paukut heti kättelyssä. Siksi se häivyttää tuollaiseen epämääräiseen vihjailuun kompetenssin puutteen.

        PS. Hassua tuo englannin kielellä elvistely. Jos luonnontieteitä työkseen tekee tai edes harrastaa, niin lähtökohtaisesti englanti on kieli. Osaan terminologiasta on varsin huonot käönnökset. Fysiikassa ja ennen kaikkea kemian lähteissä olen törmännyt jonkin verran saksaan. Itseltä saksankieliset jää väliin.


      • Anonyymi
        Anonyymi kirjoitti:

        Etkö kestä lukea ja arvioida Zillmerin esittämiä kovia tosiasioita pallomme eri puolilta.
        Vai etkö jaksa edes lukea 300 sivuista asiajulkaisua?
        Hengenvoimasi taitavat riittää vain A4 -tiivistelmiin ja Vihreään lankaan.

        Et taida (pakkoruotsin seuraamuksena) ymmärtää saksankieltä? Osaathan perustason englantia, mutta täsmällisiä asiatekstejä et edes englanniksi.

        Korro esimerkki Zillmerin esittämästä kovasta tosiasiasta


      • Anonyymi
        Anonyymi kirjoitti:

        Korro esimerkki Zillmerin esittämästä kovasta tosiasiasta

        Muistan kun joskus teininä ostin divarista Heyerdahlin Kon-Tiki -kirjan. Se oli todella vakuuttava ja mukaansatempaava. Sen luettuani olin hyvin vakuuttunut siitä, että inkat olivat asuttaneet Pääsiäissaaren. Omasta ideastaan vakuuttunut amatööri Heyerdahl osasi olla hyvin vakuuttava.

        Sitten kun on tutustunut oikeiden tutkijoiden kirjoittamiin artikkeleihin aiheesta, niin on selvää, että Heyerdahl oli täysin väärässä ja poiminut kirjaansa äärimmäisen valikoivasti vain omaa kuvitelmaansa tukevia "todisteita" ja tehnyt tulkintoja joita voi pitää jopa valehteluna. Esimerkiksi Pääsiäissaaren kuuluisat Moai-patsaat ovat tyylillisesti ja tekotavaltaan suora jatkumo muualla polynesiassa tavattavalle kiviveistos -perinteelle. Pääsiäissaarella vain patsaiden koko on suurempi kuin muualla, vaikka Heyerdahl vertasi sitä pelkästään amerikkalaisiin kiviveistoksiin (esim. olmeekkien kivipäihin) ja "unohti" edes mainita, että kiviveistoksia löytyy valtavasti myös polynesiasta.

        En yhtään epäile, etteikö amatööri Zillmer olisi samanlainen taitava kirjailija, joka saa kritiikittömän lukijen kiedottua omaan tarinaansa niin vahvasti, että ei epäile sen todenperäisyyttä. Siksi aina kannattaa säilyttää lähdekritiikki ja tarkistaa mitä oikeat tutkijat ovat asioista havainneet ja mitkä ovat heidän KOKO HAVAINTOAINEISTOON perustuvat johtopäätöksensä.


    • Anonyymi

      Miten evoluutionisti selittää nykyisen vesimäärän, joka riittää peittämään koko maapallon kolme kilometriä?

      • Anonyymi

        En ole evolutionisti enkä muukaan nisti.
        Ei Zillmerin kirja evoluutiota käsittele, vaan tapahtuneita geologisia ja muita tapahtumia, joista lukemattomat kovat todisteet kertovat.
        Mitä vesien uudelleen sijoittumiseen tulee, niitä kirja käsittelee kyllä niitä - koviin kiistattomiin todisteisiin perustuen.

        Koita luopua lapsenuskostasi. Lue (jos osaat tai ole vaiti).
        Totea itse että tapahtumat ovat olleet monimutkaisia ja -moniaiheisempia kuin mitä 50...150 vuotta sitten, melko olemattomin tiedoin teoretisoitiin.


      • Anonyymi

        Ei sitä tarvitse selittää jos on evoluutionisti.
        Jos kreationisti sanoo, että tuo vesimäärä aikoinaan aiheutti vedenpaisumuksen, silloin pitää heti väittää, ettei tuo vesi riitäkään. Pitää vedota siihen, ettei maan pinta ole voinut olla tasaisempi silloin.

        Kuitenkin, jos katselee esimerkiksi TEPUIS -vuoria, tai butte- ja mesa- vuoria, voi nähdä ettämaakerrokset ovat olleet tasaisesti kunnes vesi on vienyt pois maan ja jättänyt vain hiukan jäljelle, kuin kakkupaloja lautaselle. Kerrokset jatkuvat vaakatasossa vuoresta toiseen.

        Joten maan pinta on ollut melko tasaista aikoinaan.


      • Anonyymi

        Nykyinen vesimäärä on nykyisellä maapallolla eikä mitään vedenpaisumusta ole.

        Raamatun kertomus väittää, että vettä tuli sateena ja "syvyyden lähteistä". Jokainen joka ymmärtää jotain ilmakehästä (Raamatun kirjoittajat eivät selväsitikkään tieteäneet) tietää, että sateilla ei voi olla kuin paikallista vaikutusta, koska vettä on ilmakehässä keskimäärin todella vähän ja toisaalta se kaikki on peräisin meristä ja muualta haihtuneesta vedestä. Joten kilometritolkulla vettä täytyi tulle "syvyyden lähteistä". Missä näitä lähteitä on. Vesien volyymi oli valtava joten melkoisia röörejä pitäisi jossain olla olemassa. Vaan kun ei ole.


      • Anonyymi
        Anonyymi kirjoitti:

        Ei sitä tarvitse selittää jos on evoluutionisti.
        Jos kreationisti sanoo, että tuo vesimäärä aikoinaan aiheutti vedenpaisumuksen, silloin pitää heti väittää, ettei tuo vesi riitäkään. Pitää vedota siihen, ettei maan pinta ole voinut olla tasaisempi silloin.

        Kuitenkin, jos katselee esimerkiksi TEPUIS -vuoria, tai butte- ja mesa- vuoria, voi nähdä ettämaakerrokset ovat olleet tasaisesti kunnes vesi on vienyt pois maan ja jättänyt vain hiukan jäljelle, kuin kakkupaloja lautaselle. Kerrokset jatkuvat vaakatasossa vuoresta toiseen.

        Joten maan pinta on ollut melko tasaista aikoinaan.

        Ja tästä hm, ällistyttävän tarkasta todistusaineesta päättelet että Raamatun mukainen koko maapallon kattava vedenpaisumus v 2 500 eaa on varmasti tosi...?!
        Tässä näkyy ero siinä miten tehdään tiedettä ja, hm, "tiedettä". Jälkimmäinen 'minulla on ihan sellainen tunne (vaikka vuoret ovat miljoonien vuosien ikäisiä)' pohjalta. Vaisto ja intuitio ovat oikeasti hyviä ominaisuuksia joita tarvitaan elämässä, mutta se "ei ihan riitä" kun on kyse tieteellisestä tutkimuksesta, jostain on saatava vahvistus pelkille hypoteeseille. Mutta kun kreasta tuntuu että niin se varmaan menee niin se on heti totta hänelle.


    • Anonyymi

      Me tiedämme täysin varmasti tieteen ansiosta että ei ole ikinä ollut maailmanlaajuista vedenpaisumusta, jostain syystä kertut eivät ymmärrä tätä faktaa.

      Eivät nähtävästi ymmärrä todisteita ollenkaan tai uskalla ottaa asioista selvää.

      • Anonyymi

        Nyt kerroit itsestäsi, koska juuri itse et halua (uskalla?) ottaa selvää.
        Nimenomaan kiistattomia, kovia todisteita esiintyy paljon.
        Ei niitä ole voitu eikä voida kumota olemattomiksi.

        Monelle kaltaisellesi maallikolle on mieluista tarrautua ja lukkiutua vanhoihin teorioihin, vaikka kovat tosiasiat ovat niiden vastaisia.


      • Anonyymi

        Tiedämme aivan varmasti, että keuhkokala on yhä keuhkokala, eikä esimerkiksi kirahvi, niinkuin evoluutio opettaa.
        Jostan syystä evoluutionistit eivät ymmärrä tätä faktaa.
        Eivät nähtävästi ymmärrä todisteita ollenkaan tai uskalla ottaa asioista selvää.


      • Anonyymi
        Anonyymi kirjoitti:

        Nyt kerroit itsestäsi, koska juuri itse et halua (uskalla?) ottaa selvää.
        Nimenomaan kiistattomia, kovia todisteita esiintyy paljon.
        Ei niitä ole voitu eikä voida kumota olemattomiksi.

        Monelle kaltaisellesi maallikolle on mieluista tarrautua ja lukkiutua vanhoihin teorioihin, vaikka kovat tosiasiat ovat niiden vastaisia.

        No kerro mistä löytyy edes yksi ammattimaisesti tehty ja vertaisarvioitu geologinen tutkimus, joka viittaa, edes siihen mahdollisuuteen, että maapallon olisi peittänyt globaali vedenpaisumus vain tuhansia vuosia sitten.


      • Anonyymi
        Anonyymi kirjoitti:

        Tiedämme aivan varmasti, että keuhkokala on yhä keuhkokala, eikä esimerkiksi kirahvi, niinkuin evoluutio opettaa.
        Jostan syystä evoluutionistit eivät ymmärrä tätä faktaa.
        Eivät nähtävästi ymmärrä todisteita ollenkaan tai uskalla ottaa asioista selvää.

        Miksi kirahvin sisäelimien, silmän, aivohermojen jne rakenteet ovat kopio keuhkokalan vastaavista? Väärä vastaus on, että "sellaisiahan eläimet ovat", koska eläimistön valtaosa niin biomassaltaan kuin lajimäärältään eroaa keuhkokaloista ja niistä polveutuneista tetrapodeista.


      • Anonyymi
        Anonyymi kirjoitti:

        No kerro mistä löytyy edes yksi ammattimaisesti tehty ja vertaisarvioitu geologinen tutkimus, joka viittaa, edes siihen mahdollisuuteen, että maapallon olisi peittänyt globaali vedenpaisumus vain tuhansia vuosia sitten.

        Sellaista tutkimusta ei voi tehdä, jos haluaa saada hyväksynnän. Se on sama kun heiluttaisiu punaista vaatetta härän edessä.
        Pitää odottaa kunnes pääsee eläkkeelle.


      • Anonyymi
        Anonyymi kirjoitti:

        Sellaista tutkimusta ei voi tehdä, jos haluaa saada hyväksynnän. Se on sama kun heiluttaisiu punaista vaatetta härän edessä.
        Pitää odottaa kunnes pääsee eläkkeelle.

        Kaikkien hörhöjen vakioselitys: tämä on salaliitto!

        Jos ymmärtäisit yhtään mitään tieteestä tietäisit että tietessä ne jotka löytävät uutta ja yllättävää ovat sankareita eikä ne jotka vain löytävät lisäperusteita vallitseville teorioille. Tietenkin uudet löydöt ja johtopäätökset pitää olla vankasti perusteltuja eikä pelkkää löysää puhetta. Tämä on varmasti kreationistille outo ajatus, kun kreationisti perustuu PELKÄSTÄÄN löysään puheeseen.


      • Anonyymi
        Anonyymi kirjoitti:

        Kaikkien hörhöjen vakioselitys: tämä on salaliitto!

        Jos ymmärtäisit yhtään mitään tieteestä tietäisit että tietessä ne jotka löytävät uutta ja yllättävää ovat sankareita eikä ne jotka vain löytävät lisäperusteita vallitseville teorioille. Tietenkin uudet löydöt ja johtopäätökset pitää olla vankasti perusteltuja eikä pelkkää löysää puhetta. Tämä on varmasti kreationistille outo ajatus, kun kreationisti perustuu PELKÄSTÄÄN löysään puheeseen.

        Juuri näin. Eikä nuo laitauskovaiset ymmärrä, että tieteilijän suuri sulka hatussa olisi muuttaa vallitsevaa käsitystä (jopa tieteellisen teoriaa). On hyvin luonnollista, ettei fundisuskova ymmärrä tätä, koska uskonnollisessa yhteisössä oman uskonnollisen agendan haastaminen on ihan saatanallista puuhas.


      • Anonyymi
        Anonyymi kirjoitti:

        Juuri näin. Eikä nuo laitauskovaiset ymmärrä, että tieteilijän suuri sulka hatussa olisi muuttaa vallitsevaa käsitystä (jopa tieteellisen teoriaa). On hyvin luonnollista, ettei fundisuskova ymmärrä tätä, koska uskonnollisessa yhteisössä oman uskonnollisen agendan haastaminen on ihan saatanallista puuhas.

        Muuttakaa sitten vallitsevaa käsitystänne! Miksi yhä luulette keuhkokalan nykyään muuttuneenn vaikka kolibriksi?

        Eikö evoluutionistien pitäisi muuttaa käsityksiään vaikka fossiilien perusteella?

        Miljoonia fossiileja on löydetty, eikä yksikään tue evoluutioteoriaa.


      • Anonyymi
        Anonyymi kirjoitti:

        Muuttakaa sitten vallitsevaa käsitystänne! Miksi yhä luulette keuhkokalan nykyään muuttuneenn vaikka kolibriksi?

        Eikö evoluutionistien pitäisi muuttaa käsityksiään vaikka fossiilien perusteella?

        Miljoonia fossiileja on löydetty, eikä yksikään tue evoluutioteoriaa.

        Koska keuhkokalan ja kolibrin elimistön perusrakenne ja sitä ohjaava geenistö ovat vain vähän muunnellut kopiot toisistaan. Miksi kolibrin silmä on käytännössä kopio keuhkokalan silmästä eikä muistuta mettä syövien mesipistiäisten silmää? Miksi keuhkokalan silmän rakenne ei ole sama kuin vesissä elävillä nilviäisillä tai niveljalkaisilla?


      • Anonyymi
        Anonyymi kirjoitti:

        Koska keuhkokalan ja kolibrin elimistön perusrakenne ja sitä ohjaava geenistö ovat vain vähän muunnellut kopiot toisistaan. Miksi kolibrin silmä on käytännössä kopio keuhkokalan silmästä eikä muistuta mettä syövien mesipistiäisten silmää? Miksi keuhkokalan silmän rakenne ei ole sama kuin vesissä elävillä nilviäisillä tai niveljalkaisilla?

        Siksi että keuhkokalan silmä on tehty sopivaksi keuhkokalalle. Ja se on yhä keuhkokala, eikä kolibri!


      • Anonyymi
        Anonyymi kirjoitti:

        Koska keuhkokalan ja kolibrin elimistön perusrakenne ja sitä ohjaava geenistö ovat vain vähän muunnellut kopiot toisistaan. Miksi kolibrin silmä on käytännössä kopio keuhkokalan silmästä eikä muistuta mettä syövien mesipistiäisten silmää? Miksi keuhkokalan silmän rakenne ei ole sama kuin vesissä elävillä nilviäisillä tai niveljalkaisilla?

        Jokainen tieteen löytö
        on vienyt tieteen maailman kuvaa viimeisten satojen vuosien aikana yhä kauemmas laitauskovaisten raamatullisesta maailmankuvasta. Mm.
        - litteä maa
        - maakeskeinen universumi
        - elokehän historia
        - universumin synty
        jne.
        Seuraavat askeleet esimerkiksi bioinformatiikassa ja genetiikassa aiheuttavat sen, että kreationistien pitää keksiä taas uusi valheita peittääkseen tieteen kehityksen.

        Laitauskovaisena sinun pitää skarpata, ettet hyväksy tieteen menetelmän tuloksia ja sovelluksia.

        Minulle on muuten ihan sama mihin uskot kunhan hyvän miraalin mukaan et valehtele siitä, mitä tiede meille todellisuydesta kertoo.


      • Anonyymi
        Anonyymi kirjoitti:

        Muuttakaa sitten vallitsevaa käsitystänne! Miksi yhä luulette keuhkokalan nykyään muuttuneenn vaikka kolibriksi?

        Eikö evoluutionistien pitäisi muuttaa käsityksiään vaikka fossiilien perusteella?

        Miljoonia fossiileja on löydetty, eikä yksikään tue evoluutioteoriaa.

        "Miljoonia fossiileja on löydetty, eikä yksikään tue evoluutioteoriaa."

        Kristallinkirkas vale.
        Tiedän kyllä, että lähimmäisiään pitää kohdella rakkaudella, mutta....mutta joo, ehkä Raamatun tutkimuksenne on jo sillä tasolla että sieltä on luettavissa että valehdellaan vaikka, jos on tarpeen.
        Vähän kuin Ignatius Loyolan 'päämäärä pyhittää keinot'


    • Anonyymi

      Avauksen kysymykseen on lyhyt vastaus: valehtelemalla.
      Ainoa millä kreationistit pystyvät perustelemaan uskomuksiaan on valehtelu. Toinen vaihtoehto on jättää kokonaan vastaamatta "vaikeisiin" kysymyksiin, jonka voi tehdä joko vaikenemalla tai selittämällä diibadaabaa jostain ihan muusta asiasta ja toivoa että kysyjä eksyy sivupolulle, eikä huomaa, että puheenaihetta vaihdettiin.

      Tässäkin on päästy vedenpaisumuksen fyysisistä mahdottomuuksista kolibreihin ja Heyerdahlin kaislalaivaan.

      • Anonyymi

        Ja vastauksia ei saa millään evoluutionisteilta siitä miten keuhkokalasta on tullut kolibri!

        Ei silmän samankaltaisuus tee kalasta kolibria! Päinvastoin se osoittaa yhteistä tekijää kummallekin.


      • Anonyymi
        Anonyymi kirjoitti:

        Ja vastauksia ei saa millään evoluutionisteilta siitä miten keuhkokalasta on tullut kolibri!

        Ei silmän samankaltaisuus tee kalasta kolibria! Päinvastoin se osoittaa yhteistä tekijää kummallekin.

        Kerrotko vielä miten kykenemättömyytesi ymmärtää kolibrin evoluutiohistoriaa liittyy avauksen kysymykseen siitä mistä ihmeestä uskomustenne mukainen kilometrien korkuinen vesimassa ilmestyi aiheuttamaan vedenpaisumuksen?


    • Anonyymi

      Tämä ketju kääntyi taas kreationismin perusjankkaukseen evoluutiosta. Nooan paatti sivuaa toki tätä, mutta sen voisi perata omassa ketjussa.

      Tehdään siis pieni synteesi siitä osasta keskustelua, mikä koski avausta.

      Kukaan ei kiistänyt avauksen kohtia. Niin moninaisesti tulvan aikaisesta todellisuudesta ja Raamatun sanasta on "mielipiteitä".

      Uusi selitys ehdottaa tulvaveden olevan pimeää ainetta, jolloin se ilmestyminen maapallolle sekä häviäminen lienee jotain, mitä meidän tuntema fysiikka ei osaa selittää. Tämä voisi jopa viitata Jumalan taikatemppuun.

      Toinen osin uusi selitys korostaa, että nykyiset merien vedet riitti, koska maanpinnalla tapahtui noihin aikoihin tuollaisia kilometrien maankohoamisia. Tämä tosin palautuu tuohon pikaisiin vuoret kohoamisiin (ja järkyttäviin geologisiin tapahtumiin).

      Kaikkiaan johtopäätöksenä voimme todeta, että jo pieni katselmus nostaa lukuisia fantasioita siitä, mistä vedet tuli, mihin ne meni ja miten kuonto toipui maagisesti tulvasta. Mielenkiintoista.

      Ketju varmaan kannattaa lopettaa tähän, koska se on jo hajonnut ja pointit hukkuu.

      • Anonyymi

        Hyvä yhteenveto. Kreationistien selitykset ovat tuotettu metodilla "mitä sylki suuhun tuo, kunhan se ei täsmää tieteellisen tutkimuksen johtopäätöksiin".


    • Anonyymi

      Avausta lukiessa näkee evokkeja vaivaavan tiedonpuutteen ihan selvänä. Vesi on nyt valtamerissä ja tuli maan alta, ei kuumana vaan ihan siedettävän lämpöisenä. Maan alla ollut vesi ei ole mikään ihme laisinkaan, uskotaanhan nykyään jo aika vahvasti että jopa Pluton pinnan alla lainehtii valtameri. Miksei siis olisi voinut olla joskus maassakin ?

      • Anonyymi

        Mene perämerelle. Sielläkin JÄÄN ALLA lainehtii valtameri. Maapallon pinta on kiveä eikä jäätä.


      • Anonyymi
        Anonyymi kirjoitti:

        Mene perämerelle. Sielläkin JÄÄN ALLA lainehtii valtameri. Maapallon pinta on kiveä eikä jäätä.

        Ok, ei Perämeri ole valtameri, mutta meri kuitenkin.


      • Anonyymi

        Lisätään listaan nyanssina "siedettävän lämpöisenä".

        Näinhän kreationismi toimii. Jos todellisuudessa kuuman lähteen vesi tulee kiehuvana, niin kreationistisessa geologiassa sitä kutsutaan siedettävän lämpöiseksi.


      • Anonyymi

        Tiedon puutetta avaus tosiaan yrittää korjata. Nimittäin kreationistien "varmat totuudet" tulvatarun tapahtumista ovat niin moninaisia, että niistä yritetään saada jokin kokonaiskuva. Noita tiedon murusia tähän ketjuun onkin muutama uusi tullut. Siis millaisia fantasioita kreationismissa on.

        ps. Se, että kirjoittaja vertaa Plutoa ja masta toisiinsa, osoittaa, ettei kirjoittajalle ole kovin kummoista käsitystä olosuhteista aurinkokuntamme eri planeetoilla. Mutta fantasiassa kaikki on mahdollista.


    • Anonyymi

      Itse asiassa oman kategorian muodostaa kuvitteelliset vedenpaisumuksen jäljet, joita on muun muassa tepuis vuoret ja Grand Canyon. Joku saarnamies on näidtä keksinyt myytin ja nyt sitä jotkut toistelee täyttä päätä. Molempien geologinen historia tunnetaan eikä mitään tulvasatua niiden synnyssä tarvita.

      • Anonyymi

        Kuvitellen globaalin vedenpaisumuksen jälkien ongelma kreationisteille on tietenkin se, että noiden jälkien pitäisi olla jotenkin globaaleja. Mitä muka ovat niitä KAIKILLE MANTEREILLE yhteisiä jälkiä, joita kuviteltu vedenpaisumus olisi jättänyt? Tällaisia globaaleja jälkiä ei ole, vaan on yksittäisiä kummallisuuksia kuten vaikka Grand Canyon josta innostutaan, mutta jolle ei löydy vastinetta muualta.

        Ja jos haluaa nähdä minkälaisia jälkiä äkilliset "vedenpaisumukset" jättää kannattaa mennä Islantiin. Siellä jäätiköiden alla tapahtuneet tulivuorenpurkaukset ovat synnyttäneet syöksytulvia. Ja noiden syöksytulvien jäljet eivät ole syviä uria tyyliin Grand Canyon, vaan päin vastoin matalia, leveitä loivasti mutkittelevia uomia.


      • Anonyymi
        Anonyymi kirjoitti:

        Kuvitellen globaalin vedenpaisumuksen jälkien ongelma kreationisteille on tietenkin se, että noiden jälkien pitäisi olla jotenkin globaaleja. Mitä muka ovat niitä KAIKILLE MANTEREILLE yhteisiä jälkiä, joita kuviteltu vedenpaisumus olisi jättänyt? Tällaisia globaaleja jälkiä ei ole, vaan on yksittäisiä kummallisuuksia kuten vaikka Grand Canyon josta innostutaan, mutta jolle ei löydy vastinetta muualta.

        Ja jos haluaa nähdä minkälaisia jälkiä äkilliset "vedenpaisumukset" jättää kannattaa mennä Islantiin. Siellä jäätiköiden alla tapahtuneet tulivuorenpurkaukset ovat synnyttäneet syöksytulvia. Ja noiden syöksytulvien jäljet eivät ole syviä uria tyyliin Grand Canyon, vaan päin vastoin matalia, leveitä loivasti mutkittelevia uomia.

        Grand Canyon on varmaan valikoitunut tuohon siksi, että se on tuolla jenkkilän raamattuvyöhykkeellä. Sitä on käynyt useampi lahkosaarnaaja ihmettelemässä ja inspiroitunut kertomaan uusia taruja.


      • Anonyymi
        Anonyymi kirjoitti:

        Kuvitellen globaalin vedenpaisumuksen jälkien ongelma kreationisteille on tietenkin se, että noiden jälkien pitäisi olla jotenkin globaaleja. Mitä muka ovat niitä KAIKILLE MANTEREILLE yhteisiä jälkiä, joita kuviteltu vedenpaisumus olisi jättänyt? Tällaisia globaaleja jälkiä ei ole, vaan on yksittäisiä kummallisuuksia kuten vaikka Grand Canyon josta innostutaan, mutta jolle ei löydy vastinetta muualta.

        Ja jos haluaa nähdä minkälaisia jälkiä äkilliset "vedenpaisumukset" jättää kannattaa mennä Islantiin. Siellä jäätiköiden alla tapahtuneet tulivuorenpurkaukset ovat synnyttäneet syöksytulvia. Ja noiden syöksytulvien jäljet eivät ole syviä uria tyyliin Grand Canyon, vaan päin vastoin matalia, leveitä loivasti mutkittelevia uomia.

        Tietysti voi sanoa, että Islannin pinta on jo ehtinyt kivettyä, eikä syövy uhtä helposti kuin esimerkiksi Mt.Sant Helenan ympäristö. Siellä muodostui kanjoni jo päivässä.

        https://dl0.creation.com/articles/p119/c11976/cliffface-lge.jpg


      • Anonyymi

      • Anonyymi
        Anonyymi kirjoitti:

        Tietysti voi sanoa, että Islannin pinta on jo ehtinyt kivettyä, eikä syövy uhtä helposti kuin esimerkiksi Mt.Sant Helenan ympäristö. Siellä muodostui kanjoni jo päivässä.

        https://dl0.creation.com/articles/p119/c11976/cliffface-lge.jpg

        Pehmeään maahan syntyy uria nopammin kuin kallioon ja noiden Mt. Sant Helenan kylkiin muodostuneiden urien mittasuhteet on kuitenkin AIVAN MUUTA kuin Grand Canyon.


      • Anonyymi
        Anonyymi kirjoitti:

        Pehmeään maahan syntyy uria nopammin kuin kallioon ja noiden Mt. Sant Helenan kylkiin muodostuneiden urien mittasuhteet on kuitenkin AIVAN MUUTA kuin Grand Canyon.

        Lisätään listaa nämä omintakeiset "todisteet", joilla kreationistit ovat varmistaneet tulvan olemassa olon.

        Taisi joskus joku olla varma, että hiekkaharjut oli varma todiste tulvasta, kun oli itse havannoinut harjua kävelylenkillään. Ja samalla hän kumosi mielestään jääkaudet.


      • Anonyymi
        Anonyymi kirjoitti:

        Lisätään listaa nämä omintakeiset "todisteet", joilla kreationistit ovat varmistaneet tulvan olemassa olon.

        Taisi joskus joku olla varma, että hiekkaharjut oli varma todiste tulvasta, kun oli itse havannoinut harjua kävelylenkillään. Ja samalla hän kumosi mielestään jääkaudet.

        Hyvä kun kumosi. Eihän hiekka nouse ylöspäin harjuksi, vaan leviää tietysti virtojen mukana ympäri aluetta.

        Olisi ollut aika outoa, jos ylhäältä jäätiköltä olisi valunut reunalta alas hiekkaa jäätikön sulaessa.

        Ei nnin, vaan hiekka on kasaantunut harjuiksi vedenpaisumuksen aikana veden kerääntyessä tyynempiin paikkoihin, niinkuin nytkin joissa.


      • Anonyymi
        Anonyymi kirjoitti:

        Hyvä kun kumosi. Eihän hiekka nouse ylöspäin harjuksi, vaan leviää tietysti virtojen mukana ympäri aluetta.

        Olisi ollut aika outoa, jos ylhäältä jäätiköltä olisi valunut reunalta alas hiekkaa jäätikön sulaessa.

        Ei nnin, vaan hiekka on kasaantunut harjuiksi vedenpaisumuksen aikana veden kerääntyessä tyynempiin paikkoihin, niinkuin nytkin joissa.

        Trolli vai parodia?


      • Anonyymi
        Anonyymi kirjoitti:

        Kuvitellen globaalin vedenpaisumuksen jälkien ongelma kreationisteille on tietenkin se, että noiden jälkien pitäisi olla jotenkin globaaleja. Mitä muka ovat niitä KAIKILLE MANTEREILLE yhteisiä jälkiä, joita kuviteltu vedenpaisumus olisi jättänyt? Tällaisia globaaleja jälkiä ei ole, vaan on yksittäisiä kummallisuuksia kuten vaikka Grand Canyon josta innostutaan, mutta jolle ei löydy vastinetta muualta.

        Ja jos haluaa nähdä minkälaisia jälkiä äkilliset "vedenpaisumukset" jättää kannattaa mennä Islantiin. Siellä jäätiköiden alla tapahtuneet tulivuorenpurkaukset ovat synnyttäneet syöksytulvia. Ja noiden syöksytulvien jäljet eivät ole syviä uria tyyliin Grand Canyon, vaan päin vastoin matalia, leveitä loivasti mutkittelevia uomia.

        Globaalin vedenpaisumuksen jälkeen järviä ainakin pitäisi olla ja meriäkin pitäisi olla ja Antarktiksellakin olisi pitänyt olla vedenpaisumuksen jälkeen jopa 8 km jäätä + lunta, ja siitä lähtien siis se on vain sulanut.. Eli eikö ole näin?


      • Anonyymi
        Anonyymi kirjoitti:

        Trolli vai parodia?

        Vai todellinen selitys?


      • Anonyymi
        Anonyymi kirjoitti:

        Hyvä kun kumosi. Eihän hiekka nouse ylöspäin harjuksi, vaan leviää tietysti virtojen mukana ympäri aluetta.

        Olisi ollut aika outoa, jos ylhäältä jäätiköltä olisi valunut reunalta alas hiekkaa jäätikön sulaessa.

        Ei nnin, vaan hiekka on kasaantunut harjuiksi vedenpaisumuksen aikana veden kerääntyessä tyynempiin paikkoihin, niinkuin nytkin joissa.

        Hiekka- eli moreeniharjuja tutkittiin jo 1800-luvulla jääkausikeskustelun tiimoilta...asiaan voi tutustua kirjallisuudessa, siis monivaiheinen keskustelun geologian tieteenalan synnyinvaiheena, sitä riitti 1700-luvun lopulta aina 1900-luvulle asti ja siihen liittyi monia merkittäviä tutkijoita.se on kiinnostavaa luettavaa.
        Esim alppijäätiköiden reunoilla ja alapuolella havaittiin olevan jäätiköiden aiheuttamia moreeniharjuja.


    • Anonyymi

      Okei kysyt evoluutikkona, että onko Maapallomme edes vettä keskimäärin 10 km maapallon ala verran, niin minä kysyn, jos ette tuota edes usko ja tiedä, niin miksi KIVEÄ voi olla yli 6378 km jalkojenne alla?! Älkää aiheuttako maailmaan uutta vedenpaimustakin sillä, että uskotte asioihin vasta kuin demonit ja pakanat...

    • Taivaankannen päällä on vettä. Tarvitsi vain avata taivaan ikkunat ja päästää vettä maailmaan sen verran että maa peittyi.

      • Anonyymi

        On jo "kreationistien selitykset" listassa.


    • Anonyymi

      Onko tosiaan niin vaikeaa käsittää, että Raamatun vedenpaisumuskertomus on arkissa olevien kuva tapahtumista, eikä kaikenkattava tieteellinen selitys.

      He pystyivät näkemään veden nousun, ja kuinka vähitellen kaikki arkista näkyvät vuoretkin peittyivät usean metrin.

      Ei tuo kertomus sano mitään siitä mitä ei näkynyt.

      Siksi voimme aivan huoletta olettaa, että vesi kylläkin pyyhki eläimet ja ihmiset pois maan päältä, mutta jos samalla toisella puolen maailmaa maa nousi vastaavasti, sitä voimme vain kuvitella. Samoin on kuvittelua että esimerkiksi vesi olisi peittänyt Tiibetin vuorenhuiput, jos ne olivat jo silloin. Ei vuorten hävitys kuulunut ohjelmaan, ne saivat olla rauhassa.

      Koska maailmassa on nytkin aivan tarpeeksi vettä peittämään koko maapallon kolmen kilometrin vesikerrokseen, vettä oli aivan tarpeeksi,

      Tietysti joku hölmöläinen ei käsitä vertausta, mutta ajatellaan kiinalaista talonpoikaa:
      Ensin hän laskee vettä yhdelle pellolle, istuttaa riisiä veteen, ja kun se alkaa kasvaa, hän laskee veden sieltä toiselle pellolle. istuttaa riisiä, ja myöhemmin laskee vettä kolmannelle pellolle.
      Ei hänen tarvitse peittää joka peltoa samaan aikaan, vaan sama vesi riittää peittämään usean pellon.

      Samalla lailla vedenpaisumus muovaili eri alueita, siirtyen alueelta toiselle.

      Mahdotonta, sanot! Etkö ymmärrä?

      Levitä pressu pihamaalle ja ruiskuta sille vettä muutaman sentin verran.
      Jos työnnät vaikka laudan pressun alle, huomaat, että vesi siirtyy toiseen paikkaan.
      Jos nostat pressua hiukan, heti vesi lisääntyy toisella kohtaa.

      Tajuat siis nyt vihdoinkin, että aivan mitättömän suuruinen alustan nousu saa veden liikkeelle.
      Samalla lailla jos maan kuori nousi tai laski edes yhden promillen verran eri paikoilla, vesi virtasi valtavalla vauhdilla.
      Arkille siillä ei olisi ollut merkitystä, koska ei ollut kyseessä myrsky, joka puskee aaltoja, vaan tsunami, mitä ei laivassa huomaa ellei ole mittaria mikä näyttäisi että on noussut jonkun metrin viimeisten tuntien aikana. Vaikutus on sama kuin sulkujen välissä kanavassa, muutaman metrin nousun tajuaa vain näkemällä ympäristön liikkuvan.

      Tämä on täysin mahdotonta ateistin ymmärtää, joten toivon edes jonkun kreationistin ymmärtävän.

      Mikä olisi voinut saada maan kuoren aaltoilemaan aavistuksen verran? Asteroidi esimerkiksi. Se olisi voinut vaikka mennä maan kuoren läpi Tiibetin kohdalta, kääntäen samalla maan kuoren murusia kyljelleen, niinkuin näkyy monien jyrkkien vuorten kerrostumien suunnasta. Samoin olisi asteroidi iskeä keskelle valtamerta, niiden pohjahan on enimmäkseen tuntematonta kilometrien syvyydessä.

      Kun maan kuori jällen tuli tasapainoon, se ei tarkoittanut, että kaikki olisi ollut kuin ennen, vaan jotkut kohdat jäivät syvemmälle, toiset korkeammalle.

      Siksi löydämme yhä kaupunkien raunioita merestä kymmenien, jopa satojen metrien syvyydestä, samoin kuin löydämme kalojen fossiileja preerioilta ja vuorten rinteiltä.

      Tietysti täämä on aivan mahdotonta käsittää. Sen sijaan on monelle päivänselvää, että mantereiden vedenpäälliset osat vaeltelevat sinne tänne, vaikka vesi muodostaa vain alle promillen maapallon pinnassa, ja veden alla on 6000 kilometriä paksulta kiveä ja magmaa ja metallia.

      • Anonyymi

        "Onko tosiaan niin vaikeaa käsittää, että Raamatun vedenpaisumuskertomus on arkissa olevien kuva tapahtumista, eikä kaikenkattava tieteellinen selitys."

        Se ei ole kirjoitettu minä-muotoon, vaan jos olisit lukenut Raamatun, niin tietäisit, että vedenpaisumuskertomus on jälkikäteen paljon väitetyn vedenpaisumuksen jälkeen kirjoitettu tarina.

        "Samoin on kuvittelua että esimerkiksi vesi olisi peittänyt Tiibetin vuorenhuiput, jos ne olivat jo silloin. Ei vuorten hävitys kuulunut ohjelmaan, ne saivat olla rauhassa."

        KOKO vedenpaisumuskertomus on kuvittelua. Arkki, villieläinten saaminen arkkiin, veden ilmestyminen, katoaminen jne. Itse asiassa Nooa-satu on yksi versio paljon vanhemmasta babylonialaisesta tulvakertomuksesta, jossa Nooan esikuva Utnapistim ottaa "arkkiinsa" vain kotieläimiä. On esitetty väitteitä, että Utnapistim-tarina olisi puolestaan versio vielä varhaisemmasta Indus-kulttuurista juontavasta tarinasta.

        "Siksi voimme aivan huoletta olettaa, että vesi kylläkin pyyhki eläimet ja ihmiset pois maan päältä..."

        Tuollainen oletus on kaiken sen vastainen mitä tiedämme ihmiskunnan historiasta, genetiikasta ja kulttuurihistoriasta. On 100%:n varmaa, että ihmiskunnan kaikki eri kulttuurit eivät ole peräisin 4500 vuoden takaisesta lähi-itäläisestä rautakautisesta kulttuurista. Itse asiassa Lähi-Idässä ei ollut 4500 vuotta sitten rautakautista kulttuuria ensinkään, vaan elettiin pronssikautta, mutta Genesis on kirjoitettu niin myöhään, että koko pronssikausi oli päässyt jo unohtumaan.

        "Samalla lailla jos maan kuori nousi tai laski edes yhden promillen verran eri paikoilla, vesi virtasi valtavalla vauhdilla. Arkille siillä ei olisi ollut merkitystä..."

        Jos vedenpaisumus olisi ollut tullut tsunamina, niin se tsunami olisi murskannut hataran arkin tuhannen päreiksi, koska arkkihan rakennettiin maalle eikä sitä laskettu vesille. Raamatussa mainitaan arkin kooksi noin 150m x 25m eikä näin suuri puulaiva pysy millään koossa merenkäynnissä muutenkaan. Suurin koskaan rakennettu puulaiva oli jykevin rautatangoin vahvistettu kuunari Wyoming, jonka kansipituus oli 110m ja leveys 15m ja sitä piti pumpata jatkuvasti, koska puu jousti ja laiva vuosi. On silkkaa mielikuvitusta, että 4500 vuotta aikaisemmin olisi osattu rakentaa suurempi merikelpoinen lautta kuin KOSKAAN sen jälkeen.

        "Tajuat siis nyt vihdoinkin, että aivan mitättömän suuruinen alustan nousu saa veden liikkeelle."

        Tuo "mitätön promille" maan halkaisijasta on 12 km. Tapaninpäivän tsunamin aiheutti joidenkin metrien pohjan korkeuden muutos YHDESSÄ paikassa merenpohjaa.

        "Kun maan kuori jällen tuli tasapainoon, se ei tarkoittanut, että kaikki olisi ollut kuin ennen, vaan jotkut kohdat jäivät syvemmälle, toiset korkeammalle."

        Raamatun kertomuksessa kaikki oli niin kuin ennen. Kun tulva oli ohi, niin "maa oli kokonaan kuiva". Olet taas niitä kreationisteja, jotka yrittää todistaa Jumalan sanassa kerrotun tulvatarinan todeksi keksimällä itse paremman.


      • Anonyymi

        Hirveän pitkä selitys.

        Oletko siis sitä mieltä, että maapallon vesimäärä riitti peittämään korkeimmatkin vuoret, koska ne olivat niin matalia? Kyllä/ei. Ja sen jälkeen ne pikakohosi nykyisiin? Kyllä/ei. Eli kilometrien maankohoaminen varsin lyhyessä ajassa? Kyllä/ei.

        Yli 8 tonniset olivat Raamatun mukaan veden peitossa. Kyllä/ei.

        ps. Tämä on muuten ristiriidassa tuon taivaan kannen jutun kanssa, joten kreationistien pitäis saada tähän selvyys keskenään.


      • Anonyymi
        Anonyymi kirjoitti:

        Hirveän pitkä selitys.

        Oletko siis sitä mieltä, että maapallon vesimäärä riitti peittämään korkeimmatkin vuoret, koska ne olivat niin matalia? Kyllä/ei. Ja sen jälkeen ne pikakohosi nykyisiin? Kyllä/ei. Eli kilometrien maankohoaminen varsin lyhyessä ajassa? Kyllä/ei.

        Yli 8 tonniset olivat Raamatun mukaan veden peitossa. Kyllä/ei.

        ps. Tämä on muuten ristiriidassa tuon taivaan kannen jutun kanssa, joten kreationistien pitäis saada tähän selvyys keskenään.

        Ei ole asiani sanoa miten isot vuoret peittyivät.
        Vedet tekivät tehtävänsä. Mitä hyötyä olisi ollut pumpata maapallolle enemmän vettä kuin tarvittiin siihen mikä oli tarkoitus.
        Eikai siinä olisi ollut mitään järkeä peittää tyhjät kalliot vain sen tähden, että evoluutionisteilla olisi ihmettelemistä. Miksi esimerkiksi Mt .Everest olisi pitänyt panna veteen likoamaan; puhdistaahan sateet sitä jatkuvasti.

        Vedenpaisumus teki tehtävänsä, vettä riitti aivan tarpeeksi. Maa muuttui karuksi vuorten ja kanjonien halkomaksi monin osin aivan hedelmättömäksi erämaaksi.

        Oli varmasti hyvä, jos ei tarvinnut taistella Tyrannosaurusten kanssa.

        Ihmiset joutuivat hankkimaan toimeentulonsa kovalla työllä, jolloin oli edes toiveita että he eivät ehtineet tekemään pahaa koko ajan.

        Harmittaako se, jos ei voi viisastella jonkun vuoren korkeudella, ja kuinka paljon vettä tarvittiin?
        Tuskin Lissabonin maanjäristyksestä aikoinaan selvinneet henkilöt pohtivat mielessään, oliko vettä riittävästi tappamaan satatuhatta!


      • Anonyymi
        Anonyymi kirjoitti:

        Ei ole asiani sanoa miten isot vuoret peittyivät.
        Vedet tekivät tehtävänsä. Mitä hyötyä olisi ollut pumpata maapallolle enemmän vettä kuin tarvittiin siihen mikä oli tarkoitus.
        Eikai siinä olisi ollut mitään järkeä peittää tyhjät kalliot vain sen tähden, että evoluutionisteilla olisi ihmettelemistä. Miksi esimerkiksi Mt .Everest olisi pitänyt panna veteen likoamaan; puhdistaahan sateet sitä jatkuvasti.

        Vedenpaisumus teki tehtävänsä, vettä riitti aivan tarpeeksi. Maa muuttui karuksi vuorten ja kanjonien halkomaksi monin osin aivan hedelmättömäksi erämaaksi.

        Oli varmasti hyvä, jos ei tarvinnut taistella Tyrannosaurusten kanssa.

        Ihmiset joutuivat hankkimaan toimeentulonsa kovalla työllä, jolloin oli edes toiveita että he eivät ehtineet tekemään pahaa koko ajan.

        Harmittaako se, jos ei voi viisastella jonkun vuoren korkeudella, ja kuinka paljon vettä tarvittiin?
        Tuskin Lissabonin maanjäristyksestä aikoinaan selvinneet henkilöt pohtivat mielessään, oliko vettä riittävästi tappamaan satatuhatta!

        Kysymykset eivät ehkä olleet sinulle vaan henkilölle, joka geologisena totuutena on julistanut maanpinnan aaltoilua vedenpaisumuksen selityksenä, jopa asteroidin törmäyksen seurauksena. Lisäksi tämä neropatin pieni promillen muutos on kuulemma ihan normaalia. Se on 6 km pystysuoraa siirrosta.

        Nyt tuo kieroilija yritti häivyttää pressuselityksellä omien selitystensä mielipuolisuutta.

        Kun näitä järjettömyyksiä kääntää sellaisiksi, mitä ne todellisuudessa yksinkertaisesti olisivat, alkaa kieroilu, gish gallop tyylinen spämmäys ja puheenaiheen vaihto. Eikä ole ainakaan vielä tullut vastausta helppoihin kyllä/ei kysymyksiin, jotka tarkentaa/varmistaa kretun edelliset väitteet.

        Muille lukijoille: tämä on ilmeisesti se kreationistinen "tieteellinen menetelmä".

        ps. Vielä jää nyt auki kumpi kreationisti on oikeassa, kun jo tähän ketjuun tuli kaksi täysin poikkeavaa selitystä vesien alkuperästä.


      • Anonyymi
        Anonyymi kirjoitti:

        Ei ole asiani sanoa miten isot vuoret peittyivät.
        Vedet tekivät tehtävänsä. Mitä hyötyä olisi ollut pumpata maapallolle enemmän vettä kuin tarvittiin siihen mikä oli tarkoitus.
        Eikai siinä olisi ollut mitään järkeä peittää tyhjät kalliot vain sen tähden, että evoluutionisteilla olisi ihmettelemistä. Miksi esimerkiksi Mt .Everest olisi pitänyt panna veteen likoamaan; puhdistaahan sateet sitä jatkuvasti.

        Vedenpaisumus teki tehtävänsä, vettä riitti aivan tarpeeksi. Maa muuttui karuksi vuorten ja kanjonien halkomaksi monin osin aivan hedelmättömäksi erämaaksi.

        Oli varmasti hyvä, jos ei tarvinnut taistella Tyrannosaurusten kanssa.

        Ihmiset joutuivat hankkimaan toimeentulonsa kovalla työllä, jolloin oli edes toiveita että he eivät ehtineet tekemään pahaa koko ajan.

        Harmittaako se, jos ei voi viisastella jonkun vuoren korkeudella, ja kuinka paljon vettä tarvittiin?
        Tuskin Lissabonin maanjäristyksestä aikoinaan selvinneet henkilöt pohtivat mielessään, oliko vettä riittävästi tappamaan satatuhatta!

        Vedenpaisumuksen vesimäärä on järjetön vaikka sitä yrittäisi selitellä millä tarinalla pienemmäksi. Kuivan maan keskikorkeus on 840 metriä, mikä tarkoittaa että jo sen saavuttaminen edellytti 428484000 kuutiokilometrin vesimäärää. Yli 400 MILJOONAA kuutiokilometriä vettä joka *pim* ilmestyy "jostain" ja *hoccus-poccus* katoaa jonnekkin JÄLKIÄ JÄTTÄMÄTTÄ. Tuo 400 miljoonaa litraa on siis 20,.000 x Itämeren tilavuus. KAKSIKYMMENTÄTUHATTA Itämerta olisi pitänyt ilmestyä "jostakin" että saavutettaisiin mantereiden KESKIKORKEUS.

        Raamatun mukaan maa ei muuttunut karuksi vedenpaisumuksessa. Et selvästikkään tunne Raamattua ja sekoitat vedenpaisumuksen ja paratiisista karkoituksen. Kun Raamatun tarinassa Aadam karkoitettiin paratiisissa Jumala sanoi:
        "olkoon maa sinun takiasi kirottu. Kovalla työllä sinun on hankittava siitä elantosi niin kauan kuin elät. Maa kasvaa sinulle orjantappuraa ja ohdaketta, mutta sen kasveista joudut ottamaan ravintosi. Otsa hiessä sinun on hankittava leipäsi,..."

        Sen sijaan vedenpaisumuksen jälkeen Nooa alkoi tunnetusti viljelemään viiniä joka pikavauhtia tuotti myös satoa tunnetuin seurauksin, kun Jumalan hurskaimmaksi luokittelemä mies osottautui rapajuopoksi.


      • Anonyymi
        Anonyymi kirjoitti:

        Kysymykset eivät ehkä olleet sinulle vaan henkilölle, joka geologisena totuutena on julistanut maanpinnan aaltoilua vedenpaisumuksen selityksenä, jopa asteroidin törmäyksen seurauksena. Lisäksi tämä neropatin pieni promillen muutos on kuulemma ihan normaalia. Se on 6 km pystysuoraa siirrosta.

        Nyt tuo kieroilija yritti häivyttää pressuselityksellä omien selitystensä mielipuolisuutta.

        Kun näitä järjettömyyksiä kääntää sellaisiksi, mitä ne todellisuudessa yksinkertaisesti olisivat, alkaa kieroilu, gish gallop tyylinen spämmäys ja puheenaiheen vaihto. Eikä ole ainakaan vielä tullut vastausta helppoihin kyllä/ei kysymyksiin, jotka tarkentaa/varmistaa kretun edelliset väitteet.

        Muille lukijoille: tämä on ilmeisesti se kreationistinen "tieteellinen menetelmä".

        ps. Vielä jää nyt auki kumpi kreationisti on oikeassa, kun jo tähän ketjuun tuli kaksi täysin poikkeavaa selitystä vesien alkuperästä.

        Evoluutionisteilla ei ole edes kykyä ajatella itse, siksi he esittävät aina samaa teoriaa, jota ei ole edes kyetty vahvistamaan.

        Jos taas ihmiset ajattelevat, he tietysti esittävät eri teorioita.

        On parempi ajatella kuin syödä mitä syötetäään.

        Ei tosiaan voi selittää niin yksinkertaisesti asiuoita, että evoluutio koskaan käsittäisi edes omaa teoriaansa.

        Vai oletteko jo vihdoinkin ymmärtäneet, että ei tarvotse enempää vettä, tarvitsee vain järjenkäyttöä. Kaikilta se ei onnistu, joten antaa sellaisten jatkaa kuvitteluaan siitä kuinka koko maapallon peittävät mannerlaatat välillä kokoontuvat jollekin puolelle maapalloa.

        Olisi varmaan liiaksi vaadittu, jos he tajuaisivat teoriansa mahdottomuuden.

        Sen sijaan hyvin mitätön yhden promillen nousu tai lasku korkeussuunnassa on aivan mahdotonta heidän käsittää.

        Eivät kai osaa matem,atiikkaa. Yksi milli on tuhannesosa metriä. Ei se ole suuri nousu!

        Ei kaikkien, edes kreationistien, tarvitse uskoa samalla tavalla. Ainakin minä voin hyvin lukea eri selityksiä.


      • Anonyymi
        Anonyymi kirjoitti:

        Vedenpaisumuksen vesimäärä on järjetön vaikka sitä yrittäisi selitellä millä tarinalla pienemmäksi. Kuivan maan keskikorkeus on 840 metriä, mikä tarkoittaa että jo sen saavuttaminen edellytti 428484000 kuutiokilometrin vesimäärää. Yli 400 MILJOONAA kuutiokilometriä vettä joka *pim* ilmestyy "jostain" ja *hoccus-poccus* katoaa jonnekkin JÄLKIÄ JÄTTÄMÄTTÄ. Tuo 400 miljoonaa litraa on siis 20,.000 x Itämeren tilavuus. KAKSIKYMMENTÄTUHATTA Itämerta olisi pitänyt ilmestyä "jostakin" että saavutettaisiin mantereiden KESKIKORKEUS.

        Raamatun mukaan maa ei muuttunut karuksi vedenpaisumuksessa. Et selvästikkään tunne Raamattua ja sekoitat vedenpaisumuksen ja paratiisista karkoituksen. Kun Raamatun tarinassa Aadam karkoitettiin paratiisissa Jumala sanoi:
        "olkoon maa sinun takiasi kirottu. Kovalla työllä sinun on hankittava siitä elantosi niin kauan kuin elät. Maa kasvaa sinulle orjantappuraa ja ohdaketta, mutta sen kasveista joudut ottamaan ravintosi. Otsa hiessä sinun on hankittava leipäsi,..."

        Sen sijaan vedenpaisumuksen jälkeen Nooa alkoi tunnetusti viljelemään viiniä joka pikavauhtia tuotti myös satoa tunnetuin seurauksin, kun Jumalan hurskaimmaksi luokittelemä mies osottautui rapajuopoksi.

        Taas yksi asiantuntija! Oliko se pikavauhtia, jos siinä vaiheessa kun Nooa istutti viinitarhansa, Haamilla oli jo poika juoksentelemassa, ilmeisesti osasyyllisenä skandaalin paljastamisessa.

        Ennen vedenpaisumusta maapaaloa ei oltu mitenkään muotoiltu uudella tavalla.

        Vedenpaisumuksessa metsät peittyivät puolen kilometrin maamäärän alle, ja muuttuivat kivihiileksi ja öljyksi.

        Tietysti joihinkin paikkoihin kertyi myös ruokamaata, kutan mustan mullan maihin, joissa sitä voi olla viidenkymmenen metrin paksuudelta.

        Suomessa mullat huuhtoutui savon kukkuloilta jonnekin Uudelle Maalle.

        Ajatelkaa, että ei edes tropiikin sademetsissä ole kuin muutama kymmenen
        senttiä ruokamaata.

        Joten jokainen järkevä ihminen voi todeta, että maapallo on muuttunut karuksi monin paikoin veden paisumuksen jälkeen.
        Myös kivettyneet metsät kertovat kasvillisuuden runsaudesta ennen vedenpaisumusta.


      • Anonyymi
        Anonyymi kirjoitti:

        Evoluutionisteilla ei ole edes kykyä ajatella itse, siksi he esittävät aina samaa teoriaa, jota ei ole edes kyetty vahvistamaan.

        Jos taas ihmiset ajattelevat, he tietysti esittävät eri teorioita.

        On parempi ajatella kuin syödä mitä syötetäään.

        Ei tosiaan voi selittää niin yksinkertaisesti asiuoita, että evoluutio koskaan käsittäisi edes omaa teoriaansa.

        Vai oletteko jo vihdoinkin ymmärtäneet, että ei tarvotse enempää vettä, tarvitsee vain järjenkäyttöä. Kaikilta se ei onnistu, joten antaa sellaisten jatkaa kuvitteluaan siitä kuinka koko maapallon peittävät mannerlaatat välillä kokoontuvat jollekin puolelle maapalloa.

        Olisi varmaan liiaksi vaadittu, jos he tajuaisivat teoriansa mahdottomuuden.

        Sen sijaan hyvin mitätön yhden promillen nousu tai lasku korkeussuunnassa on aivan mahdotonta heidän käsittää.

        Eivät kai osaa matem,atiikkaa. Yksi milli on tuhannesosa metriä. Ei se ole suuri nousu!

        Ei kaikkien, edes kreationistien, tarvitse uskoa samalla tavalla. Ainakin minä voin hyvin lukea eri selityksiä.

        Kyllä on kova hinku sitkea keskudtelu. Yllä on muutama kysymys koskien vesimäärää, joka peitti maat ja vuoret. Ei luulisi olevan vaikea vastata noin helppoihin kysymyksiin. kyllä/ei.

        Yhden promillen nousu maan säteestä on 6km, siis tuhannesosa maapallon säteestä. Olet siis esittänyt, että maanpinta kohosi pikavauhtia n. 6 kilometriä. Lukitaanko vastaus?


      • Anonyymi
        Anonyymi kirjoitti:

        Vedenpaisumuksen vesimäärä on järjetön vaikka sitä yrittäisi selitellä millä tarinalla pienemmäksi. Kuivan maan keskikorkeus on 840 metriä, mikä tarkoittaa että jo sen saavuttaminen edellytti 428484000 kuutiokilometrin vesimäärää. Yli 400 MILJOONAA kuutiokilometriä vettä joka *pim* ilmestyy "jostain" ja *hoccus-poccus* katoaa jonnekkin JÄLKIÄ JÄTTÄMÄTTÄ. Tuo 400 miljoonaa litraa on siis 20,.000 x Itämeren tilavuus. KAKSIKYMMENTÄTUHATTA Itämerta olisi pitänyt ilmestyä "jostakin" että saavutettaisiin mantereiden KESKIKORKEUS.

        Raamatun mukaan maa ei muuttunut karuksi vedenpaisumuksessa. Et selvästikkään tunne Raamattua ja sekoitat vedenpaisumuksen ja paratiisista karkoituksen. Kun Raamatun tarinassa Aadam karkoitettiin paratiisissa Jumala sanoi:
        "olkoon maa sinun takiasi kirottu. Kovalla työllä sinun on hankittava siitä elantosi niin kauan kuin elät. Maa kasvaa sinulle orjantappuraa ja ohdaketta, mutta sen kasveista joudut ottamaan ravintosi. Otsa hiessä sinun on hankittava leipäsi,..."

        Sen sijaan vedenpaisumuksen jälkeen Nooa alkoi tunnetusti viljelemään viiniä joka pikavauhtia tuotti myös satoa tunnetuin seurauksin, kun Jumalan hurskaimmaksi luokittelemä mies osottautui rapajuopoksi.

        ...ja sitten on vielä se hupaisa pikku yksityiskohta jota ei (edes) Raamatussa mainita lainkaan, mutta luominen.fi ssa kyllä; kun maa oli kokonaan kuiva seurasikin 700 vuotta kestänyt jääkausi....
        Mutta se nyt on ihan pikkujuttu verrattuna ihmeelliseen vedenpaisumukseen joten turha edes mainita


      • Anonyymi
        Anonyymi kirjoitti:

        Taas yksi asiantuntija! Oliko se pikavauhtia, jos siinä vaiheessa kun Nooa istutti viinitarhansa, Haamilla oli jo poika juoksentelemassa, ilmeisesti osasyyllisenä skandaalin paljastamisessa.

        Ennen vedenpaisumusta maapaaloa ei oltu mitenkään muotoiltu uudella tavalla.

        Vedenpaisumuksessa metsät peittyivät puolen kilometrin maamäärän alle, ja muuttuivat kivihiileksi ja öljyksi.

        Tietysti joihinkin paikkoihin kertyi myös ruokamaata, kutan mustan mullan maihin, joissa sitä voi olla viidenkymmenen metrin paksuudelta.

        Suomessa mullat huuhtoutui savon kukkuloilta jonnekin Uudelle Maalle.

        Ajatelkaa, että ei edes tropiikin sademetsissä ole kuin muutama kymmenen
        senttiä ruokamaata.

        Joten jokainen järkevä ihminen voi todeta, että maapallo on muuttunut karuksi monin paikoin veden paisumuksen jälkeen.
        Myös kivettyneet metsät kertovat kasvillisuuden runsaudesta ennen vedenpaisumusta.

        Edelleen kukaan teistä ihan itse ajattelevista (sic!) ei ole kommentoinut miten voi olla että tämä myyttinen vedenpaisumus joka muutti puoli kilometrin korkeudelta metsät öljyksi, kurssi Grand Canyonin jne jne, ei sekoittanut merenpohjien sedimenttikerroksia lainkaan. Eikä murtanut rakenteilla olleita pyramideja Egyptissä, ei siirtänyt esim todisteita savitauluilla nuolenpäääkirjoituksesta Sumerissa yhtään minnekään, ei huuhtonut ulos luolien sisältämä neandertalilaisten ja mammuttien luita ym ym ym. Vedenpaisumus sai vuoret kohoamaan hetkessä (voi hyvä tavaton) mutta suuria osia ympäristöstä pysyi kauniin koskemattomana....eikä koko vedenpaisumusmyytin edes ole muualla kuin Lähi-idän uskonnoissa, how come? Miksei Kiinassa tai Amerikassa?
        JOS vedenpaisumustaru olisi mm geologian kanssa yhtäpitävä, sen seuraukset havaittaisiin objektiivisesti myös siellä, missä koko myyttiä ei edes tunneta! Havaintojen perusteella. Mutta tällaisia oikeaksi huom todistettuja havaintoja ei vaan ole...


      • Anonyymi
        Anonyymi kirjoitti:

        Evoluutionisteilla ei ole edes kykyä ajatella itse, siksi he esittävät aina samaa teoriaa, jota ei ole edes kyetty vahvistamaan.

        Jos taas ihmiset ajattelevat, he tietysti esittävät eri teorioita.

        On parempi ajatella kuin syödä mitä syötetäään.

        Ei tosiaan voi selittää niin yksinkertaisesti asiuoita, että evoluutio koskaan käsittäisi edes omaa teoriaansa.

        Vai oletteko jo vihdoinkin ymmärtäneet, että ei tarvotse enempää vettä, tarvitsee vain järjenkäyttöä. Kaikilta se ei onnistu, joten antaa sellaisten jatkaa kuvitteluaan siitä kuinka koko maapallon peittävät mannerlaatat välillä kokoontuvat jollekin puolelle maapalloa.

        Olisi varmaan liiaksi vaadittu, jos he tajuaisivat teoriansa mahdottomuuden.

        Sen sijaan hyvin mitätön yhden promillen nousu tai lasku korkeussuunnassa on aivan mahdotonta heidän käsittää.

        Eivät kai osaa matem,atiikkaa. Yksi milli on tuhannesosa metriä. Ei se ole suuri nousu!

        Ei kaikkien, edes kreationistien, tarvitse uskoa samalla tavalla. Ainakin minä voin hyvin lukea eri selityksiä.

        "Eivät kai osaa matem,atiikkaa. Yksi milli on tuhannesosa metriä. Ei se ole suuri nousu"

        Mitä oikein sössötät millimetristä, kun itse olet puhunut maapallon säteen promillen muutoksista. Se on hieman enemmän kuin 1 mm. Osaatko sinä matikkanero laskea, kuinka korkeita maanpinnan "aaltoja" oletkaan kuvitellutkaan?


      • Anonyymi
        Anonyymi kirjoitti:

        Taas yksi asiantuntija! Oliko se pikavauhtia, jos siinä vaiheessa kun Nooa istutti viinitarhansa, Haamilla oli jo poika juoksentelemassa, ilmeisesti osasyyllisenä skandaalin paljastamisessa.

        Ennen vedenpaisumusta maapaaloa ei oltu mitenkään muotoiltu uudella tavalla.

        Vedenpaisumuksessa metsät peittyivät puolen kilometrin maamäärän alle, ja muuttuivat kivihiileksi ja öljyksi.

        Tietysti joihinkin paikkoihin kertyi myös ruokamaata, kutan mustan mullan maihin, joissa sitä voi olla viidenkymmenen metrin paksuudelta.

        Suomessa mullat huuhtoutui savon kukkuloilta jonnekin Uudelle Maalle.

        Ajatelkaa, että ei edes tropiikin sademetsissä ole kuin muutama kymmenen
        senttiä ruokamaata.

        Joten jokainen järkevä ihminen voi todeta, että maapallo on muuttunut karuksi monin paikoin veden paisumuksen jälkeen.
        Myös kivettyneet metsät kertovat kasvillisuuden runsaudesta ennen vedenpaisumusta.

        Jos halutaan tietää, mihin johtopäätökseen järkevät ihmiset ovat faktaperusteisti tulleet kannaattaa tulustua tieteen tuloksiin.

        Nyt puhuttiin Raamatun vedenpaisumustarinasta ja siitä, miten uskovaiset sitä selittelevät eikä siitä, mihin johtopäätöksiin järkevät ihmiset voivat tulla faktapohjaisesti.

        Selitykset siitä, että metsät muuttuivat ruokamullaksi ja öljyksi (yhtä aikaa?) on yhtä naurettavaa kuin kaikki muutkin kretujen selittelyt. Syvimmät öljylähteet ovat muuten yli 10km syviä ja todennäköisesti syy, ettei syvempiä ole on se, että syvemmältä pumppaaminen on tekninesti vaikeaa ja/tai kaupallisesti kannattamatonta.


      • Anonyymi
        Anonyymi kirjoitti:

        Edelleen kukaan teistä ihan itse ajattelevista (sic!) ei ole kommentoinut miten voi olla että tämä myyttinen vedenpaisumus joka muutti puoli kilometrin korkeudelta metsät öljyksi, kurssi Grand Canyonin jne jne, ei sekoittanut merenpohjien sedimenttikerroksia lainkaan. Eikä murtanut rakenteilla olleita pyramideja Egyptissä, ei siirtänyt esim todisteita savitauluilla nuolenpäääkirjoituksesta Sumerissa yhtään minnekään, ei huuhtonut ulos luolien sisältämä neandertalilaisten ja mammuttien luita ym ym ym. Vedenpaisumus sai vuoret kohoamaan hetkessä (voi hyvä tavaton) mutta suuria osia ympäristöstä pysyi kauniin koskemattomana....eikä koko vedenpaisumusmyytin edes ole muualla kuin Lähi-idän uskonnoissa, how come? Miksei Kiinassa tai Amerikassa?
        JOS vedenpaisumustaru olisi mm geologian kanssa yhtäpitävä, sen seuraukset havaittaisiin objektiivisesti myös siellä, missä koko myyttiä ei edes tunneta! Havaintojen perusteella. Mutta tällaisia oikeaksi huom todistettuja havaintoja ei vaan ole...

        Suomessakin on järviä joiden pohjasedimentit yli 8000 vuoden ajalta ovat pysyneet ihan järjestyksessä eivätkä huomanneet lainkaan kilometrien korkuisia tsunameja. Suomi oli varmaan jo tuolloin niin kaukana sieltä, missä kaikki jännä tapahtuu.


      • Anonyymi
        Anonyymi kirjoitti:

        Ei ole asiani sanoa miten isot vuoret peittyivät.
        Vedet tekivät tehtävänsä. Mitä hyötyä olisi ollut pumpata maapallolle enemmän vettä kuin tarvittiin siihen mikä oli tarkoitus.
        Eikai siinä olisi ollut mitään järkeä peittää tyhjät kalliot vain sen tähden, että evoluutionisteilla olisi ihmettelemistä. Miksi esimerkiksi Mt .Everest olisi pitänyt panna veteen likoamaan; puhdistaahan sateet sitä jatkuvasti.

        Vedenpaisumus teki tehtävänsä, vettä riitti aivan tarpeeksi. Maa muuttui karuksi vuorten ja kanjonien halkomaksi monin osin aivan hedelmättömäksi erämaaksi.

        Oli varmasti hyvä, jos ei tarvinnut taistella Tyrannosaurusten kanssa.

        Ihmiset joutuivat hankkimaan toimeentulonsa kovalla työllä, jolloin oli edes toiveita että he eivät ehtineet tekemään pahaa koko ajan.

        Harmittaako se, jos ei voi viisastella jonkun vuoren korkeudella, ja kuinka paljon vettä tarvittiin?
        Tuskin Lissabonin maanjäristyksestä aikoinaan selvinneet henkilöt pohtivat mielessään, oliko vettä riittävästi tappamaan satatuhatta!

        "Harmittaako se, jos ei voi viisastella jonkun vuoren korkeudella, ja kuin..."

        Ei lainkaan harmita nämä s24:n jutut. Kiinnostaa vain nähdä, miten kreationistien "luonnontiede" vänkää todellisuutta ;)


    • Anonyymi

      Miten evoluutio selittää ihmisen vesimäärän? Miksi ihmiseen tarvitaan niin paljon, eikö vähemmälläkin pärjättäisi?

      Kun oli pelkkiä hyönteisiä, jotka on melko kuivakkaita, niin elävyys kaikkeen tulee veden kautta. Kun ihminen on pahasti kuivunut, niin ei siinä eloa juuri ole. Muumiotkin on aika kuivia ja elottomia.

      • Anonyymi

        Mitä hittoa evoluutiolla on tekemistä tämän keskustekyn kanssa?!? Kysymys on tulvatarun vesimäärän mysteeristä.


      • Anonyymi
        Anonyymi kirjoitti:

        Mitä hittoa evoluutiolla on tekemistä tämän keskustekyn kanssa?!? Kysymys on tulvatarun vesimäärän mysteeristä.

        Juuri näin! Kreationistien epätoivo suorastaan haisee näissä "entä evoluutio" -kysymyksissä, joista suorataan haisee epätoivo, kun oman uskomuksen tueksi ei ole pienintäkään perustelun sirpaletta.

        Elämä kehittyi vedessä ja siksi jokaisessa solussa on pisara alkumerta, jonka suolaisuus oli n. 1% (kuten ihmisen).


      • Anonyymi
        Anonyymi kirjoitti:

        Juuri näin! Kreationistien epätoivo suorastaan haisee näissä "entä evoluutio" -kysymyksissä, joista suorataan haisee epätoivo, kun oman uskomuksen tueksi ei ole pienintäkään perustelun sirpaletta.

        Elämä kehittyi vedessä ja siksi jokaisessa solussa on pisara alkumerta, jonka suolaisuus oli n. 1% (kuten ihmisen).

        Onko koppakuoriaisessakin tuo alkumeri ja sama suhteellinen vesimäärä kuin ihmisessä? Kertokaa, kun olette niin viisaita.

        En ole kreationisti, mutta sanonta vedestä ja hengestä voisi pitää paikkansa, siten, että henki, joka tarkoittaisi jonkinlaista sähköistä toimintaa yhdistettynä veteen, joka elävöittää ruumiin, voisi liittyä maapalloon, sen aktivoitumiseen, ns. kohoamiseen ja sitä kautta myös eliöstön ja ihmisen elävöitymiseen.

        Älkää olko niin hyökkääviä ja epätoivoisia.


      • Anonyymi
        Anonyymi kirjoitti:

        Onko koppakuoriaisessakin tuo alkumeri ja sama suhteellinen vesimäärä kuin ihmisessä? Kertokaa, kun olette niin viisaita.

        En ole kreationisti, mutta sanonta vedestä ja hengestä voisi pitää paikkansa, siten, että henki, joka tarkoittaisi jonkinlaista sähköistä toimintaa yhdistettynä veteen, joka elävöittää ruumiin, voisi liittyä maapalloon, sen aktivoitumiseen, ns. kohoamiseen ja sitä kautta myös eliöstön ja ihmisen elävöitymiseen.

        Älkää olko niin hyökkääviä ja epätoivoisia.

        Kaikissa eläinsoluissa on erilaisia suoloja. Henkiä ei ole löytynyt kuin hörhöjen mielikuvituksesta.


      • Anonyymi
        Anonyymi kirjoitti:

        Kaikissa eläinsoluissa on erilaisia suoloja. Henkiä ei ole löytynyt kuin hörhöjen mielikuvituksesta.

        Et osannut vastata. Sen sijaan piti vielä kumota sähköinen toiminta. Niinpä niin...


      • Anonyymi
        Anonyymi kirjoitti:

        Onko koppakuoriaisessakin tuo alkumeri ja sama suhteellinen vesimäärä kuin ihmisessä? Kertokaa, kun olette niin viisaita.

        En ole kreationisti, mutta sanonta vedestä ja hengestä voisi pitää paikkansa, siten, että henki, joka tarkoittaisi jonkinlaista sähköistä toimintaa yhdistettynä veteen, joka elävöittää ruumiin, voisi liittyä maapalloon, sen aktivoitumiseen, ns. kohoamiseen ja sitä kautta myös eliöstön ja ihmisen elävöitymiseen.

        Älkää olko niin hyökkääviä ja epätoivoisia.

        On epätoivoista käydä keskustelua kreationismin kysymyksistä, koska vadtapuoli ei uskalla keskustella kreationismin perusteista eli raamatun kirjainellisesta tulkinnasta. Ehkä pelkäävät totuutta.


      • Anonyymi
        Anonyymi kirjoitti:

        On epätoivoista käydä keskustelua kreationismin kysymyksistä, koska vadtapuoli ei uskalla keskustella kreationismin perusteista eli raamatun kirjainellisesta tulkinnasta. Ehkä pelkäävät totuutta.

        Keakustellaan vaan raamatun kirjaimellisesta tulkinnasta, aloita uusi keskustelu!
        Tämä on jo niin pitkä (ja uusi aihe), niin on hankala seurata.


      • Anonyymi
        Anonyymi kirjoitti:

        Keakustellaan vaan raamatun kirjaimellisesta tulkinnasta, aloita uusi keskustelu!
        Tämä on jo niin pitkä (ja uusi aihe), niin on hankala seurata.

        Miten koppakuoriaisen solunesteet liittyy vedenpaisumuksen vesiin?


    • Anonyymi

      Aloitukseen on vastattu monta kertaa, ja jopa useampia selityksiä on annettu. Lopputulos: Ei ollut mikään mahdottomuus peittää vedellä mantereet, sen sijaan on täysi mahdottomuus saada ateistit ymmärtämään se.

      • Anonyymi

        Lopputuloksena on koottu liuta keskenään ristiriitaisia mielikuvitustarinoita.

        Kun johonkin tarinaa yrittää saada tarkennuksia, jotka helposti osoittaa tarinan järjettömyyden jo alakoululaiselle, niin kretu alkaa venkoilla eikä kykene vastaamaan helppoon kyllä/ei kysymykseen. Sen sijaan alkaa normaali kieroilu ja suu vaahdossa evoluution hokeminen.

        Tämä testi alkaa selvästi lähentyä sulkemisaikaa.


      • Anonyymi

        No kerro nyt vielä, miten (edes) ongelmaton mahdollisuus peittää mantereet hypoteettisesti vedellä TODISTAA vedenpaisumuksesta v 2 500 eaa?
        Ihan kaikesta voi rakentaa hypoteeseja, ne eivät vielä itsessään todista mitään, vai etkö ole perehtynyt tieteen metodeihin lainkaan.
        Ihan oikeasti?


      • Anonyymi
        Anonyymi kirjoitti:

        Lopputuloksena on koottu liuta keskenään ristiriitaisia mielikuvitustarinoita.

        Kun johonkin tarinaa yrittää saada tarkennuksia, jotka helposti osoittaa tarinan järjettömyyden jo alakoululaiselle, niin kretu alkaa venkoilla eikä kykene vastaamaan helppoon kyllä/ei kysymykseen. Sen sijaan alkaa normaali kieroilu ja suu vaahdossa evoluution hokeminen.

        Tämä testi alkaa selvästi lähentyä sulkemisaikaa.

        Minun puolestani voisit kyllä vastata ensin mikä sinun teorioissasi on järkevää! Vai eikö sinulla ole mitään velvollisuutta esittää jotain järkevää?

        Ethän sinä ole edes kiinnostunut vedenpaisumuksen mahdollisuudesta, joten voimme aloittaa sinua kiinnostava keskustelu.


      • Anonyymi
        Anonyymi kirjoitti:

        Minun puolestani voisit kyllä vastata ensin mikä sinun teorioissasi on järkevää! Vai eikö sinulla ole mitään velvollisuutta esittää jotain järkevää?

        Ethän sinä ole edes kiinnostunut vedenpaisumuksen mahdollisuudesta, joten voimme aloittaa sinua kiinnostava keskustelu.

        Kun tieteessä esitetään jotain uutta, niin jopa ennen kuin se saa edes hypoteesin statusta, ajatus alistetaan testiin. Ymmärtääkseni kreationistit olisivat tyrkyttämässä tulvatarua luonnonhistoriaan, joten tässä pienimuotoisesti katsellaan, mitä se todellisuudessa tarkoittaa.

        Nyt siis tämän ketjun perspektiivi on sellaisen vesimäärän ilmestyminen maapallolle, että korkeimmat vuoretkin peittyi. Siis Raamatun kirkainellista tulkintaa. Ja tähän olemme koonneet kreationistien esittämiä vaihtoehtoja. Ja sitten muutama onkin tarjonnut uusia villejä selityksiä, jotka ovat kyllä todettu mahdolliseksi vain mielikuvituksessa. Ne ovat myös Raamatun vastaisia ilmiselvästi.

        ps. Minulla ei ole mitään ehdotelmaa (maalikon sanoin teoriaa) tulvatarusta, koska se ei ole todellisuudessa mahdollinen.


      • Anonyymi

        Luin "vastaukset" läpi ja missään ei ole vastattu suoraan siihen mistä vedet tulivat ja mihin menevät. Eniten jankutettu vastaus on, että maan pinta jotenkin mystisesti aaltoili 6km korkeina aaltoina ja tämä aiheutti megatsunameja, jotka eivät kuitenkaan peittäneet ehkä kaikkein korkeimpia vuoria tai sitten peittivät tai ehkä ne eivät olleetkaan 6 km korkeita vaan millimetrin.

        Ja edelleen puuttuu kaikki HAVAINNOT siitä, että mitä globaaleja jälkiä moinen globaali veden maan päälle vyöryminen ja pois valuminen jätti?


      • Anonyymi
        Anonyymi kirjoitti:

        Luin "vastaukset" läpi ja missään ei ole vastattu suoraan siihen mistä vedet tulivat ja mihin menevät. Eniten jankutettu vastaus on, että maan pinta jotenkin mystisesti aaltoili 6km korkeina aaltoina ja tämä aiheutti megatsunameja, jotka eivät kuitenkaan peittäneet ehkä kaikkein korkeimpia vuoria tai sitten peittivät tai ehkä ne eivät olleetkaan 6 km korkeita vaan millimetrin.

        Ja edelleen puuttuu kaikki HAVAINNOT siitä, että mitä globaaleja jälkiä moinen globaali veden maan päälle vyöryminen ja pois valuminen jätti?

        No eksä nyt tiärä;
        Grand Canyon (vaikka se onkin uurtunut hitaasti miljoonien vuosien ajan mutta on se "silti" todiste!!) ja tepuisvuoret, joiden vuorenhuippujen välillä oleva kiviaines, tuota, huuhtoutui kai mereen?
        Tarviiko muuta todistusaineistoa?!
        Kyl me voidaan keksiä eiku löytää lisääkin jos sää vaa haluut ku me mietitään ihan ite!


      • Anonyymi
        Anonyymi kirjoitti:

        Luin "vastaukset" läpi ja missään ei ole vastattu suoraan siihen mistä vedet tulivat ja mihin menevät. Eniten jankutettu vastaus on, että maan pinta jotenkin mystisesti aaltoili 6km korkeina aaltoina ja tämä aiheutti megatsunameja, jotka eivät kuitenkaan peittäneet ehkä kaikkein korkeimpia vuoria tai sitten peittivät tai ehkä ne eivät olleetkaan 6 km korkeita vaan millimetrin.

        Ja edelleen puuttuu kaikki HAVAINNOT siitä, että mitä globaaleja jälkiä moinen globaali veden maan päälle vyöryminen ja pois valuminen jätti?

        Paitsi että vedenpaisumus selittää mustanmullan maiden valtavat maakerrokset, öljyn muodostumisen, kivihiilen muodostumisen, maan katoamisen Suomen Savon mäkien päältä, samalla kun kivet eivät liikkuneet minnekään, fossiililöydöt, merenalaiset kaupungit, toisten kaupunkien peittymisen vuorten huipulla, Tepuis-vuoret, jopa savi- ja sorakerrokset, Kanjonit, harjut, jne.

        Valitettavasti tämä vaatii hiukan muutakin kuin järjen puutetta.


      • Anonyymi
        Anonyymi kirjoitti:

        Paitsi että vedenpaisumus selittää mustanmullan maiden valtavat maakerrokset, öljyn muodostumisen, kivihiilen muodostumisen, maan katoamisen Suomen Savon mäkien päältä, samalla kun kivet eivät liikkuneet minnekään, fossiililöydöt, merenalaiset kaupungit, toisten kaupunkien peittymisen vuorten huipulla, Tepuis-vuoret, jopa savi- ja sorakerrokset, Kanjonit, harjut, jne.

        Valitettavasti tämä vaatii hiukan muutakin kuin järjen puutetta.

        Miksi tuputtaa samat potaskat, jotka on ammuttu alas ties kuinka monta kertaa?

        Mutta yritämme ymmärtää seuraavaa tässä ketjussa. Raamatun mukaan korkeimmat vuoret peittyi tulvassa, niin mistä helevatusta se tolkuton vesimäärä ilmestyi ja mihin se hävisi? Korkeimmat vuoret on kuitenkin nykyisestä meren pinnasta mitattuna yli 8km korkeudessa.


      • Anonyymi
        Anonyymi kirjoitti:

        Paitsi että vedenpaisumus selittää mustanmullan maiden valtavat maakerrokset, öljyn muodostumisen, kivihiilen muodostumisen, maan katoamisen Suomen Savon mäkien päältä, samalla kun kivet eivät liikkuneet minnekään, fossiililöydöt, merenalaiset kaupungit, toisten kaupunkien peittymisen vuorten huipulla, Tepuis-vuoret, jopa savi- ja sorakerrokset, Kanjonit, harjut, jne.

        Valitettavasti tämä vaatii hiukan muutakin kuin järjen puutetta.

        kerropa tarkemmin, miten vedenpaisumus tarkalleen v silloin ja silloin "selittää" fossiilit


      • Anonyymi
        Anonyymi kirjoitti:

        Paitsi että vedenpaisumus selittää mustanmullan maiden valtavat maakerrokset, öljyn muodostumisen, kivihiilen muodostumisen, maan katoamisen Suomen Savon mäkien päältä, samalla kun kivet eivät liikkuneet minnekään, fossiililöydöt, merenalaiset kaupungit, toisten kaupunkien peittymisen vuorten huipulla, Tepuis-vuoret, jopa savi- ja sorakerrokset, Kanjonit, harjut, jne.

        Valitettavasti tämä vaatii hiukan muutakin kuin järjen puutetta.

        "Mustanmullan syntymisen edellytys on lämmin mannerilmasto, puuton arokasvillisuus ja kuohkea, kalkkipitoinen maalaji."lähde wiki

        Eli vedepaisumus aiheutti mitä?! Lämpimän mannerilmaston? Entä se (700 v kestänyt) jääkausi?


      • Anonyymi
        Anonyymi kirjoitti:

        Paitsi että vedenpaisumus selittää mustanmullan maiden valtavat maakerrokset, öljyn muodostumisen, kivihiilen muodostumisen, maan katoamisen Suomen Savon mäkien päältä, samalla kun kivet eivät liikkuneet minnekään, fossiililöydöt, merenalaiset kaupungit, toisten kaupunkien peittymisen vuorten huipulla, Tepuis-vuoret, jopa savi- ja sorakerrokset, Kanjonit, harjut, jne.

        Valitettavasti tämä vaatii hiukan muutakin kuin järjen puutetta.

        Kun Nooa tiivisti arkkinsa maaöljyllä, niin miten se muka syntyi vasta vedenpaisumuksessa?


      • Anonyymi
        Anonyymi kirjoitti:

        Luin "vastaukset" läpi ja missään ei ole vastattu suoraan siihen mistä vedet tulivat ja mihin menevät. Eniten jankutettu vastaus on, että maan pinta jotenkin mystisesti aaltoili 6km korkeina aaltoina ja tämä aiheutti megatsunameja, jotka eivät kuitenkaan peittäneet ehkä kaikkein korkeimpia vuoria tai sitten peittivät tai ehkä ne eivät olleetkaan 6 km korkeita vaan millimetrin.

        Ja edelleen puuttuu kaikki HAVAINNOT siitä, että mitä globaaleja jälkiä moinen globaali veden maan päälle vyöryminen ja pois valuminen jätti?

        Nuo maanpinnan kilometrien korkuiset "aallot" kyllä aiheuttaisi sellaiset maanjäristykset ja tulivuoret, että niiden kaasut ois tappanut jo muutenkin tukehtumassa olevan arkin lastin. Ja viimeistään tuota seuraisi ydintalvi vuosikymneniksi, mikä ois tappanut ainakin nisäkkäät. Nämä kaikki näkyisi ympäri maapalloa nyt vaan eipä näy.


      • Anonyymi
        Anonyymi kirjoitti:

        Paitsi että vedenpaisumus selittää mustanmullan maiden valtavat maakerrokset, öljyn muodostumisen, kivihiilen muodostumisen, maan katoamisen Suomen Savon mäkien päältä, samalla kun kivet eivät liikkuneet minnekään, fossiililöydöt, merenalaiset kaupungit, toisten kaupunkien peittymisen vuorten huipulla, Tepuis-vuoret, jopa savi- ja sorakerrokset, Kanjonit, harjut, jne.

        Valitettavasti tämä vaatii hiukan muutakin kuin järjen puutetta.

        No nimenomaan järjen puutetta (ja sokeaa, naiivia uskoa taruihin)...

        Sorat ja harjut selittyvät kyllä jääkausilla, joskus 1800-luvun alkupuolella oli vielä valloilla käsitys että Raamatun, tai joku muu, vedenpaisumus selittää monet jäljet luonnossa mutta (nimenomaan paikan päällä,) etenevät tutkimukset osoittivat että se ei ole vastaus. Eräs (muistaakseni) William Buckland - niminen tutkija oli hartaasti vedenpaisumuksen kannalla, mutta kun useat paikan päällä tehdyt tutkimukset maaperästä, siirtolohkareista ym etenivät hänkin siirtyi innokkaasti kannattamaan jääkausiteoriaa.

        Vasta muistaakseni 1960-luvulla tehdyt (voi olla jo aikaisemmin) merten alaiset sedimenttikerrosten kairaamiset antoivat geologeille tiedon milloin jääkausi oli peittänyt lähes kolmanneksen maapallosta. Se valisti myös että pitkien glasiaali- eli jäätymisvaiheiden välissä on ollut lyhyempiä lämpimiä kausia. Sedimenttikerroksissa, jotka olivat kauniissa virheettömässä järjestyksessä, löytyi trooppisten eläinten ja kasvien jäänteitä esim Britanniassa asti, mikä on aika vastaansanomaton todiste interglasiaalista.

        Myös napamantereiden kairaukset kertoivat samaa tarinaa jääkausista. Vedenpaisumuksesta ei löytynyt jälkiä, mikä on hyvin outoa mikäli sellainen olisi oikeasti tapahtunut, muutenkin kuin kreationistien mielikuvituksessa.


      • Anonyymi
        Anonyymi kirjoitti:

        No eksä nyt tiärä;
        Grand Canyon (vaikka se onkin uurtunut hitaasti miljoonien vuosien ajan mutta on se "silti" todiste!!) ja tepuisvuoret, joiden vuorenhuippujen välillä oleva kiviaines, tuota, huuhtoutui kai mereen?
        Tarviiko muuta todistusaineistoa?!
        Kyl me voidaan keksiä eiku löytää lisääkin jos sää vaa haluut ku me mietitään ihan ite!

        Jeps. Nämä miettii ja keksii ihan itte. Se on kreationistinen tutkimustapa, missä vain mielikuvitus on rajana, jopa Raamattua voi "täydentää" omilla jutuillaan.


      • Anonyymi
        Anonyymi kirjoitti:

        No nimenomaan järjen puutetta (ja sokeaa, naiivia uskoa taruihin)...

        Sorat ja harjut selittyvät kyllä jääkausilla, joskus 1800-luvun alkupuolella oli vielä valloilla käsitys että Raamatun, tai joku muu, vedenpaisumus selittää monet jäljet luonnossa mutta (nimenomaan paikan päällä,) etenevät tutkimukset osoittivat että se ei ole vastaus. Eräs (muistaakseni) William Buckland - niminen tutkija oli hartaasti vedenpaisumuksen kannalla, mutta kun useat paikan päällä tehdyt tutkimukset maaperästä, siirtolohkareista ym etenivät hänkin siirtyi innokkaasti kannattamaan jääkausiteoriaa.

        Vasta muistaakseni 1960-luvulla tehdyt (voi olla jo aikaisemmin) merten alaiset sedimenttikerrosten kairaamiset antoivat geologeille tiedon milloin jääkausi oli peittänyt lähes kolmanneksen maapallosta. Se valisti myös että pitkien glasiaali- eli jäätymisvaiheiden välissä on ollut lyhyempiä lämpimiä kausia. Sedimenttikerroksissa, jotka olivat kauniissa virheettömässä järjestyksessä, löytyi trooppisten eläinten ja kasvien jäänteitä esim Britanniassa asti, mikä on aika vastaansanomaton todiste interglasiaalista.

        Myös napamantereiden kairaukset kertoivat samaa tarinaa jääkausista. Vedenpaisumuksesta ei löytynyt jälkiä, mikä on hyvin outoa mikäli sellainen olisi oikeasti tapahtunut, muutenkin kuin kreationistien mielikuvituksessa.

        Jääkausiteoria syntyi jo ennen Darwinin Lajien syntyä. Samoin jo ennen Darwinia vedenpaisumusta pidettiin geologien piirissä taruna, ja he pitivät maapalloa sadan tai satojen miljoonien vuosien ikäisenä. Kaiken lisäksi Jo ennen Darwinin syntymää kuollut Carl von Linne puhui sujuvasti "sukulaislajeista", vaikka ei vielä ymmärtänyt, mistä elonkehän hierarkinen taksonomia johtui.

        Kreationistien väite, että kyse olisi jostain Darwinin teorian tueksi tehdystä salaliitosta eivät täsmää todellisuuteen.


      • Anonyymi
        Anonyymi kirjoitti:

        Jääkausiteoria syntyi jo ennen Darwinin Lajien syntyä. Samoin jo ennen Darwinia vedenpaisumusta pidettiin geologien piirissä taruna, ja he pitivät maapalloa sadan tai satojen miljoonien vuosien ikäisenä. Kaiken lisäksi Jo ennen Darwinin syntymää kuollut Carl von Linne puhui sujuvasti "sukulaislajeista", vaikka ei vielä ymmärtänyt, mistä elonkehän hierarkinen taksonomia johtui.

        Kreationistien väite, että kyse olisi jostain Darwinin teorian tueksi tehdystä salaliitosta eivät täsmää todellisuuteen.

        Aivan, tutkijoiden piirissä pohdittiin maaperään ym liittyviä juttuja ennemmin jo 1700-luvun lopulla vaikka varsinaisena tieteenalana geologiaa ei vielä ollut. Se kai sai enemmän tai vähemmän alkunsa juuri koko 1800-luvun ajan kestäneestä jääkausi keskustelusta. Tai näin olen ymmärtänyt?


      • Anonyymi
        Anonyymi kirjoitti:

        No nimenomaan järjen puutetta (ja sokeaa, naiivia uskoa taruihin)...

        Sorat ja harjut selittyvät kyllä jääkausilla, joskus 1800-luvun alkupuolella oli vielä valloilla käsitys että Raamatun, tai joku muu, vedenpaisumus selittää monet jäljet luonnossa mutta (nimenomaan paikan päällä,) etenevät tutkimukset osoittivat että se ei ole vastaus. Eräs (muistaakseni) William Buckland - niminen tutkija oli hartaasti vedenpaisumuksen kannalla, mutta kun useat paikan päällä tehdyt tutkimukset maaperästä, siirtolohkareista ym etenivät hänkin siirtyi innokkaasti kannattamaan jääkausiteoriaa.

        Vasta muistaakseni 1960-luvulla tehdyt (voi olla jo aikaisemmin) merten alaiset sedimenttikerrosten kairaamiset antoivat geologeille tiedon milloin jääkausi oli peittänyt lähes kolmanneksen maapallosta. Se valisti myös että pitkien glasiaali- eli jäätymisvaiheiden välissä on ollut lyhyempiä lämpimiä kausia. Sedimenttikerroksissa, jotka olivat kauniissa virheettömässä järjestyksessä, löytyi trooppisten eläinten ja kasvien jäänteitä esim Britanniassa asti, mikä on aika vastaansanomaton todiste interglasiaalista.

        Myös napamantereiden kairaukset kertoivat samaa tarinaa jääkausista. Vedenpaisumuksesta ei löytynyt jälkiä, mikä on hyvin outoa mikäli sellainen olisi oikeasti tapahtunut, muutenkin kuin kreationistien mielikuvituksessa.

        Tietysti löytyy trooppisten eläinten ja kasvien jätteitä Britanniasta, onhan dinosaurusten fossiileja löytynyt etelämantereeltakin. Se on todistus maailmasta ennen vedenpaisumusta.

        Voit kyllä kairailla vedenpaisumusta vaikka kuinka, ei vedestä saa mitään kaivaustulosta, ainoastaan liejusta, jonka taas voi tulkita kuinka haluaa.

        Kyllä maalaisjärkikin sanoo, että jos jäätiköt olisivat liukuneet Savon mäkien yli, niin kyllä kivikot olisivat siirtyneet laaksoihin ja mäkien päällä olisi vain kalliot ja hiukan ruokamultaa päällä.

        Ja miten ne harjut sitten syntyivät? Oliko hiekka korkealla jäätikössä, ja valui alas samalle kohden jäätikön sulaessa, muodostaen sadan metrin harjun? Jos jää suli, kai vesi virtasi alaspäin, ja olisi tietysti tehnyt uoman jäätikön pohjan tasolle, eikä kasannut vuorta eteen.

        Miksi vesi ei virrannut alhaisimman kohdan kautta, miksi vesi lastasi hiekkaa koholle?
        Ei käy järkeen, eikä sitä voi käytännössä todistaa.

        Riittää kun vain uskoo sokeasti!


      • Anonyymi
        Anonyymi kirjoitti:

        Jeps. Nämä miettii ja keksii ihan itte. Se on kreationistinen tutkimustapa, missä vain mielikuvitus on rajana, jopa Raamattua voi "täydentää" omilla jutuillaan.

        Evoluutionistisen tutkimustavan mukaan taas ei saa missään tapauksessa tukea vedenpaisumusteoriaa.


      • Anonyymi
        Anonyymi kirjoitti:

        Evoluutionistisen tutkimustavan mukaan taas ei saa missään tapauksessa tukea vedenpaisumusteoriaa.

        Tieteen tulokset on JOHTOPÄÄTÖKSIÄ - ei ennalta päätettyjen uskomusten todistelua. Toiseksi tiede perustuu HAVAINTOIHIN ja niistä tehtyihin LOOGISIIN.

        Vedenpaisumusukomusta ei tue havainnot eikä logiikka. Vedenpaisumususkomusta ei pidetä naurettavana ja mahdottomana sen takia että olisi joku suuri uskosi vastainen salaliitto, vaan sen takia, että se on naurettava ja mahdoton.


      • Anonyymi
        Anonyymi kirjoitti:

        Tietysti löytyy trooppisten eläinten ja kasvien jätteitä Britanniasta, onhan dinosaurusten fossiileja löytynyt etelämantereeltakin. Se on todistus maailmasta ennen vedenpaisumusta.

        Voit kyllä kairailla vedenpaisumusta vaikka kuinka, ei vedestä saa mitään kaivaustulosta, ainoastaan liejusta, jonka taas voi tulkita kuinka haluaa.

        Kyllä maalaisjärkikin sanoo, että jos jäätiköt olisivat liukuneet Savon mäkien yli, niin kyllä kivikot olisivat siirtyneet laaksoihin ja mäkien päällä olisi vain kalliot ja hiukan ruokamultaa päällä.

        Ja miten ne harjut sitten syntyivät? Oliko hiekka korkealla jäätikössä, ja valui alas samalle kohden jäätikön sulaessa, muodostaen sadan metrin harjun? Jos jää suli, kai vesi virtasi alaspäin, ja olisi tietysti tehnyt uoman jäätikön pohjan tasolle, eikä kasannut vuorta eteen.

        Miksi vesi ei virrannut alhaisimman kohdan kautta, miksi vesi lastasi hiekkaa koholle?
        Ei käy järkeen, eikä sitä voi käytännössä todistaa.

        Riittää kun vain uskoo sokeasti!

        "Tietysti löytyy trooppisten eläinten ja kasvien jätteitä Britanniasta, onhan dinosaurusten fossiileja löytynyt etelämantereeltakin. Se on todistus maailmasta ennen vedenpaisumusta."

        Nuo kertovat siitä, että ilmasto noissa paikoissa on muuttunut. Se voi johtua joko suoraan ilmaston muuttumisesta tai siitä, että litosfäärilaatat, joilla nuo alueet ovat ovat aikojen kuluessa liikkuneet ja olleet ennen eri leveysasteilla.

        Ongelma kreationistien ja muiden kummallisiin avaruusolio- tms uskovien ja tieteentekijöiden välillä on se, että kun vaikka Britanniasta löytyy trooppisten kasvien jäänteitä, niin tieteentekijä kysyy:
        "Mistä tämä löytöö ja mitä voimme siitä päätellä yhdessä muiden löytöjen kanssa?"
        Kun taas yhteen salaliittoteoriaan hurahtanut kysyy:
        "Tukeehan tämä löytö minun uskomista, tukeehan?"

        "Ja miten ne harjut sitten syntyivät? ..."

        Matkusta islantiin. Siellä (pienet) jäätiköt vetäytyvät ja niiden jäljiltä jää soraharjuja. Ei tarvitse askarrella, paskarrella oman pään sisällä kun voi käydä katsomassa.


      • Anonyymi
        Anonyymi kirjoitti:

        Tietysti löytyy trooppisten eläinten ja kasvien jätteitä Britanniasta, onhan dinosaurusten fossiileja löytynyt etelämantereeltakin. Se on todistus maailmasta ennen vedenpaisumusta.

        Voit kyllä kairailla vedenpaisumusta vaikka kuinka, ei vedestä saa mitään kaivaustulosta, ainoastaan liejusta, jonka taas voi tulkita kuinka haluaa.

        Kyllä maalaisjärkikin sanoo, että jos jäätiköt olisivat liukuneet Savon mäkien yli, niin kyllä kivikot olisivat siirtyneet laaksoihin ja mäkien päällä olisi vain kalliot ja hiukan ruokamultaa päällä.

        Ja miten ne harjut sitten syntyivät? Oliko hiekka korkealla jäätikössä, ja valui alas samalle kohden jäätikön sulaessa, muodostaen sadan metrin harjun? Jos jää suli, kai vesi virtasi alaspäin, ja olisi tietysti tehnyt uoman jäätikön pohjan tasolle, eikä kasannut vuorta eteen.

        Miksi vesi ei virrannut alhaisimman kohdan kautta, miksi vesi lastasi hiekkaa koholle?
        Ei käy järkeen, eikä sitä voi käytännössä todistaa.

        Riittää kun vain uskoo sokeasti!

        Vaikka postaus olikin denialistin peruslistaus, niin nostetaan oari seikkaa.

        Tulvatarun perustelu fossiileilla on kuin ampuisi itseään jalkaan.

        Jännä että soraharjuja on vain siellä missä jääkausi vaikutti (muut luonnossa okevat jäljet). Tulvahan tarun mukaan oli globaali.

        Savon kivikot on jo käsitelty. Ihan vastaavia "mäkien päällä" olevia kivikoita voit havaita eri jäätiköiden reunoilla. Ei jatkoon tuo jankkaus.


        Et kuitenkaan pysty vastaamaan, että mikä on tulvan vesien mysteeri vaan yrität jankuttaa ihan suvuseikkoba. Eihän tulvaa voi olla jos jostain ei tullut pirusti vettä maapallolle. 8km nykyisen meren pinnan tason päälle?


      • Anonyymi
        Anonyymi kirjoitti:

        Tietysti löytyy trooppisten eläinten ja kasvien jätteitä Britanniasta, onhan dinosaurusten fossiileja löytynyt etelämantereeltakin. Se on todistus maailmasta ennen vedenpaisumusta.

        Voit kyllä kairailla vedenpaisumusta vaikka kuinka, ei vedestä saa mitään kaivaustulosta, ainoastaan liejusta, jonka taas voi tulkita kuinka haluaa.

        Kyllä maalaisjärkikin sanoo, että jos jäätiköt olisivat liukuneet Savon mäkien yli, niin kyllä kivikot olisivat siirtyneet laaksoihin ja mäkien päällä olisi vain kalliot ja hiukan ruokamultaa päällä.

        Ja miten ne harjut sitten syntyivät? Oliko hiekka korkealla jäätikössä, ja valui alas samalle kohden jäätikön sulaessa, muodostaen sadan metrin harjun? Jos jää suli, kai vesi virtasi alaspäin, ja olisi tietysti tehnyt uoman jäätikön pohjan tasolle, eikä kasannut vuorta eteen.

        Miksi vesi ei virrannut alhaisimman kohdan kautta, miksi vesi lastasi hiekkaa koholle?
        Ei käy järkeen, eikä sitä voi käytännössä todistaa.

        Riittää kun vain uskoo sokeasti!

        "Miksi vesi ei virrannut alhaisimman kohdan kautta, miksi vesi lastasi hiekkaa koholle?"

        Ja kreationisti yrittää todistella vedenpaisumusta soraharjuilla joita vesi ei ole voinut synnyttää?!?

        Tämä on melkein yhtä hyvä tarina kuin se, että maaöljy jota Nooa käytti Raamatun mukaan arkkinsa tiivistämiseen on syntynyt vedenpaisumuksen jälkeen puista jotka vedenpaisumus jätti maakerroksen alle. Jep jep. Jos hylätään kaikki muukin logiikka niin miksi aikakin ei voisi välillä kulkea myös taaksepäin?


      • Anonyymi
        Anonyymi kirjoitti:

        Tietysti löytyy trooppisten eläinten ja kasvien jätteitä Britanniasta, onhan dinosaurusten fossiileja löytynyt etelämantereeltakin. Se on todistus maailmasta ennen vedenpaisumusta.

        Voit kyllä kairailla vedenpaisumusta vaikka kuinka, ei vedestä saa mitään kaivaustulosta, ainoastaan liejusta, jonka taas voi tulkita kuinka haluaa.

        Kyllä maalaisjärkikin sanoo, että jos jäätiköt olisivat liukuneet Savon mäkien yli, niin kyllä kivikot olisivat siirtyneet laaksoihin ja mäkien päällä olisi vain kalliot ja hiukan ruokamultaa päällä.

        Ja miten ne harjut sitten syntyivät? Oliko hiekka korkealla jäätikössä, ja valui alas samalle kohden jäätikön sulaessa, muodostaen sadan metrin harjun? Jos jää suli, kai vesi virtasi alaspäin, ja olisi tietysti tehnyt uoman jäätikön pohjan tasolle, eikä kasannut vuorta eteen.

        Miksi vesi ei virrannut alhaisimman kohdan kautta, miksi vesi lastasi hiekkaa koholle?
        Ei käy järkeen, eikä sitä voi käytännössä todistaa.

        Riittää kun vain uskoo sokeasti!

        "Kyllä maalaisjärkikin sanoo, että jos jäätiköt olisivat liukuneet Savon mäkien yli, niin kyllä kivikot olisivat siirtyneet laaksoihin ja mäkien päällä olisi vain kalliot ja hiukan ruokamultaa päällä."

        Eli nyt sitä vedenpaisumuksen jälkeistä pikku jääkautta ei ollutkaan...?
        Jos sinua kiinnostaa hiekkaharjujen synty niin googlaamalla Salpausselät saat jo jotain tietoa. Salpausselän synnystä on lisää tietoa useissa jääkausia käsittelevissä teoksissa; eikun kirjastoon!
        Alunperin, esim 1800-luvulla moreeniharjuja tutkittiin Alppien jäätiköiden reunamilla ja pohdittiin niiden syntyä koska harjuja löytyi myös etäänpänä kuin jäätikön reuna.


      • Anonyymi
        Anonyymi kirjoitti:

        Evoluutionistisen tutkimustavan mukaan taas ei saa missään tapauksessa tukea vedenpaisumusteoriaa.

        Kyse ei ole siitä ettei se "saa" tukea vedenpaisumusta, se vaihtoehto on vaan havaittu paikkansapitämättömäksi yli 200 vuotta sitten.

        Mutta, saahan yect siihen uskoa, ei siinä mitään. Kyllä siihenkin vaihtoehtoon on aikanaan suhtauduttu asianmukaisella vakavuudella, vaikka ette taida uskoa että niin on. Tai oikein tajuta, että vedenpaisumus on skenaariona hylätty.


      • Anonyymi
        Anonyymi kirjoitti:

        Kyse ei ole siitä ettei se "saa" tukea vedenpaisumusta, se vaihtoehto on vaan havaittu paikkansapitämättömäksi yli 200 vuotta sitten.

        Mutta, saahan yect siihen uskoa, ei siinä mitään. Kyllä siihenkin vaihtoehtoon on aikanaan suhtauduttu asianmukaisella vakavuudella, vaikka ette taida uskoa että niin on. Tai oikein tajuta, että vedenpaisumus on skenaariona hylätty.

        PS
        ja geologia ei ole evoluutiotutkimusta, vaikka te kreat laitatte kovin mielellänne sen etiketin kaikkeen mikä ei ole teidän mieleenne ;)


      • Anonyymi
        Anonyymi kirjoitti:

        Tietysti löytyy trooppisten eläinten ja kasvien jätteitä Britanniasta, onhan dinosaurusten fossiileja löytynyt etelämantereeltakin. Se on todistus maailmasta ennen vedenpaisumusta.

        Voit kyllä kairailla vedenpaisumusta vaikka kuinka, ei vedestä saa mitään kaivaustulosta, ainoastaan liejusta, jonka taas voi tulkita kuinka haluaa.

        Kyllä maalaisjärkikin sanoo, että jos jäätiköt olisivat liukuneet Savon mäkien yli, niin kyllä kivikot olisivat siirtyneet laaksoihin ja mäkien päällä olisi vain kalliot ja hiukan ruokamultaa päällä.

        Ja miten ne harjut sitten syntyivät? Oliko hiekka korkealla jäätikössä, ja valui alas samalle kohden jäätikön sulaessa, muodostaen sadan metrin harjun? Jos jää suli, kai vesi virtasi alaspäin, ja olisi tietysti tehnyt uoman jäätikön pohjan tasolle, eikä kasannut vuorta eteen.

        Miksi vesi ei virrannut alhaisimman kohdan kautta, miksi vesi lastasi hiekkaa koholle?
        Ei käy järkeen, eikä sitä voi käytännössä todistaa.

        Riittää kun vain uskoo sokeasti!

        "Voit kyllä kairailla vedenpaisumusta vaikka kuinka, ei vedestä saa mitään kaivaustulosta, ainoastaan liejusta, jonka taas voi tulkita kuinka haluaa."

        Meinaatko, ettei kolmen kilometrin korkuisesta vesimäärästä, en nyt muisa kuinka kauan se vedenpaisumuksenne kesti kaiken kaikkiaan, mutta siis, meinaatko siis ettei massiivisesta hypersupertulvasta jäisi mitään merkkejä maaperään?! Eli sen aikaiseen sedimenttikerrokseen? Miksi ei?
        Liejusta saa selville vaikka mitä, oikeilla laitteilla ja asiatiedoilla; siellä on kasvien jäänteintä ja - siemeniä, fossiileja ym ja maakerroksen ominaislaatu on tutkittavissa, siis mistä se koostuu...Luuletko ihan oikeasti että ihmiset joiden elämäntyönä on jokin tieteenalan tutkiminen, ihan vaan lonkalta heittävät arvauksia vähän sinnepäin tai ei edes sitä ;). Ja että heiltä (kaikilta...hehe) puuttuu totaalisesti se sinun peräänkuuluttamasi maalaisjärki.

        Olet sinä aika ennakkoluuloinen tapaus.


    • Anonyymi

      Ainoa paikka missä vesi on paisunut on uskovaisten päät.

      • Anonyymi

        Ei vesi paisukkaan, paitsi silloin kun se on kiehunut kkoko maankuoren ollessa yhdellä puolella Pangeassa.


      • Anonyymi
        Anonyymi kirjoitti:

        Ei vesi paisukkaan, paitsi silloin kun se on kiehunut kkoko maankuoren ollessa yhdellä puolella Pangeassa.

        te ette opi mitään missään ajassa mutta sanottakoon nyt taas, maankuorta on myös aina merten alla. Got it?


      • Anonyymi
        Anonyymi kirjoitti:

        te ette opi mitään missään ajassa mutta sanottakoon nyt taas, maankuorta on myös aina merten alla. Got it?

        Kreationistin ainoa mahdollisuus pysyä kreationistina on olla ymmärtämättä ja olla oppimatta. Jos pirulle antaa pikkusormen ja alkaa tutustumaan siihen, minkälainen todellisuus todella on, niin usko rapisee.

        Siksi tämäkin kretu tulee toistamaan tuota "Pangean aikaan koko maankuori oli yhdellä puolella maapalloa" -väitettä. Kreationistin on pakko pysyä sivistymättömänä.


      • Anonyymi
        Anonyymi kirjoitti:

        Ei vesi paisukkaan, paitsi silloin kun se on kiehunut kkoko maankuoren ollessa yhdellä puolella Pangeassa.

        Yritä päästä tuosta pakkomielteestäsi eroon. Teet itsestäsi vain pellen toistelemalla tuota itse keksimääsi älyttömyyttä.


    • Anonyymi

      Transuilulla sitä vettä ei saakaan aikaan toiste.

    • Anonyymi

      On moneen kertaan osoitettu, että maapallon vesimäärä riittää peittämään koko maapallon jos sen haluaa ymmärtää. Tietysti voi aina kuvitella että lumimiehet Himalajalla olisivat voineet kiivetä pakoon vuoren huipulle ja siellä syödä lunta kuukauden, kunnes vesi taas laski, mutta on aivan tarpeetonta tunkea maapallolle enemmän vettä, kun nykyinenkin riittää vedenpaisumukseen . Nyt pelätään jo parin metrin vedenpinnan korotusta ! Se kuulemma on jo katastrofi!

      Sen sijaan ei kukaan ole osannut antaa selvitystä siihen miksi mantereiden pintakerrokset olisivat kerran olleet yhdellä puolen maapalloa, kun samalla sanotaan, että mannerlaatat peittivätkin koko maapallon!!! Miten se nyt olikaan??

      Vedenpaisumuksen todisteita on soitettu monta kertaa, mutta on kyse ideologiasta, eikä todisteista. Evoluutionistia ei voi muuksi muuttaa ellei hän ole kiinnostunut tosiasioista!

      • Anonyymi

        Sait jo näpillesi näistä ja yhä urhoollisesti taistelet vastaan ja vahvistat olevasi tosi pelle.

        1) Kukaan ei osoittanut missään ketjussa, miten reaalimaailmassa olisi korkeimmat vuoret peittyneet. Vain sinun harhainen mieli kuvittelee niin.

        2) Sinulle jäi nyt päälle itse keksimäsi harhainen käsitys laattatektoniikasta. Yritä päästä tästä fetissistäsi eroon.

        3) Et ole esittänyt yhtään todistetta, jolle ei olisi geologista hyvin tunnettua ilmiötä - ilman mitään taikatulvaa. Kaikki on vain kreationistien tai jopa sinun omia juttujasi kuten Savon kivikot.

        Mennäänpä vielä tuohon sinun aiempaan gish gallop tyyliin. Et nimittäin vastannut suoraan kysymykseen, että aaltoiliko maankuori tulvatarussa kilometrien korkuisina aaltoina (6000 km / 1000 = 6km). Vastaatko vai oletko luikuri ja luikitko häntä koipien välissä taas karkuun?


      • Anonyymi
        Anonyymi kirjoitti:

        Sait jo näpillesi näistä ja yhä urhoollisesti taistelet vastaan ja vahvistat olevasi tosi pelle.

        1) Kukaan ei osoittanut missään ketjussa, miten reaalimaailmassa olisi korkeimmat vuoret peittyneet. Vain sinun harhainen mieli kuvittelee niin.

        2) Sinulle jäi nyt päälle itse keksimäsi harhainen käsitys laattatektoniikasta. Yritä päästä tästä fetissistäsi eroon.

        3) Et ole esittänyt yhtään todistetta, jolle ei olisi geologista hyvin tunnettua ilmiötä - ilman mitään taikatulvaa. Kaikki on vain kreationistien tai jopa sinun omia juttujasi kuten Savon kivikot.

        Mennäänpä vielä tuohon sinun aiempaan gish gallop tyyliin. Et nimittäin vastannut suoraan kysymykseen, että aaltoiliko maankuori tulvatarussa kilometrien korkuisina aaltoina (6000 km / 1000 = 6km). Vastaatko vai oletko luikuri ja luikitko häntä koipien välissä taas karkuun?

        Etkö tajkua sitä vähää, että aivan mitätön maan kuoren nousu tai lasku olisi hyvin helposti voinut pyyhkiä ihmiset maan päältä.
        Kai tajuat, että 6 kilometria olisi vain yksi promille maapallon halkaisijasta, ja noinkin suuri liike olisi ollut mahdollinen, siis suhteessa 1 mm /1 metri, mitätön ero.

        Se oli vain esimerkki. Taimaassa tarvittiin vain 20 metrin nousu meren pohjassa, ja tuli katastrofi.

        Ei ollut tarvis pestä Mount Everestin rinteiltä ihmisiä pois, kun siellä ei kukaan olisi kuitenkaan selvinnyt useampaa kuukautta sateessa ja nälässä. Nooa kertoi vain mitä pystyi näkemään.

        Tietysti on helpompaa joillekin kuvitella sitä että merenpinnan yläpuolella näkyvät mantereiden pintakerrokset olisivat liukuneet yhdelle puolelle maapalloa toisiaansa tapaamaan, mutta eikö järkikin sano, että myös merien alla on laattoja kiinni toinen toisissaan. Koska on olemassa rajoja eri laattojen välillä, on selvää että laatat pystyvät nousemaan ja laskemaan, eikä sebn vauhdin tarvitse päätä huimata sen enempää kuin nousuvesi. Onhan paljon todisteita, että kalafiossiileja löytyy preerioilta ja kaupunkeja löytyy merwn syvyydestä.

        Tietenkään mikään todistus ei tehoa, mutta evoluutionistienkin uskomuksille voi nauraa vaikka yhdessä harakan kanssa, joka evoluutionistien mukaan on kalan jälkeläinen.


      • Anonyymi
        Anonyymi kirjoitti:

        Etkö tajkua sitä vähää, että aivan mitätön maan kuoren nousu tai lasku olisi hyvin helposti voinut pyyhkiä ihmiset maan päältä.
        Kai tajuat, että 6 kilometria olisi vain yksi promille maapallon halkaisijasta, ja noinkin suuri liike olisi ollut mahdollinen, siis suhteessa 1 mm /1 metri, mitätön ero.

        Se oli vain esimerkki. Taimaassa tarvittiin vain 20 metrin nousu meren pohjassa, ja tuli katastrofi.

        Ei ollut tarvis pestä Mount Everestin rinteiltä ihmisiä pois, kun siellä ei kukaan olisi kuitenkaan selvinnyt useampaa kuukautta sateessa ja nälässä. Nooa kertoi vain mitä pystyi näkemään.

        Tietysti on helpompaa joillekin kuvitella sitä että merenpinnan yläpuolella näkyvät mantereiden pintakerrokset olisivat liukuneet yhdelle puolelle maapalloa toisiaansa tapaamaan, mutta eikö järkikin sano, että myös merien alla on laattoja kiinni toinen toisissaan. Koska on olemassa rajoja eri laattojen välillä, on selvää että laatat pystyvät nousemaan ja laskemaan, eikä sebn vauhdin tarvitse päätä huimata sen enempää kuin nousuvesi. Onhan paljon todisteita, että kalafiossiileja löytyy preerioilta ja kaupunkeja löytyy merwn syvyydestä.

        Tietenkään mikään todistus ei tehoa, mutta evoluutionistienkin uskomuksille voi nauraa vaikka yhdessä harakan kanssa, joka evoluutionistien mukaan on kalan jälkeläinen.

        Kuuden kilometrin maapallon pinnan aaltoilu todellakin tappaisi kaikki ihmiset ja suurimman osan eläimistä. Ihan sama olisiko arkissa tai lentotukialuksella. Tarinan arkkihan tehtiin maalle, jossa hyökyaallot olisivat murskanneet sen tuhannen päreiksi. Tapaninpäivän tsunami on hyvä vertailukohta. Tsunamin nosti 20m merenpohjan muutos pystysuunnassa joka tapahtui suhteellisen pienessä paikassa. Mitä saisi aikaan 300x suurempi pohjan muutos, joka tapahtuu joka puolella.

        Arkki, joka Raamatun mukaan oli puinen 150m x 50m lautta ei olisi selvinnyt edes Suomenlahdella saati valtamerillä. Suurin KOSKAAN rakennettu puulaiva oli kansipituudeltaan 110m ja leveydeltään 15m ja se vuosi jatkuvasti huolimatta että sitä oli yritetty jäykistää 90 raskaalla rautatangolla. Mutta 4000 vuotta aikaisemmin muka tehtiin huomattavasti suurempi? Eikä sen jälkeen koskaan ole pystytty rakentamaan edes lähelle samankokoista puualusta.

        Laattatektoniikka sinulle on väännetty rautalangasta niin monta kertaa, että täydellinen tyhmyytesi asian suhteen täytyy olla tahallista.


      • Anonyymi
        Anonyymi kirjoitti:

        Kuuden kilometrin maapallon pinnan aaltoilu todellakin tappaisi kaikki ihmiset ja suurimman osan eläimistä. Ihan sama olisiko arkissa tai lentotukialuksella. Tarinan arkkihan tehtiin maalle, jossa hyökyaallot olisivat murskanneet sen tuhannen päreiksi. Tapaninpäivän tsunami on hyvä vertailukohta. Tsunamin nosti 20m merenpohjan muutos pystysuunnassa joka tapahtui suhteellisen pienessä paikassa. Mitä saisi aikaan 300x suurempi pohjan muutos, joka tapahtuu joka puolella.

        Arkki, joka Raamatun mukaan oli puinen 150m x 50m lautta ei olisi selvinnyt edes Suomenlahdella saati valtamerillä. Suurin KOSKAAN rakennettu puulaiva oli kansipituudeltaan 110m ja leveydeltään 15m ja se vuosi jatkuvasti huolimatta että sitä oli yritetty jäykistää 90 raskaalla rautatangolla. Mutta 4000 vuotta aikaisemmin muka tehtiin huomattavasti suurempi? Eikä sen jälkeen koskaan ole pystytty rakentamaan edes lähelle samankokoista puualusta.

        Laattatektoniikka sinulle on väännetty rautalangasta niin monta kertaa, että täydellinen tyhmyytesi asian suhteen täytyy olla tahallista.

        Tsunamit on pieni murhe, jos maanpinnalla tapahtuu vuodessa kilometrien pystysuoria siirroksia. Maankuori on silloin niin rikki, että ilma on täynnä laavapurkauksien kaasuja ja tuhkaa. Se tapaisi kaiken elollisen ehkä mikrobeja lukuunottamatta. Myös meren happamoituisi elinkelvottomiksi. Tästä myös seuraisi vuosikymmenien ydintalvi jopa päivän tasaajalla.

        Tämä on vain todellisuus niillä ehdoilla, joita tuo kuvitteellinen maanpinnan muutos tarkoittaisi. Tuossahan siis on kyse kilometreistä, joita noilla promille/millimetri puheilla yritetään häivyttää.

        Se, että alat vääntelemään Raamatun tekstiä sopivammaksi, on positiivinen piirre. Kuvitteellinen Nooa eli joku Lähi-idän tarinankertoja tuskin tiesi Everestiä, muita Himalajan, ja varmasti ei tiennyt Andeja. Ehkä tiesi jonkin lähellä olevan vuoren, jonne pakeni jotain paikallista tulvaa. Aikansa kuluksi keksi tulvataru, joka pitkän suullisen tarinankerronta perinteen kautta päätyi sumerialaisille, joilta se kulkeutui Raamattuun. Käsittääkseni historian tutkijat ovat osoittaneet nuo sumerialaisjuuret tälle tarulle.


    • Anonyymi

      Se että ateistit eivät osaa edes kuvitella mahdollisuutta, että staviisikymmentä metriä pitkä pihkalla päällystetty jykevistä puista tehty alus ei pysyisi veden pinnalla on turhaa höyryilyä.

      Tukkilautat ja niputkin uivat aivan hyvin veden pinnalla, vaikka niitä ei ole edes tervattu.

      Kuvitelma, että arkki tarkoittaisi venettä, on virheellinen, se tarkoittaa laatikkoa.

      Oletetaan nyt vaikka että yksinkertaisuuden vuoksi tervataan muutama tuhat puun runkoa, ja niputetaan ne sitten 150 X25 metrin lautaksi, paksuutta vaikka pari metriä.
      Tuollainen lautta kelluisi vaikka ei edes olisi vesitiivis. Eikä sen tarvitse puskea aaltoja vastaan, vaan ainoastaan kellua seitsemän kuukautta, kunnes juuttuu vuorelle.
      Joten aloittakaa edes arkin suunnittelu sellaiseksi, että kestää, älkääkä tehkö siitä hentoa soutuvenettä aallokkoon.

      Ei vedenpaisumuksen ole tarvinnut merkitä valtavia aaltoja, vaan Raamattu puhuu veden nousemisesta, siis rauhallisesta noususta niinkuin nousuvedessä.

      Jos haluaa kuvitella vedenpaisumuksen mahdolliseksi, kannattaa miettiä muitakin rakennustapoja kuin pikku laivojen rakennusta.

      Nooan kertomus on ainoa joka on silminnäkijän kuvaus päivämäärineen. Toiset ovat myöhempiä kuulopuheisiin perustuvia tarinoita.

      Täällä on osoitettu, ettei maapallolta puutu vettä, se riittää.

      Myös esimerkiksi väestönkasvu osoittaa että jostain syystä ihmisiä ei juuri ollut viisi vuosituhatta sitten, mutta nyt on miljardeja.

      Ja ihmisten rakennelmat kertovat myös vedenpaisumuksesta: sitä varhaisempia rakennuksia ei juuri löydy, paitsi maahan hautautuneina tai vaikka mereen vajonneina.

      Ja Kolme suurta pyramidia kertoo ajasta jolloin osattiin sahata graniiittia ja myös päällystää pyramidit täsmällisillä marmorilaatoilla.

      • Anonyymi

        Tässä korostuu eräs kreationismin piirre, jota kirjoittaja ehkä tahtomattaan korostaa.

        Ensinnäkin ateistit ovat siis kirjoittajan kannalta omaan lahkoon kuulumattomat jopa kristityt, jotka eivät ole fundisuskovia.

        Mutta se piirre - se on kyky kuvitella. Kirjoittaja painottaa kaiken aikaa, että häntä ei rajoita mikään todellisuus vaan ainoastaan oma mielikuvitus. On muiden syy, jos heitä kahlitsee todellisen maailman lainalaisuudet. Kretun pitää vain kieltää tieteen tuottama tietoa ja keksiä omiaan, jopa ohi Raamatun tekstien.

        Sitten voi aina pistää pientä lööperiä. Raamatun nooan taru on sumerialaista perua oleva myytti, ei mikään "silminnäkijäkertomus". Raamatun historiallinen tutkimus on muuten näiden fundisuskovien kannalta ikävä.


      • Anonyymi

        Missään ei ole todistettavasti rakennettu 150 metriä pitkää laivaa / lauttaa pelkästä puusta. Suurin todistetusti aikaansaatu puulaiva on kuunari Wyoming, jonka kansipituus oli 110m ja joka oli jäykistetty 90 jykevällä rautapalkilla, mutta vuosi silti koko ajan ja edellytti jatkuvaa pumppujen käyttöä. Kreationisti väittää että laivanrakennus oli 4500 vuotta sitten pitemmällä kuin 1900-luvulla.

        Jos kyseessä olisi kuvittelukyky, niin kreationistit ovat toki ylivoimaisia kuvittelemaan kaikkea mahdotonta. Mutta jos tutkitaan sitä mitä TIEDÄMME ja MISTÄ ON HAVAINTOJA TAI KOKEMUSTA, niin tilanne on kovin erilainen.

        Raamatun tulvasadussa Nooa tiivistää laivansa maapiellä eli raakaöljypohjaisella tuotteella. Koska kreationisti jäi häpeällisesti kiinni siitä, että sivistymättömänä ei tunne edes palvomansa Raamtun tarinoita, vaan väitti että maaöljy syntyi vasta tulvan jälkeen, niin nyt hän yrittää selvitä nolon häpeällisestä tilanteesta sillä AINOALLA TAVALLA jonka kreationisti osaa eli VALEHTELEMALLA että laiva olisikin muka tiivistetty "pihkalla" toisin kuin Raamatussa kerrotaan. Kreationisti ei näe ongelmaa VALEHDELLA palvomansa RAAMATUN SISÄLLÖSTÄ.

        Tyhmyydelleen kreationisti ei välttämättä mitään voi - niillä korteilla on pelattava, jotka on jaettu. Sivistymättömyys ja oppimattomuuskin voi olla väistämätöntä jos kykyä omaksua tietoa ei kerta kaikkiaan ole (toki se voi olla myös valinta). MUTTA se että kreationisti SILMITTÄMÄTTÖMÄSTI VALEHELEE ja on VALINTA ja osoitus omaksutusta EETTISESTÄ ALA-ARVOISUUDESTA.


      • Anonyymi
        Anonyymi kirjoitti:

        Missään ei ole todistettavasti rakennettu 150 metriä pitkää laivaa / lauttaa pelkästä puusta. Suurin todistetusti aikaansaatu puulaiva on kuunari Wyoming, jonka kansipituus oli 110m ja joka oli jäykistetty 90 jykevällä rautapalkilla, mutta vuosi silti koko ajan ja edellytti jatkuvaa pumppujen käyttöä. Kreationisti väittää että laivanrakennus oli 4500 vuotta sitten pitemmällä kuin 1900-luvulla.

        Jos kyseessä olisi kuvittelukyky, niin kreationistit ovat toki ylivoimaisia kuvittelemaan kaikkea mahdotonta. Mutta jos tutkitaan sitä mitä TIEDÄMME ja MISTÄ ON HAVAINTOJA TAI KOKEMUSTA, niin tilanne on kovin erilainen.

        Raamatun tulvasadussa Nooa tiivistää laivansa maapiellä eli raakaöljypohjaisella tuotteella. Koska kreationisti jäi häpeällisesti kiinni siitä, että sivistymättömänä ei tunne edes palvomansa Raamtun tarinoita, vaan väitti että maaöljy syntyi vasta tulvan jälkeen, niin nyt hän yrittää selvitä nolon häpeällisestä tilanteesta sillä AINOALLA TAVALLA jonka kreationisti osaa eli VALEHTELEMALLA että laiva olisikin muka tiivistetty "pihkalla" toisin kuin Raamatussa kerrotaan. Kreationisti ei näe ongelmaa VALEHDELLA palvomansa RAAMATUN SISÄLLÖSTÄ.

        Tyhmyydelleen kreationisti ei välttämättä mitään voi - niillä korteilla on pelattava, jotka on jaettu. Sivistymättömyys ja oppimattomuuskin voi olla väistämätöntä jos kykyä omaksua tietoa ei kerta kaikkiaan ole (toki se voi olla myös valinta). MUTTA se että kreationisti SILMITTÄMÄTTÖMÄSTI VALEHELEE ja on VALINTA ja osoitus omaksutusta EETTISESTÄ ALA-ARVOISUUDESTA.

        On muuten irvokasta, että kreationistit korostaa, miten vain Raamatun pohjalta voi olla moraalia. Tälläkin palstalla on kretu otettu toistuvasti suorista valheista kiinni.

        Julkisuutta ehkä yksi eniten saanut valhe on tuo Mary Scweizerin tutkimus, jota ilmeisesti kretujen apostolilääkärikin on levitellyt. Valheen levitys on jatkunut vaikka tutkija itse on kieltänyt.

        Kretu ilmeisesti elää niin täysin omassa mielikuvitusmaailmassa, etteivät he edes ymmärrä valehtelevansa.


      • Anonyymi
        Anonyymi kirjoitti:

        On muuten irvokasta, että kreationistit korostaa, miten vain Raamatun pohjalta voi olla moraalia. Tälläkin palstalla on kretu otettu toistuvasti suorista valheista kiinni.

        Julkisuutta ehkä yksi eniten saanut valhe on tuo Mary Scweizerin tutkimus, jota ilmeisesti kretujen apostolilääkärikin on levitellyt. Valheen levitys on jatkunut vaikka tutkija itse on kieltänyt.

        Kretu ilmeisesti elää niin täysin omassa mielikuvitusmaailmassa, etteivät he edes ymmärrä valehtelevansa.

        Kreationistien täydellinen moraalinen kelvottomuus tulee mainiosti esiin siinä, miten he suhtautuvat Raamattuun. He väittävät pitävänsä sitä pyhänä Jumalan sanana, mutta siitä huolimatta he TÄYSIN PIDÄKKEETTÖMÄSTI VALEHTELEVAT siitä, mitä Raamatussa lukee.

        Kun heidän tulkintansa mukaan Pyhän Kirjaan kirjatuista Jumalan lausumista voi vapaasti levittää SAASTAISIA VALHEITA, niin voi vain kuvitella, miten he suhtautuvat asioihin, joita he eivät pidä pyhinä. Iljettävän epärehellistä porukkaa!


      • Anonyymi

        >> Ei vedenpaisumuksen ole tarvinnut merkitä valtavia aaltoja, vaan Raamattu puhuu veden nousemisesta, siis rauhallisesta noususta niinkuin nousuvedessä.

        Näinhän tarinassa kerrotaan. Et taida ollakaan sitä uskonsuuntaa, jolle vedenpaisu mus oli kaikki geologiset muodostelmat kerralla aikaansaanut myllerrys. Nyt sitten selittelemään hetkellisesti moninkertaistunutta vesimäärää. Mistä se tuli ja minne se meni?

        Raamatun taru kertoo syvyyden lähteistä ja taivaan akkunoista. Lisäksi vesi paisui. Niin ikään, tarussa veden laskuun riitti vain se, että tuuli puhalsi ja vesi haihtui pois! Jos otat tuon todesta, sinulla on ongelma. Tosiasiat hyväksyvälle ihmiselle se kertoo muinaisten tarinankertojien alkeellisesta maailmankuvasta. Ainoa mahdollinen yhtymä todellisuuteen on ollut jokin merkittävä paikallinen tulva.


      • Anonyymi

        "Myös esimerkiksi väestönkasvu osoittaa että jostain syystä ihmisiä ei juuri ollut viisi vuosituhatta sitten, mutta nyt on miljardeja."

        sinisilmänen mutta sinnikäs vale. Vuonna 3 000 eaa siis ennen Nooa ja arkkia, on arvioitu olleen 14 miljoonaa ihmistä, vuonna 2 000 eaa, siis Nooan ja "vedenpaisumuksen" jälkeen 27 miljoonaa. Mitään nollaantumista noin v 2500 ei ollut.
        Jos tuo (faktan) ohittaa toteamalla että sen jälkeen väestönkasvu vasta pääsi vauhtiin niin se on selvää totuuden kiertämistä (suomeksi valehtelua) jos yrittää todistaa jotain vedenpaisumuksesta ja ihmisten määrästä. Jos ihmisiä oli mytologisen tulvan aikana about 20 miljoonaa, on aikamoista luikuria sanoa "ettei niitä juuri ollut", tajuat kai itsekin?
        nojoo, luultavasti et

        Teidän raamatussanne ei myöskään mainita jääkautta sanallakaan, mikä on hyvin outoa. Ei myöskään missään muissa kirjoituksissa. Ja jos tulvan loistavana ideana oli tuhota siis tappaa hukuttamalla kaikki ihmiset ja eläimet paitsi Nooan jengiä, siinä sivussa meni kasvitkin, niin mikä funktio jääkaudella oli? Ei jääkausi ole mikään tulvan luonnollinen seuraus. Jos olisi meillä ei muuta olisikaan kuin tulvia ja jääkausia vuorotellen, koska tulvat ilmaantuvat oikeasti jääkausien loputtua


    • Anonyymi

      Ateistit luulevat osaavansa Raamatrtua ja tietävänsä millaista oli ennen vedenpaisumusta. Kuitenkaan he eivät ole edes huomanneet että maa oli tullut täyteen väkivaltaa, siis ihmisiä oli valtavasti. Toisekseen he luulevat, ettei silloin osattu käsitellä metalleja, vaikka ne mainitaan, ja esimerkiksi pyramidien marmori- ja graniittilaatat on sahattu millimetrien tarkkuudella, jolloin ammattitaitoa on ollut, samoin kuin koneita. Ja yhdestä pyramidista on jopa löydetty rautamöykky kivien välistä.
      Siksi ei ole lainkaan ongelmaa käsittää kuinka Nooa pystyi rakentamaan arkin joka kesti kellua veden päällä seitsemän kuukautta.

      Koska emme tiedä, silloisia olosuhteita, meidän on mahdotonta tietää mitä aineita silloin oli puun kyllästämiseen, onhan silloinkin täytynyt osata suojata puita mätänemistä vastaan.
      Ja on turha vertailla laivoja arkkiin, ovathan ne aivan eri asia.

      Raamattu on kirjoitettu käyttäen sanoja jotka ovat saattaneet muuttaa merkitystään. Sanaa "piki" ei nimenomaan mainita, vaan se on tulkinta, sanotaan, että piti suojata sisältä ja päältä, suoja-aineella, alkutekstissä. Emme edes tiedä mitä puulajia sana GOPHER tarkoittaa.

      Tietysti voi aina kuvitella ettei silloin osattu rakentaa, mutta silti pystyttiin rakentamaan pyramideja, joista ainakin yksi on nykyään veden alla Azorin lähistöllä.

      • Anonyymi

        Miksi pitäisi Raamattu tuntea? Uskonnollisia pyhiä kirjoituksia on lukuisia ja ne ovat keskenään ihan yhtä hyviä. Et pysty objektiivisesti perustelemaan, että Raamattu olisi mitenkään erikoisempi noiden joukossa


      • Anonyymi

        Ei ollut mitään vedenpaisumusta. Kukaan ei siis tiedä millaista oli "ennen" tai "jälkeen" vedenpaisumuksen. Se on ihan sama kuin kysyisi, että millaista oli ennen kuin Tylypahka rakennettiin.

        On hauskaa ja huvittavaa huomata, miten paljon teitä valehtelevia kreationisteja syö se, että Raamatussa on arkin tiivisteenä on mainittu "maapiki". Paljon kertoo se, mistä tämä selittelyntarve johtuu. Raamattu ei sano etteikö maaöljyä olisi ollut ennen tarujen vedenpaisumusta, mutta JOKU VALEHTELEVA KREATIONISTI meni möläyttämään, että maaöljy syntyi vedenpaisumuksen maalla peittämistä puista, joten HE OVAt VALMIIT KYSEENALAISTAMAAN RAAMATUN SANAN, mutta ei omaa ad-hoc selityksiään.

        Tämä esimerkki osoittaa KIISTATTA sen, että kiihkouskovaisuus on ennen kaikkea uskoa omaan erehtyymättömyyteen. Jos käy ilmi, että kreationisti ja Raamattu on eri mieltä, niin sen pahempi Raamatulle. Jos todellisuus ei sovi kreationistin käsityksiin, niin sen pahempi todellisuudelle.


      • Anonyymi

        Minkälaisella paatilla ne väkivaltaiset ihmiset oli seilanneet mukanaan eläimet ym.? Eli mistä ne syntyi?


      • Anonyymi
        Anonyymi kirjoitti:

        Miksi pitäisi Raamattu tuntea? Uskonnollisia pyhiä kirjoituksia on lukuisia ja ne ovat keskenään ihan yhtä hyviä. Et pysty objektiivisesti perustelemaan, että Raamattu olisi mitenkään erikoisempi noiden joukossa

        Raamattu (ja mikä tahansa lähde) on hyvä tuntea, jos esittää mielipiteitä siitä. Sitä paremmin, mitä yksityiskohtaisemmin ottaa kantaa. Ei muuten.


      • Anonyymi
        Anonyymi kirjoitti:

        Raamattu (ja mikä tahansa lähde) on hyvä tuntea, jos esittää mielipiteitä siitä. Sitä paremmin, mitä yksityiskohtaisemmin ottaa kantaa. Ei muuten.

        Jos ei tunne, niin käy kuin kreationistille, joka selitti, että maaöljy syntyi vedenpaisumuksen peittämistä kasveista, vaikka Raamatun mukaan Nooan arkki tiivistettiin maaöljypohjaisella piellä jo ennen vedenpaisumusta. Sellainen on kamalan noloa.


      • Anonyymi
        Anonyymi kirjoitti:

        Raamattu (ja mikä tahansa lähde) on hyvä tuntea, jos esittää mielipiteitä siitä. Sitä paremmin, mitä yksityiskohtaisemmin ottaa kantaa. Ei muuten.

        Toki ne kohdat, vaikkapa luomiskertomus, on tullut luettua. Eihän noista muuten osaisi kysellä.

        Eikö Raamatussa mm. todeta, että vesi peitti korkeimmatkin vuoret?

        Tuo on muuten hyvä ohje kreationisteille. Täällä kreationistit esittelevät hyvin menestyksekkäästi omaa umpitollouttaan luonnontieteissä. Kannattaisi opiskella asiasta, josta yrittää keskustella.


      • Anonyymi
        Anonyymi kirjoitti:

        Jos ei tunne, niin käy kuin kreationistille, joka selitti, että maaöljy syntyi vedenpaisumuksen peittämistä kasveista, vaikka Raamatun mukaan Nooan arkki tiivistettiin maaöljypohjaisella piellä jo ennen vedenpaisumusta. Sellainen on kamalan noloa.

        Sana joka on ymmärretty pieksi, on alkutekstissä hepreassa suojausaine, päällyste. eikä sitä sen tarkemmin selitetä.

        Piki on kääntäjien käsitys.

        Samoin puulaji GOOFER on meille tuntematon nimi, vaikka sen onkin kuviteltu tarkoittavan honkaa.

        Käytetään niitä sanoja jotka ovat tunnetut lukijalle
        Samoin esimerkiksi sana lumivalkea on jossakin Afrikassa käännetty "valkea kuin kookospähkinän ydin".


      • Anonyymi
        Anonyymi kirjoitti:

        Sana joka on ymmärretty pieksi, on alkutekstissä hepreassa suojausaine, päällyste. eikä sitä sen tarkemmin selitetä.

        Piki on kääntäjien käsitys.

        Samoin puulaji GOOFER on meille tuntematon nimi, vaikka sen onkin kuviteltu tarkoittavan honkaa.

        Käytetään niitä sanoja jotka ovat tunnetut lukijalle
        Samoin esimerkiksi sana lumivalkea on jossakin Afrikassa käännetty "valkea kuin kookospähkinän ydin".

        Eli Draamattu ei olekaan absoluuttista jumalansanaa,vaan lähinnä seliseliä!
        Kiitos,tämä selvä!


      • Anonyymi
        Anonyymi kirjoitti:

        Sana joka on ymmärretty pieksi, on alkutekstissä hepreassa suojausaine, päällyste. eikä sitä sen tarkemmin selitetä.

        Piki on kääntäjien käsitys.

        Samoin puulaji GOOFER on meille tuntematon nimi, vaikka sen onkin kuviteltu tarkoittavan honkaa.

        Käytetään niitä sanoja jotka ovat tunnetut lukijalle
        Samoin esimerkiksi sana lumivalkea on jossakin Afrikassa käännetty "valkea kuin kookospähkinän ydin".

        Omaan erehtymättömyyteensä rakastuneet itsekeskeiset valehtelevat kreationistit eivät VOI SIETÄÄ SITÄ, että he ovat VÄÄRÄSSÄ missään osassa. Kuten sanottu, jos Raamattu ja kreationistin ad-hoc selitys on ristiriidaassa, niin SE PERKELEEN RAAMATTU ON VÄÄRÄSSÄ!

        Väitätkö sinä onneton valehtelija, että SINÄ osaat kääntää paremmin Raamatun kuin esimerkiksi vuoden 1992 käännöksen tehnyyt professoreista ja asiantuntijoista koostunut työryhmä? Onko omahyväisyydellesi edes taivas rajana?


      • Anonyymi
        Anonyymi kirjoitti:

        Omaan erehtymättömyyteensä rakastuneet itsekeskeiset valehtelevat kreationistit eivät VOI SIETÄÄ SITÄ, että he ovat VÄÄRÄSSÄ missään osassa. Kuten sanottu, jos Raamattu ja kreationistin ad-hoc selitys on ristiriidaassa, niin SE PERKELEEN RAAMATTU ON VÄÄRÄSSÄ!

        Väitätkö sinä onneton valehtelija, että SINÄ osaat kääntää paremmin Raamatun kuin esimerkiksi vuoden 1992 käännöksen tehnyyt professoreista ja asiantuntijoista koostunut työryhmä? Onko omahyväisyydellesi edes taivas rajana?

        Esimerkiksi sana AION on käännetty neljäkymmentä kertaa sanalla maailma, maailmanaika, mutta silloin kun siitä saadaan tukea kirkon käsitykselle, se käännetäänkin sanalla ikuisuus, ja ikuinen piina. Joten kustantaja päättää miten käännetään.

        Luen muuten alkutekstiä aina silloin tällöin muutaman luvun, kun en saa unta. Enkä tarvitse sanakirjaa.


      • Anonyymi
        Anonyymi kirjoitti:

        Esimerkiksi sana AION on käännetty neljäkymmentä kertaa sanalla maailma, maailmanaika, mutta silloin kun siitä saadaan tukea kirkon käsitykselle, se käännetäänkin sanalla ikuisuus, ja ikuinen piina. Joten kustantaja päättää miten käännetään.

        Luen muuten alkutekstiä aina silloin tällöin muutaman luvun, kun en saa unta. Enkä tarvitse sanakirjaa.

        Oletko mielestäsi enemmän omahyväinen kuin epärehellinen?


      • Anonyymi
        Anonyymi kirjoitti:

        Esimerkiksi sana AION on käännetty neljäkymmentä kertaa sanalla maailma, maailmanaika, mutta silloin kun siitä saadaan tukea kirkon käsitykselle, se käännetäänkin sanalla ikuisuus, ja ikuinen piina. Joten kustantaja päättää miten käännetään.

        Luen muuten alkutekstiä aina silloin tällöin muutaman luvun, kun en saa unta. Enkä tarvitse sanakirjaa.

        Enkä tarvitse sanakirjaa.

        - Se voisikin häiritä harmittavaisesti.


      • Anonyymi
        Anonyymi kirjoitti:

        Enkä tarvitse sanakirjaa.

        - Se voisikin häiritä harmittavaisesti.

        Sanojen merkityksen oppii tarkemmin lukemalla tarpeeksi. Esimerkiksi englannin kielen sana NICE ei ole käännettävissä yhdellä sanalla, vaan sen oppii asiayhteyksistä.

        Raamatunkäännöksissä on pieniä eroavaisuuksia, suurelta osalta siksi, että noudatetaan kunkin kirkkokunnan linjaa, eikä olla uskollisia alkutekstille.


    • Anonyymi

      Globalisaation selittäminenkin on vielä täysin kesken?

    • Anonyymi

      > Lähtötietoina on ainakin se, että vesi peitti kaikki maat ja korkeimmat vuoret, vesi tuli sateena 40 päivän ajan ja vuoden päästä vedet laskivat "normaali" tasolle.

      Raamattu kertoo veden vapautuneen suuren syvyyden lähteistä. Tätä varten pitää ymmärtää, että Raamatun mukaan alussa oli vain yksi manner, joka oli vedetty suuren vesimassan yli. Tulva ja vesisade syntyi kun tuo alkumanner murtui ja vajosi. Ja kun vedenpinta alkoi laskea, siihen oli kaksi syytä, maan painuminen suuresta vedenpaineesta ja seurauksena ollut jääkausi, joka sitoi vettä suuriin jäätikköihin.

      • Anonyymi

        Juu ei. Raamattu ei puhu mitään mantereiden määrästä. Valehteleva kreationisti keksii taas tarinaa omasta päästään ja pidäkkeettömästi lisäilee ja vääristelee "pyhän" Raamatun kertomuista. Jääkausi Raamatussa?!? Voi perse tätä valheiden määrää!


      • Anonyymi

        Jos jätetään Raamatun vääristelyt ja tuumitaan vähän mitä tuollainen mantereen vajoaminen geologian näkökulmasta tarkoittaisi. Noh sehän tarkoittaisi, että tuollaisen tapahtuman seurauksena elämä tosiaan tuhoutuisi, ehkä mikrobeja lukuunottamatta. Ja seurauksena olisi satojen vuosien ydintalvi. Nooan porukoineen olisi kuollut joko tukehtumalla, tsunameissa, ..., ja viimeistään kylmyyteen.

        Toki mielikuvituksessa voit siirrellä ja halkoa mantereita, mutta tuolla ei ole mitään tekemistä todellisuuden kanssa.

        Tämä kuvitelmatyyppi taisi olla jo listassa.


      • Anonyymi
        Anonyymi kirjoitti:

        Jos jätetään Raamatun vääristelyt ja tuumitaan vähän mitä tuollainen mantereen vajoaminen geologian näkökulmasta tarkoittaisi. Noh sehän tarkoittaisi, että tuollaisen tapahtuman seurauksena elämä tosiaan tuhoutuisi, ehkä mikrobeja lukuunottamatta. Ja seurauksena olisi satojen vuosien ydintalvi. Nooan porukoineen olisi kuollut joko tukehtumalla, tsunameissa, ..., ja viimeistään kylmyyteen.

        Toki mielikuvituksessa voit siirrellä ja halkoa mantereita, mutta tuolla ei ole mitään tekemistä todellisuuden kanssa.

        Tämä kuvitelmatyyppi taisi olla jo listassa.

        Kukin voi kuvitella mitä haluaa, se on vapaata.

        On totta, että esimerkiksi Azorin pyramidi on vajonnut sen jälkeen kun se on rakennettu, siis manner on siellä vajonnut, sitäpaitsi juuri siellä missä pitäisi muka muodostua uutta mannerta.

        Ei sen vajoamisen tarvitse aiheuttaa muuta kuin veden nousua toisilla kohdin. Ei se tuhoa muuta elämää kuin mitä Raamattu kertoo: ilmaa hengittävät eläimet.

        Sitten voi tietysti kuvitella, että Jumala, joka muuttaa koko maapallon pinnan muotoja, ei pystyisi tekemään sitä aiheuttamatta suurempaa katastrofia kuin Hän sallii.

        Kukin uskokoon miten haluaa! Eivät ateistit kuitenkaan edes tunnusta Jumalan olemassaoloa eikä voimaa.


      • Anonyymi
        Anonyymi kirjoitti:

        Kukin voi kuvitella mitä haluaa, se on vapaata.

        On totta, että esimerkiksi Azorin pyramidi on vajonnut sen jälkeen kun se on rakennettu, siis manner on siellä vajonnut, sitäpaitsi juuri siellä missä pitäisi muka muodostua uutta mannerta.

        Ei sen vajoamisen tarvitse aiheuttaa muuta kuin veden nousua toisilla kohdin. Ei se tuhoa muuta elämää kuin mitä Raamattu kertoo: ilmaa hengittävät eläimet.

        Sitten voi tietysti kuvitella, että Jumala, joka muuttaa koko maapallon pinnan muotoja, ei pystyisi tekemään sitä aiheuttamatta suurempaa katastrofia kuin Hän sallii.

        Kukin uskokoon miten haluaa! Eivät ateistit kuitenkaan edes tunnusta Jumalan olemassaoloa eikä voimaa.

        Miksi meriargeologit eivät tunne mitään "Azorien pyramidia"? Miksi tuollainen sensaatiomainen ei ole saanut tutkijoita tuonne perkaamaan? Salaliitto, heh?

        Vain muutama photoshopattu kuva ja jotkut nielee tuon huuhaan.


      • Anonyymi
        Anonyymi kirjoitti:

        Kukin voi kuvitella mitä haluaa, se on vapaata.

        On totta, että esimerkiksi Azorin pyramidi on vajonnut sen jälkeen kun se on rakennettu, siis manner on siellä vajonnut, sitäpaitsi juuri siellä missä pitäisi muka muodostua uutta mannerta.

        Ei sen vajoamisen tarvitse aiheuttaa muuta kuin veden nousua toisilla kohdin. Ei se tuhoa muuta elämää kuin mitä Raamattu kertoo: ilmaa hengittävät eläimet.

        Sitten voi tietysti kuvitella, että Jumala, joka muuttaa koko maapallon pinnan muotoja, ei pystyisi tekemään sitä aiheuttamatta suurempaa katastrofia kuin Hän sallii.

        Kukin uskokoon miten haluaa! Eivät ateistit kuitenkaan edes tunnusta Jumalan olemassaoloa eikä voimaa.

        Kertokaapa (tai kerropa) nyt vielä, onko kreojen virallinen linja että kaikki muut uskovaiset kuten kristittyjen eri ryhmät, ovat siis ateisteja? Minkä perustelun annatte sille?
        Pitäisiköhän tästä tehdä oma toppiikkinsa


    Ketjusta on poistettu 0 sääntöjenvastaista viestiä.

    Luetuimmat keskustelut

    1. Pupuhuhdasta löytyi lähes sadan kilon miljoonalasti huumeita

      Pupuhuhdasta löytyi lähes sadan kilon miljoonalasti huumeita – neljä Jyväskylän Outlaws MC:n jäsentä vangittu: "Määrät p
      Jyväskylä
      56
      1836
    2. Persut petti kannattajansa, totaalisesti !

      Peraujen fundamentalisteille, vaihtkaa saittia. Muille, näin sen näimme. On helppo luvata kehareille, eikä ne ymmärrä,
      Maailman menoa
      48
      1619
    3. Ei luottoa lakko maahan

      Patria menetti sovitun ksupan.
      Suomen Keskusta
      52
      1564
    4. Nähtäiskö ylihuomenna taas siellä missä viimeksikin?

      Otetaan ruokaöljyä, banaaneita ja tuorekurkkuja sinne messiin. Tehdään taas sitä meidän salakivaa.
      Ikävä
      5
      1507
    5. Sinäkö se olit...

      Vai olitko? Jostain kumman syystä katse venyi.. Ajelin sitten miten sattuu ja sanoin ääneen siinä se nyt meni😅😅... Lis
      Ikävä
      6
      1495
    6. Housuvaippojen käyttö Suomi vs Ulkomaat

      Suomessa housuvaippoja aletaan käyttämään vauvoilla heti, kun ne alkavat ryömiä. Tuntuu, että ulkomailla housuvaippoihin
      Vaipat
      6
      1405
    7. Hyvää yötä ja kauniita unia!

      Täytyy alkaa taas nukkumaan, että jaksaa taas tämän päivän haasteet. Aikainen tipu madon löytää, vai miten se ärsyttävä
      Tunteet
      8
      1306
    8. Lepakot ja lepakkopönttö

      Ajattelin tehdä lepakkopöntön. Tietääkö joku ovatko lepakot talvella lepakkopöntössä ´vai jossain muualla nukkumassa ta
      12
      1271
    9. Revi siitä ja revi siitä

      Enkä revi, ei kiinnosta hevon vittua teidän asiat ja elämä. Revi itte vaan sitä emborullaas istuessas Aamupaskalla
      Varkaus
      4
      1163
    10. Kello on puoliyö - aika lopettaa netin käyttö tältä päivältä

      Kello on 12, on aika laittaa luurit pöydälle ja sallia yörauha kaupungin asukkaille ja työntekijöille. It is past midni
      Hämeenlinna
      4
      1128
    Aihe